You are on page 1of 227

Study Guide

World History
INSTRUCTIONS TO STUDENTS 1
LESSON ASSIGNMENTS 9
LESSON 1: FORAGERS, FARMERS,
AND BUILDERS 11
LESSON 2: THE AXIAL AGE AND
FITFUL TRANSITIONS 41
LESSON 3: CONTACTS, CONFLICTS,
AND THE CRUCIBLE 81
LESSON 4: CONVERGENCE, DIVERGENCE,
AND GLOBAL ENLIGHTENMENTS 119
LESSON 5: FRUSTRATIONS OF PROGRESS;
CHAOS AND COMPLEXITY 167
SELF-CHECK ANSWERS 215
iii
C
o
n
t
e
n
t
s
C
o
n
t
e
n
t
s
INTRODUCTION
Welcome to your course in World History! Youre about to
take an amazing journey from the earliest evidence of human
origins to the world we all share today. The value of your
journey will be directly proportional to how much effort you
put into study and reflection. But, along the way, keep this
in mind: Your knowledge of history will do more than make
you witty at parties or prepare you to be a contestant on
Jeopardy. Historical knowledge will help you better under-
stand current events, making you a more informed citizen.
At the same time, your knowledge of history will help you
better understand the contradictions of human nature and
make wiser life choices.
OBJECTIVES
When you complete this course, youll be able to
n Discuss the emergence and expansion of Homo sapiens
based on paleontological and archaeological data and the
development of herding and farming following the end of
the last Ice Age
n Describe the acceleration of social and cultural change
as organized states emerged along the great river valleys
of Africa and Eurasia
n Discuss the succession of early civilizations across the
globe, including those that emerged in Asia, Africa, the
Middle East, the Mediterranean, and the central high-
lands of Mexico
n Describe and discuss the major features of the axial age,
including the important schools of religion and philosophy
as well as the nature of the great empires, such as those
of Persia, Rome, India, and China, in relationship to
lesser states in Japan, Korea, and Mesoamerica
1
I
n
s
t
r
u
c
t
i
o
n
s
I
n
s
t
r
u
c
t
i
o
n
s
n Describe and discuss forces that challenged established
empires, such as those of Rome and China, in response
to the rise of the Muslim world and the incursions of
nomadic barbarian invaders from northern Europe and
the Asian Steppes
n Explain important influences that accompanied of the
rise of the great world religions, including Christianity,
Islam, and Buddhism
n Discuss factors that prevented geographically isolated
societies, such as those of sub-Saharan Africa, Australia,
and the Americas, from developing stable, long-term
states or empires
n Describe and discuss the impact of nomadic peoples who
lived along the boundaries of the Islamic world, the
Byzantine Empire, and imperial China between 1000 and
1200 C.E.
n Explain the impact of the Asian Mongols on Eurasian
societies during the thirteenth and fourteenth centuries
n Discuss and explain factors, such as climate change and
the bubonic plague, that adversely impacted Europe and
China, but that provided opportunities to areas beyond
the plague zone, such as in India, Southeast Asia, and
western Africa
n Describe and outline the rise of new empires during the
sixteenth and seventeenth centuries as the European
Age of Discovery began an era of Western-dominated
imperialism and colonialism, consequently producing an
ecological revolution as new sources of wealth and food
were transported from the New World to the Old World
n Discuss and describe the impact of revolutionary devel-
opments that pitted the interests of religion against a
rising tide of humanism and the accelerated importance
of science in the West, thus triggering political and social
changes across the globe during the 1600s and beyond
Instructions to Students
2
n Describe and explain the era of global enlightenments
between 1700 and 1800, relating Enlightenment influ-
ences to urbanization and other population trends,
expanded interaction between global regions, and the
spread of European Enlightenment ideas beyond their
immediate revolutionary impact on European societies
n Describe significant developments since the beginning of
the twentieth century, including new energy technolo-
gies, the rise of industrialization, new approaches to
agriculture, and changes in working conditions
n Discuss and describe significant nineteenth-century
global developments, including the expansion of
Western-dominated empires across the globe, the rise of
nationalism, and new faces of radicalism in the political
sphere
n Discuss and describe key issues of the twentieth cen-
tury, including the global impact of Western science, the
two World Wars, the Cold War, and the global trend
toward independence (decolonization) among former
Western colonies
n Describe various forces that have led to the current era
of globalization in the context of burgeoning population
growth, regional conflicts impacting civilian populations,
and pressing issues related to climate change and
ecological destruction
COURSE MATERIALS
This course includes the following materials:
1. This study guide, which contains an introduction to your
course, plus
n A lesson assignments page with a schedule of study
assignments, as well as projects for the six lessons
youll complete during this course
n Assignment lessons emphasizing the main points in
the textbook
Instructions to Students
3
n Self-checks and answers to help you assess your
understanding of the material
2. Your course textbook, Felipe Fernndez-Armestos The
World: A Brief History, which contains the assignment
reading material
3. A DVD-ROM that supplements your course textbook
To complete this course successfully, youll need to do the
following:
1. Read and follow the study guide. Its designed to
complement your work with the textbook.
2. Study the assigned material in your textbook.
3. Use this guide as a tool for review and for completing
the required self-checks. The 30 chapters of the textbook
are divided into 10 parts. A self-check is provided for
each part.
4. Take all lesson examinations.
KNOW YOUR TEXTBOOK
Success in this course depends on your knowledge of the
textbook. For that reason, you should take some time to look
through it from front to back. Get a feel for how the material
is arranged and presented. The following are some of the key
features of your textbook.
Front Matter
n The Brief Contents are found on page iii.
n An extended table of contents is provided on pages
ivxviii.
n A list of maps featured in the textbook is provided on
pages xixxxi.
n Introducing the World on pages xxiixxx offers insight
into the authors approach to his textbook and acknowl-
edges people and academic institutions that contributed
to his work on it.
Instructions to Students
4
n You can learn a bit about the author, Felipe Fernndez-
Armesto, on page xxxi.
Content Features
n Each part of the book is introduced with engaging graphics,
a list of the chapters included in that part, and two time
lineson for environmental events and another for cultural
events. You can use the time lines to keep yourself oriented
as you work your way from the past toward the present.
Chapter Features
n Chapter topics are listed at the top of the first page of
each chapter.
n A sidebar (feature box) on the next page challenges you
with a set of focus questions. Study them to better
understand what you should get from the chapter.
n Making Connections features offer information that
supplements the main chapter content.
n Maps used for various purposes are a major feature of
each chapter.
n Illustrations and their extended captions should be
studied as carefully as the main textbook material.
n Small feature boxes pop up here and there, usually to
help elaborate the time lines that head each part of the
textbook.
n A Chronology feature at the end of each chapter provides
dates for the material covered in the chapter.
n At the end of each chapter, youll find mind-teasing
questions under the heading Problems and Parallels,
along with references to important Documents in Global
History and a Read On feature that guides you to
important historical sources relevant to that chapter.
Instructions to Students
5
End Matter
n A glossary is provided on pages G-1 through G-9. You
can refer to it at any point it you become a bit hazy
about theories, concepts, or terms.
n A Note on Dates and Spelling is on page DS-1
n Notes for key chapter sources begin on page N-1.
n Credits for each chapter begin on page C-1.
n The textbooks index begins on page I-1.
n A list of the contents on the DVD-ROM that accompanies
the textbook is provided on pages DVD-1 through DVD-10.
A STUDY PLAN
Here are some time-tested ideas for getting the most from
your course.
1. Set aside a regular time for reading and writing. Write
down your intended schedule for reading and writing.
You might want to use a wall calendar to show what
you need to do and when. Check off assignments as
you complete them to see your progress.
2. Read everything twice, or at least review after careful
reading. No one gets everything on the first reading. Use
the questioning method (posing each topic heading as a
question to be answered) to test your understanding as
you review.
3. Dont look up answers in the key before you complete
the self-checks at the end of a chapter. Not only is that
dishonest, it also defeats the purpose of the exercises.
Instructions to Students
6
4. Give yourself credit for completing each assignment.
Your work and self-discipline will take you through
this course. You deserve the credit. So give yourself
a pat on the back as you complete each assignment.
5. If you have any questions, e-mail your instructor.
Instructions to Students
7
NOTES
Instructions to Students
8
9
A
s
s
i
g
n
m
e
n
t
s
A
s
s
i
g
n
m
e
n
t
s
Lesson 1: Foragers, Farmers, and Builders
For: Read in the Read in
study guide: the textbook:
Assignment 1 Pages 1113 Pages 225
Assignment 2 Pages 1418 Pages 2647
Assignment 3 Pages 1927 Pages 4873
Assignment 4 Pages 2732 Pages 7495
Assignment 5 Pages 3340 Pages 96121
Examination 007678 Material in Lesson 1
Lesson 2: The Axial Age and Fitful Transitions
For: Read in the Read in
study guide: the textbook:
Assignment 6 Pages 4147 Pages 122149
Assignment 7 Pages 4856 Pages 150177
Assignment 8 Pages 5765 Pages 178209
Assignment 9 Pages 6671 Pages 210235
Assignment 10 Pages 7280 Pages 236259
Examination 007679 Material in Lesson 2
Lesson 3: Contacts, Conflicts, and the Crucible
For: Read in the Read in
study guide: the textbook:
Assignment 11 Pages 8187 Pages 260285
Assignment 12 Pages 8796 Pages 286311
Assignment 13 Pages 97103 Pages 312341
Assignment 14 Pages 103109 Pages 342373
Assignment 15 Pages 109118 Pages 374407
Examination 007680 Material in Lesson 3
Lesson Assignments
10
Lesson 4: Convergence, Divergence, and Global
Enlightenments
For: Read in the Read in
study guide: the textbook:
Assignment 16 Pages 119124 Pages 408433
Assignment 17 Pages 125132 Pages 434461
Assignment 18 Pages 132137 Pages 462487
Assignment 19 Pages 137146 Pages 488513
Assignment 20 Pages 147150 Pages 514537
Assignment 21 Pages 151156 Pages 538561
Assignment 22 Pages 156165 Pages 562585
Examination 007681 Material in Lesson 4
Lesson 5: Frustrations of Progress; Chaos and
Complexity
For: Read in the Read in
study guide: the textbook:
Assignment 23 Pages 167172 Pages 586613
Assignment 24 Pages 173178 Pages 614637
Assignment 25 Pages 178183 Pages 638663
Assignment 26 Pages 184192 Pages 664689
Assignment 27 Pages 193196 Pages 690717
Assignment 28 Pages 197204 Pages 718747
Assignment 29 Pages 204208 Pages 748773
Assignment 30 Pages 208214 Pages 774796
Examination 007682 Material in Lesson 5
Note: To access and complete any of the examinations in this study
guide, click on the appropriate Take Exam icon on your My Courses
page. You should not have to enter the examination numbers. These
numbers are for reference only if you have reason to contact Student
Services.
11
L
e
s
s
o
n

1
L
e
s
s
o
n

1
Foragers, Farmers,
and Builders
FORAGERS, FARMERS
The first part of Lesson 1 includes Chapter 1, Out of the Ice:
Peopling the Earth, and Chapter 2, Out of the Mud:
Farming and Herding after the Ice Age. Be sure to take
advantage of the time line found in the Part 1 opening
spread.
ASSIGNMENT 1: OUT OF THE
ICE: PEOPLING THE EARTH
Read this assignment. Then read the Part 1 opening spread and
Chapter 1, pages 225, in your textbook.
So You Think Youre Human
Humans are genetically related to anthropoid apes, such as
the chimpanzee, the gorilla, and the gibbon, as well as to a
variety of related, extinct hominids. Hominids are creatures
with humanoid or humanlike physical characteristics. The
earliest evidence of physically modern humans dates to about
150,000 years B.C.E (meaning Before the Current Era) in a
part of Africa now known as Ethiopia. The most recent of our
genetic cousins were the Neanderthals. As youll learn, their
physical characteristics and their level of social development
were very similar to what we find in Homo sapiens. Indeed,
evidence reveals that they lived more or less side-by-side
with their human cousins. For reasons that remain uncertain,
Neanderthals vanished from the scene about 30,000 years ago.
World History
12
Out of Africa
Current evidence suggests that our species evolved over a
period of about 4 million years from hominids that occupied
areas around the Great Rift Valley of northeast Africa. In any
case, a peculiar fact may fascinate you. Regardless of where
we find modern humans, anywhere on the globe, they have
the same basic genetic makeup. So, how did that happen?
See Map 1.1 on pages 1011 of your textbook.
Based on genetic evidence, all living humans trace their
ancestry to a prototypical breeding female, whom scientists
have named Eve, who lived in present-day Ethiopia about
150,000 years ago. In any case, heres the evidence in a
nutshell: The cells of modern humans contain something
called mitochondrial DNA. Female mitochondrial DNA changes
(mutates) over time at a known rate. By looking at DNA
samples of human populations around the planet, scientists
have been able to work out the likely track of human migra-
tions out of Africa.
The Last Great Ice Age
For the last few millions of years or so, Earth has been
subject to a 100,000-year cycle of cooling and warming. A
number of ice ages and so-called interglacial periods have
occurred over this long span of time. The peopling of the
Earth occurred during a period of intense climatic challenges
that accompanied the last great Ice Age.
Study Map 1.2 on pages 1415 to better understand how the
continents were impacted by the Ice Age that began to thaw
150,000 years ago.
As the polar cap began to shrink and the deep carpet of ice
began to retreat in the Northern Hemisphere, herds of migrating
mammals extended their range northward. Humans followed the
herds into the great expanse of the thawing tundra. Migrating
Ice-Age humans (like their Neanderthal cousins) thrived on the
high-calorie fat from Ice-Age mammals. Ice-Age foragers, such
as Cro-Magnon man in Europe, experienced Ice-Age affluence
Lesson 1
13
in that their diet was highly nutritious. Indeed, at least in
terms of caloric intake, human diets wouldnt be as nutri-
tious again until well into modern times.
For archeologists and paleontologistsscientists who study
the physiology and evolution of ancient speciesIce-Age
globalization refers to the fact that artifacts as well as reli-
gious practices have been found to follow the same patterns
wherever evidence of Ice-Age humans has been found. In
effect, the archaeological evidence indirectly supports the
thesis of common genetic heritage among early human
migrants out of Africa.
However, one of the mysteries of human migration around
the globe involves the peopling of North and South America.
Substantial evidence suggests that humans were in the New
World from 8,000 to 13,000 B.C.E. However, although the
issue is barely touched on in your textbook, some recent
archaeological findings seem to support much earlier human
colonization in areas as far south as Chile. In light of that
fact, you should keep in mind that all history, and certainly
humankinds ancient history, is interpretive and not neces-
sarily exhaustive. Mysteries and unanswered questions
abound.
Study Map 1.3 on page 20 for graphical information on the
peopling of the New World.
Survival of the Foragers
As Earths climate warmed after the Ice Age, humans turned
away from foraging and hunting to pursue husbandry and agri-
culture. Husbandry is the herding and breeding of animals.
Agriculture is the systematic planting and farming of food
crops. Climate change certainly played a part in this process.
Deciduous forests spread after the Ice Age. As that happened,
sources of edible plants increased in regions that included rich
temperate forest soils suitable for planting crops. Husbandry
in regions previously devoted to foraging and hunting may
have helped offset seasonal food-shortage problems related
to animal migration patterns. People may also have turned
to husbandry and agriculture for social reasons.
World History
14
ASSIGNMENT 2: OUT OF THE
MUD: FARMING AND HERDING
AFTER THE ICE AGE
Read this assignment. Then read Chapter 2, pages 2647, in
your textbook.
There are two key ideas to take away from this chapter. First,
with the advent of husbandry, Darwinian natural selection
gave way to something radically different. As animals were
consciously bred to favor particular characteristics, such as
docility among cattle and agile responsiveness among horses,
natural selection gave way to managed selection. Second, as
important a revolution as was the advent of husbandry,
the advent of agriculture was, so to speak, the really big
revolution. Complex hierarchical states, the organization of
societies into social classes, and the advent of imperialism all
followed on the rise of agriculture.
The Problem of Agriculture
Around 14,000 to 15,000 years ago, permanent settlements
arose in the Middle Eastern regions that include present-day
Turkey, areas south of the Caucasus, as well as parts of Iran,
Iraq, and Syria. These areas had an abundance of wild plants
and game animals such that it was possible to sustain stable
village communities without the bother of systematic farming.
In short, archaeologists have found evidence of permanent
houses and social arrangements in places where foraging
remained the main way to gather food.
Study Map 2.1 on page 29 of your textbook for information on
preagricultural settlements in the Middle East.
Husbandry in Different Environments
Early on, herding tended to develop in regions where plants
were either sparse or not digestible by humans but were
digestible by animals such as goats, cattle, or horses. By
contrast, where soils were suitable for farming, tillers made
Lesson 1
15
their living by farming as well as by way of husbandry. Thats
the issue in a nutshell. However, in this section, your chal-
lenge is become aware of the different kinds of herding and
tilling environments, which are described on pages 3238 of
the textbook.
Spend some quality time with the Making Connections
feature on page 31 of your textbook. Note the significant differ-
ences between the ways of life and worldviews of foragers
and farmers.
The Spread of Agriculture
How did agriculture spread? Diffusion is one likely cause.
Agricultural techniques and practices could have spread
through social or cultural contact, as seems to have been the
case for the spread of agriculture outward from Anatolia and
the Jordan valley. Diffusion could also have resulted from
migration or invasion (or both). Scenarios for diffusion offered
in your textbook include the possibility that migrants out of
Asia brought their agricultural practices with them into
Europe. The fact that the Indo-European languages found
today from India to England originated with these Aryan
migrants offers some support for that diffusion hypothesis.
Another possibility, of course, is that agriculture was invented
in different places at different times. In fact, its likely that
both of these things happened.
Map 2.2 on pages 4041 of your textbook deserves you careful
attention. Note the locations of swamplands, uplands, and
floodplains. Study the Early Crop Sites table on page 40 of
your textbook.
So Why Did Farming Start?
You textbook offers seven basic theories as to why food
production started:
n Population pressures: The advent of agriculture in an
area was often followed by population growth, which, in
turn, required more effort and energy devoted to large-
scale food production through organized farming. The
population-pressure hypothesis is called a stress theory.
World History
16
n Ecological abundance: In areas where the fishing is good
and the living is easy, people have leisure time to experi-
ment with plants. However, theres no necessary reason
to develop agriculture in such an environment.
n Politics: When a society develops a hierarchy of social
class and status, the people at the top tend to exhibit
their wealth and power through celebratory feasting. As
a result the little people have more pressure on them to
produce ever larger harvests.
n Cult agriculture: Fertility rituals represented by the cycle
of planting through harvest played a part in early human
social history. In effect, religious beliefs and practices
have may have inspired agriculture long before it became
a practical political necessity.
n Climatic instability: Earth became hotter and drier
13,000 to 11,000 years ago. In areas like the Middle
East, drier conditions favored grasses of the sort that
produce edible grains. As a result, people became
increasingly dependent on edible grains and learned how
to plant and harvest grass crops.
n Agriculture by accident: Your textbook refers you to a
common nineteenth-century theory that agriculture was
discovered by accident, probably by women, because
they were more likely to be plant foragers as opposed to
game hunters. Charles Darwin favored this theory.
n Production as an outgrowth of procurement: Improved
techniques for obtaining edible grains from grasses could
be a logical extension of plant foraging. In other words,
the work of procuring food may show us how to produce
more food.
Be sure to conclude your study of Chapter 1 with a careful
contemplation of the Chronology sidebar on page 45 of your
textbook.
Please complete Self-Check 1 now.
Lesson 1
17
Self-Check 1
At the end of each section of World History, youll be asked to pause and check your
understanding of what youve just read by completing a Self-Check exercise. Answering
these questions will help you review what youve studied so far. Please complete Self-
Check 1 now.
Indicate whether each of the following statements is True or False.
______ 1. Potatoes were first grown for food in Africa.
______ 2. Speculations that Asians may have migrated into Europe are supported by the fact that
Indo-European languages are found in early Europe.
______ 3. The last Ice Age was the last great era of what we now call globalization.
______ 4. Between 9,000 and 11,000 years ago, farming villages appeared in what is now called
Turkey.
______ 5. The Bantu-speaking peoples lived in North Africa.
______ 6. Neanderthals and Homo sapiens were essentially members of the same species.
______ 7. Its reasonable to assume that societies were more inclined to risk men in war because
women were necessary for reproduction and the care of infants.
Fill in the blank with the correct term.
8. River deltas that renew good soil for farming on a regular basis are called _______ plains.
9. The Scythians of the Asian steppe domesticated the _______ and developed the wheel and
axle approximately 6,000 year ago.
10. Evidence supports the hypothesis that the earliest Homo sapiens occupied regions of east
_______.
(Continued)
World History
18
Self-Check 1
11. Peoples of Mesoamerica developed farming that concentrated on growing maize, _______,
and squash.
12. In early human tribes and groups, the role of the _________ was to act as an intermediary
between humans and gods or spirits.
Answer each question in no more than four complete sentences.
13. What would climate instability have to do with the development of farming?
__________________________________________________________
__________________________________________________________
__________________________________________________________
__________________________________________________________
14. What is meant by the term Ice-Age affluence?
__________________________________________________________
__________________________________________________________
__________________________________________________________
__________________________________________________________
Check your answers with those on page 215.
Lesson 1
19
BUILDERS
The second part of Lesson 1 includes Chapter 3, The Great
River Valleys: Accelerating Change and Developing States;
Chapter 4, A Succession of Civilizations: Ambition and
Instability; and Chapter 5, Rebuilding the World: Recoveries,
New Initiatives, and Their Limits.
ASSIGNMENT 3: THE GREAT
RIVER VALLEYS:
ACCELERATING CHANGE AND
DEVELOPING STATES
Read this assignment. Then read the Part 2 opening spread and
Chapter 3, pages 4873, in your textbook.
Growing Communities, Divergent
Cultures
Remarkable differences separated societies that adopted agri-
culture as their main source of food. Why? You can think of
it like this: Agriculture sustains denser populationsmore
people can live in a limited space. As population density
increases, farming must be organized. Food must be available
for the populations day-to-day needs, and harvest surpluses
must be set aside for times when the harvest is poor. High
levels of social organization require some kind of lawfully
recognized elitepeople who are empowered to decide who
will do what.
Those are the common factors that appear with increasing
population density. However, just as there are often several
paths to the top of a mountain, there are all kinds of ways a
society may go about organizing itself, establishing norms of
behavior, differentiating classes of people, or inventing sacred
myths that justify the way the society is organized.
World History
20
For example, as population densities increase, people tend
to get sorted into categories. Social organization can be
horizontal (hierarchical) or vertical. In a hierarchical social
organization, people are organized into social classes, such
as ruling elites, bureaucrats who administer functions of the
state, and commoners, most of whom are farmers. In a
vertical social organization, people are organized by common
allegiances, such as clan membership, place of origin, or
shared mythical beliefs. In most societies, there has been
some kind of mixture of these two modes of social organization.
Large cities and states developed in the regions of Peru and
Mexico from about 2000 B.C.E. In Eurasia, significant socio-
cultural developments arose from the British Isles to areas
across Eastern Europe and beyond. The horse was first
domesticated around 5000 B.C.E. in the region of present-day
Ukraine. The first chariots appeared around 2000 B.C.E. in
the region of the southern Ural Mountains. The earliest mon-
umental stone buildings have been found on the island of
Malta between Sicily and North Africa of all places! Stone
circle monuments, such as Stonehenge in England and
similar monumental efforts in Western Europe, certainly
suggest complex societies with amazing technological skills.
Study Map 3.1 on page 55 to reflect on intensified settlements
in Western Eurasia between 5000 and 2000 B.C.E.
The Ecology of Civilization
Civilization has become a somewhat discredited term, in
part, because theres no sound evidence of distinct stages in
social evolution. Even so, we can use that term to refer to
societies that ambitiously altered their natural environment,
attempting to remodel the rest of nature to suit human pur-
poses (textbook, page 56)for better or for worse.
Lesson 1
21
The Great Floodplains
The author of your textbook refers to the ecology of civiliza-
tion with good reason. The four most ambitious societal
developments occurred where great rivers provided alluvial
flood plains that enriched the soil on a more or less regular
annual cycle.
The Ecology of Egypt
Egypt is the Nile; the Nile is Egypt. Thats a poetic way of
saying that without the regular flooding of the Nile, ancient
Egypt would never have existed. The Nile, originating from
the region of Lake Victoria, flows north to the Mediterranean.
Upper Egypt is defined by large measure by the rich soils and
wetlands of the Nile Delta. The Pharaoh Menes is credited
with uniting Upper and Lower Egypt around 3600 B.C.E. and
the first capital of united Egypt was Memphis, near the mod-
ern city of Cairo.
Map 3.2 on page 58 of your textbook shows the sources of
precious resources and the important trade routes in ancient
Egypt. Gold, turquoise, copper, and ivory were a fundamental
basis of Egyptian trade and wealth.
Egyptian agriculture was sufficient for feeding a large popula-
tion, but most people ate only moderately above subsistence
level.
The Indus Valley
The ancient Harappan civilization arose in the broad Indus
delta region. From an archaeological point of view, Harappan
civilization is poorly understood. This is the case in large
measure because the rivers of the Indus system have
changed course so many times and because the drying up of
the Saraswati River long ago seems to have sounded a death
knell for the Harappan peoples. Archaeologists have noted
the fine workmanship of Harappan seals (see illustrations on
page 58), but their writing system has been lost to history.
World History
22
Map 3.3 on page 59 of your textbook depicts the Indus river
system that existed mainly in present-day Pakistan. The city
of Harappa is located on one the five rivers that feed the Indus
from the Himalayas. The largest Harappan site is located at
Mohenjodaro on the banks of the Indus.
Mesopotamia
Unlike the fairly consistent annual flooding of the Nile, the
Tigris and Euphrates Rivers (located in present-day Iraq)
drain a land of climactic turbulence, regular sandstorms, and
uncertain alluvial deposits from river flooding. Nevertheless,
the first significant cities arose near the rivers deltas along
the Persian Gulf. One of these, the fabled Ur, is associated
with Abraham of the Hebrews. Later, Akkad and other city
states in northern Mesopotamia would dominate the region.
The ancient mythology of this area posited a sort of endless
tension between the Earth-mother goddess Nintu and the
assertive land-managing god Enki, even as both deities were
forced to submit to the wrath of storm gods.
Map 3.4 on page 60 of your textbook illustrates the so-called
Fertile Crescent that extends from the Tigris and Euphrates
valleys onto the edge of modern-day Turkey and then south-
ward along the eastern coast of the Mediterranean through
modern-day Syria, Jordan, and Lebanon. Needless to say, it
was more fertile thousands of years ago than it is today.
Early China
Egypt engaged in a steady trading relationship with
Mesopotamia. Mesopotamian city states were in touch
with and traded with the Harappan peoples. By contrast,
the complex society that arose along the course of the
Yellow River in China was isolated from the other great
river societies. Therefore, the distinctive cultural features of
Chinese society developed on their own, without significant
influence from the outside world. As a result of isolation,
the Chinese have tended to see themselves as inhabiting a
distinct and central place in the world.
Lesson 1
23
The tendency of societies to see themselves as unique and
special leads to a universal tendency toward ethnocentrism
and, often, xenophobia. Ethnocentrism is the tendency to see
ones societys beliefs and customs as superior to those of
other societies. Xenophobiafear of strangersrefers to
fear, loathing, and contempt associated with barbarian
outsiders.
The Yellow River gets its name from the loose, windborne
dusts from Mongolia. Sweeping out of the west, they color
both the soil and the river yellow. The annual spring flooding
of the Yellow River arrives torrentially from the highlands of
Shaanxi province. The flooding deposits immensely fertile
alluvial soilssoils sufficiently productive to feed what, even
in ancient times, was one of the densest populations any-
where on the globe.
The turbulent uncertainty of flooding along the Yellow River
brought with it the need for canals, dikes, and reservoirs as
insurance against drought. As you can imagine, these civil
engineering challenges would have required a high degree of
hierarchical social organization.
The primary food grain produced by Yellow River agriculture
was millet. (Millet isnt a single plant species but a family of
grasses that produce edible grains. Millet is about as nutri-
tious as wheat and tends to resist drought.) Over time, the
attractions of rice, and somewhat milder climatic conditions,
encouraged a gradual process of Chinese colonization
southward toward the Yangtze River.
Map 3.5 on page 61 shows the relationship of the Yellow River
valley to wet-rice growing areas in the seaward regions of the
Yangtze River.
Configurations of Society
Patterns of Settlement and Labor
Population density encourages a greater complexity in the
division of labor. Food surpluses permit people to engage in
specialized activities like weaving, pottery-making, carpentry,
building construction, stone masonry, and metallurgy.
World History
24
Craft specialization tends to be associated with trade, espe-
cially as advances in maritime technology lead to improved
travel over lakes, rivers, and oceans. Furthermore, trade
and conquest have ever tended to be associated in one way
or another. In that context, for example, developments in
metallurgy have, inevitably, been associated with ever more
efficient tools and ever more lethal weapons of war.
Study the Making Connections feature on page 63 of your
textbook. Compare the ecology of civilizations based on the
Nile, the Indus, the Tigris and Euphrates, and the Yellow
River in China. Remember that the term ecology refers to the
relationships of species within types of habitats (prairies, wet-
lands, river valleys). In the historians perspective, the focus is
on human relationships to an environment.
Politics
All of the great river valley civilizations lent themselves to
tyranny. This was the case because minutely controlled col-
lective efforts of individuals were vital to agriculture aimed at
producing surpluses and handling threats by way of flood
control. All of the river states featured divine kingship and
rigid social hierarchies that required everyone to be sub-
servient to the state.
The Egyptian State
The top survival priorities for Egypt were harvest storage as
insurance against drought and flood control. The guidance or
vision of the state was decreed in the sacred orders from the
mouth of the divine pharaoh, in some ways a representative
and in other contexts the virtual incarnation of the sun god,
Ra. Egypt also featured a literate priestly caste. Priests inter-
preted and decreed a divine moral code to which even the
pharaoh was more or less subservient.
Interestingly, monumental Egyptian statuary and architecture,
including the fabulous pyramids of Giza near present-day
Cairo, were seen in a somewhat different light in Egypts
later development. Tombs and monuments that had initially
Lesson 1
25
glorified kings increasingly represented an Egyptian concern
with moral virtue, the weighing of souls, and preparation for
an afterlife that rather resembled Egypts earthly character.
Statecraft in Mesopotamia
Mesopotamia was composed of a number of rival kingdoms,
all of which were typically represented by a single city, such
as Akkad in the north or Ur in the south. The kings decisions
were guided by oracles provided by court augurers, who tried
to foretell the future by reading messages perceived from the
entrails of sacrificed animals. The king could also seek his
own oracles by sleeping in temples and awaiting messages
through dreams; thus the will of the king wasnt necessarily
guided by religion or priestly decrees.
Consider the Epic of Gilgamesh to better understand the
nature of Mesopotamian kingship.
The First Documented Chinese State
Along the course of the Yellow River, the first documented
Chinese state was ruled by the Shang dynasty throughout
the second millennium (2000 to 1000 B.C.E.). The Shang state
was unitary; that is, it was considered a single entity that
embraced the identities and social roles of everyone in the
population. As in Egypt and Mesopotamia, kingship was con-
nected to the management of food and water, and, as in
those places, the Chinese king was, above all, a mediator of
the gods. In that sense, the king replaced the shaman (spirit
talker) of earlier tribal societies.
The king was charged with foretelling the future. Will there
be drought? Will the harvest be good? Will we defeat our
enemy? The means of divining the future was assigned to
diviners. Diviners would read the cracks and fissures of
heated animal bones and tortoise shells in rather the same
manner as Gypsies might read messages in tea leaves. As
oracle interpretations were etched onto the bones by the
diviners, they became oracle bones (or shells). Oracle bones
and tortoise shells, in turn, would provide archaeologists with
records of kingly deeds and significant events. In any case,
World History
26
although wealth and warfare were essential to successful king-
ship, success in meeting the needs of the people was also
considered important.
Study the chronology of major events in the sidebar feature on
page 68. Its important to keep time lines sorted out in your
mind as you move along in this course.
Ruling the Harappan World
A remarkable feature of Harappan ruins is the geometric
consistency of urban design and the nature of building
construction. The design of the largest Harappan cities
(Harappa and Mohenjodaro) is echoed in the layout of the
more than 1,500 villages excavated or studied throughout the
Harappan lands. Scholars speculate that the rigid consis-
tency of urban and village design suggests a rigid social class
structure, possibly even a caste system rather like the one
that would emerge in India. However, because theres little
evidence of kingly palaces or even clear archaeological
distinctions between rich and poor, others have speculated
that Harappa was either a republic or a theocracy. In any
case, because we have no knowledge of the Harappan writing
system, and because art objects are all but absent, none of
these speculations can be grounded in solid evidence.
The Politics of Expansion
All four of the great giver states expanded their territory
through conquest. In every case, the processes of expansion
would challenge the stability of the river valley societies.
Expansion has limits imposed both by geography as well as
by the finite resources available to any human enterprise.
Study the Making Connections feature on page 69 of your
textbook to contrast and compare politics and state power in
the great river valley societies.
The social organization required for survival in all of the great
river societies inclined them to tyrannyto autocratic rule.
Autocratic rule is all about subjugating commoners to the
will of the elites. Over time, the gap in wealth between haves
Lesson 1
27
and have-nots leads to civil discontent. By about 1500 B.C.E.,
the river states were being destabilized by revolt and resist-
ance from the common people.
Literate Culture
Writing has often been thought of a necessary precondition
for the rise of civilizations. A cuneiform system for writing on
clay tablets was developed in Mesopotamia. Both in Egypt
and in China, written language consisted of logograms.
Egyptian hieroglyphics and Chinese characters can be
thought of as icons, a bit like those we see on traffic signs,
which suggested an object or a concept. All of these writing
systems were well developed. However, your textbook will
challenge you to consider three reasons why we cant assume
that writing and advanced social development are more or
less joined at the hip. These reasons are discussed on
pages 7172 of the textbook.
ASSIGNMENT 4: A SUCCESSION
OF CIVILIZATIONS: AMBITION
AND INSTABILITY
Read this assignment. Then read Chapter 4, pages 7495, in
your textbook.
Features of the great river societies, including high population
densities, cities, intensive agriculture, and stratification, arose in
other places. The overall number of developed states increased
considerably, manifesting a wide variety in approaches to social
organization and resource production. Accelerating change was
characterized by warfare and by various kinds of environmental
catastrophes, from plague to famine to ecological destruction.
The Case of the Hittite Kingdom
The rise and fall of the Hittite Kingdom illustrates the major his-
torical themes of the second millennium (20001000 B.C.E.). The
heartland of the Hittite Kingdom (from 1800 to 1500 B.C.E.) was
northern Anatolia (Turkey). The Hittites called themselves the
World History
28
Children of Hatti. Hittite palaces, storehouses, town, cities,
and armies were all on a scale similar to that of Egypt or
Mesopotamia. Because the Anatolian environment was harsh,
the power of the Hittites was derived from war, tribute, and
trade over an astonishing range of influence.
Map 4.1 on pages 7879 of your textbook deserves careful
study. Pay attention to the trade routes as well as the kinds of
goods that were traded, such as granite, gold, copper, and
timber.
The Hittite king was seen as the sun gods earthly messenger
and was referred to as My Sun, just as one would call
Queen Elizabeth Her Majesty. The kings court featured an
elaborate array of clerks, administrative bureaucrats, military
officials, and concubines. Communication of the kings will
was recorded in writing and distributed throughout the
kingdom. The Hittite economy was based on unifying the
contributions of both farmers and herders. That strategy
helped offset the disadvantages of the relatively harsh envi-
ronment of Anatolia.
Hittites are often associated with the use of iron in warfare,
which they used in weapons and chariots. However, their
iron-smelting techniques were undependable and rudimen-
tary. Bronze (an alloy of tin and copper) was the basic metal
used for armor, weapons, tools, and crafted metal objects. In
any case, Hittite society was male dominated and fixated on
war. Women had their voice as court diviners, healers, and
seers. However, the great flocks of royal concubines were
treated as the rivets of the kingdom. Their charms were
offered as good-faith currency in trade and to the end of
forging alliances with the elites of bordering states. The
Hittite Kingdom went into decline around 1300 B.C.E. and
vanished from historical reference after about 1210 B.C.E.
Instability and Collapse in the Aegean
During the second millennium, two fascinating and some-
what mysterious civilizations arose in the southern reaches of
the Aegean Sea. The Minoan culture arose on the island of
Crete. Mycenae arose in southern Greece.
Lesson 1
29
Elaborate palace architecture was appearing on Crete by
2000 B.C.E. The lavishly furnished palaces were the domain of
elites. Food and forage were stacked in storehouses, and
maritime trade was a vital source of Cretan wealth. However,
wealth and food distribution was apparently inefficient. The
average commoner was likely to have a lifespan of about
40 years, just getting by on a diet barely sufficient for
sustenance.
Cretan society was ravaged by natural disasters, including
earthquakes and volcanic eruptions. The great palace in the
Cretan city of Knossos was rebuilt several times, the last time
being around 1400 B.C.E. Around that date, two patterns
emerged. The first is evidence of warfare, some of it possibly
internal. The second is that documents began to be written in
an early form of Greek.
At that point, the Minoan and the Greek Mycenaean cultures
became entangled. Gold-enriched royal tombs and fortified
cities are found in Mycenae from about 1500 B.C.E. Artifacts
and architecture are very like those of Crete. By 1100 B.C.E.,
the cities of Crete and Mycenae had been abandoned. Its not
clear how Cretan or Mycenaean culture is connected to the
rise of the Greek city-states hundreds of years later.
Map 4.2 on page 83 of your textbook will help you understand
the trade and cultural contacts that occurred around the east-
ern Mediterranean between 2000 and 1200 B.C.E. Notice that
trade routes extended all through Western Europe, reaching
the British Isles and Scandinavia. To the east, trade routes
connected Mycenae and Crete to Anatolia and Egypt.
A General Crisis in the Eastern
Mediterranean World?
Archaeological evidence from the second half of the second
millennium indicates repeated incursions and invasions.
Because their boundary areas were reasonably hospitable for
human habitation, Anatolia, Mesopotamia, and Harappa were
always subject to intermittent invasions. However, during
this same period, outsider invasions had a significant impact
elsewhere. Around 1500 B.C.E. invaders from North Africa
the Hyksosinvaded Egypt, occupying delta areas for quite
World History
30
some time before they were finally expelled under Pharaoh
Ramses III. Then, around 1190 B.C.E., mysterious seaborne
invaders called the Sea People threatened settled populations
all around the eastern Mediterranean.
These second-millennium invasions stemmed mainly from
unstable populations driven to migration by hunger. These
effects, in turn, resulted from competitive politics, the ecolog-
ical fragility of farmed lands, and, in some cases, disease and
natural disasters.
The Extinction of Harappan Civilization
In an era when empires were rising and falling like bowling
pins, the erosion and decline of the Harappan civilization
was unprecedented. Later Indo-European migrants into the
Harappan regions left us historical and mythological writing in
documents called the Rig Veda. Interpretations of that ancient
document (appearing in written form around 800 B.C.E.) suggest
that, along with the drying up of parts of the region, people
were driven to another land by disease. The likely disease in
a region where standing water accompanied irrigation efforts
would most likely have been malaria. In any case, historians
and archaeologists continue to sift through long-abandoned
ruins in search of answers as to the demise of the Harappan
civilization.
Conflict on the Yellow River
Empires rise and fall. Thats the way of the world. By 1100 B.C.E,
the extent and power of the Shang dynasty was fading. Around
this time, according to the evidence, the Zhou peoples, who first
appeared in the mountains of western China, were inspired
by Shang turtle-shell oracles to take up arms and pursue
conquest. Zhou herders became warriors and defeated the
Shang dynasty in a single battle in 1045 B.C.E. The Zhou
then annexed the lands of Shang as a colony, swore nominal
allegiance to the cultural values of Shang, and established
their capital somewhat to the north.
Zhou dominance in China lasted from about 1000 to
700 B.C.E. Thereafter, they would be increasingly challenged
by rival states. The Zhou invented the idea that Chinas
Lesson 1
31
hegemony was decreed by a Mandate of Heaven. Even though
China would be convulsed by warring states and political
rivalries over the centuries, the idea of China as a unitary
domain has remained.
State-Building in the Americas
In the Americas, empires arose following many of the same
patterns we see in the Old World. Along the Andean high-
lands and in central Mexico, ambitious efforts were made to
alter the natural environment in favor of a human vision.
Trade was a foundation of empire building.
In the Andean regions, various experiments in state-building
among culturally diverse peoples tended to be short-lived.
However, what these peoples achieved with regard to civil
engineering, urban design, and building construction is
remarkable. The city complex of Chavn de Huantar is offered
as an example and is described on page 89 of your textbook.
All along the Andesfrom Colombia to Bolivia and beyond
one finds remarkable ruins of complex societies.
Map 4.3 on page 90 of your textbook will help you visualize
the extent of Andean and Olmec civilizations. It will also give
you some understanding of why changes in Pacific Ocean
coastal currents, wrought by El Nio once or twice each
decade, created periods of torrential rains that posed serious
environment problems for Andean societies.
In Mesoamerica, in the region we now call Mexico, the Olmec
Empire arose along the swampy coastal areas west of the
Yucatan Peninsula during the second millennium. Around
1200 B.C.E., monumental stone buildings were appearing on
agricultural mounds built in drained areas of the wetlands.
These became the centers of urbanized regions with relatively
dense populations and sophisticated civil engineering that
included sewage systems, canals, and stone-paved plazas
and avenues.
World History
32
Characteristics of Olmec culture would appear in later
Mesoamerican states and empires. These included
n Intensive cultivation of maize, squash, and beans
n Imperial or expansionist agendas formulated by way of
shamanic rituals that guided the decisions of power
elites
n Powerful similarities in art and architecture.
Assessing the Damage
By around 1000 B.C.E., the Minoan and Mycenaean civiliza-
tions were gone. The Hittites were no more. Harappan society
had vanished. In Mesopotamia, the Sumerian peoples of Ur
and other sites closer to the Persian Gulf had fallen under
the sway to the Akkadian peoples of the upper regions of the
Tigris and Euphrates. The Akkadian language became
dominant, and the Sumerian language began to fade into
obscurity.
Egypt, in contrast, was a survivor. The Hyksos were driven from
Egypt at around the same time that Biblical accounts record the
emigration of Moses and the Hebrew tribes across the Sinai and
onward to regions of the eastern Mediterranean. The Sea Peoples
were finally defeated. However, Egypts empire was weakened.
Around 2000 B.C.E., Nubia was an independent state, although
its customs, architecture, and culture were essentially Egyptian.
Nubia was a source of trade, wealth, tribute, and mercenaries.
However, after 1000 B.C.E., records of Nubia vanish. The
Egyptian empire, weakened by invasions, had contracted.
Be sure to spend some time studying the Making Conn-
ections feature on page 93 of your textbook to consider the
instabilities that led to the downfall of kingdoms between
2000 and 1000 B.C.E. Also review the Chronology sidebar
on page 94 of your textbook.
Lesson 1
33
ASSIGNMENT 5: REBUILDING
THE WORLD: RECOVERIES, NEW
INITIATIVES, AND THEIR LIMITS
Read this assignment. Then read Chapter 5, pages 96121, in
the textbook.
The end of the second millennium was marked by fading
empires and what amounted to dark ages in areas where
the quality of life regressed or became stagnant. In this chap-
ter, youll be challenged to think about how the parts and
pieces of earlier cultures and empires were reassembled to
aid new experiments in empire-building. Youve already seen
that trade is a primary foundation of empire. In that context,
keep in mind that during first millennium the main approach
to trading with neighboring states was to colonize them.
Trade and Recovery in
the Middle East
Phoenicia
By around 1000 B.C.E., Phoenician ships were traveling, trading
with, and/or colonizing sites around the Mediterranean.
Phoenicias heartland consisted of ports at Byblos and other
cities in what is present-day Lebanon. In those days, the
fabled cedars of Lebanon still existed in abundance. Their
timber provided raw materials for Phoenicias seaworthy
ships as well as a vital trade commodity.
Around 500 B.C.E., a Phoenician bid for empire centered on
its resource-rich colony at Carthage, in North Africa. However,
the Phoenicians also had colonies in present-day Spain,
Sardinia, Sicily, and even West Africa. The Carthaginian bid
for empire would peak with the fabled assault on Rome by
a Carthaginian general named Hannibal. After three major
wars, Rome defeated Carthage in 146 B.C.E., and that was
the bitter end of Phoenicia. Well, almost. The Phoenicians
invented the alphabet. The Greeks, Romans, and others
World History
34
adopted this novel approach to writing. Today, the words
youre reading are one result of what Phoenicia left to the
world.
Assyria
The wealth of the Kings of Assur came largely from raiding,
looting, and pillaging. With the decline of Hatti, new conquest
opportunities presented themselves. By around 1000 B.C.E.,
Assyrian kings had forged a state along the upper Tigris. By
750 B.C.E., these kings were replacing local governors, in
effect building an empire based on domination.
The massive, monumental character of Assyrian buildings
and statuary aimed at intimidation. Assyrian rulers, as
illustrated by King Ashurbanipal, were self-aggrandizing
imperialists who presented themselves as intimates of the
gods and entitled lords of the world.
Babylon
Ancient Babylon was located on the banks of the Euphrates
River in Mesopotamia. It was a center of trade and political
power that became a prized part of the Assyrian Empire.
However, the people of Babylon rebelled on a regular basis.
The Assyrian kings Sennacherib and Ashurbanipal, in partic-
ular, responded with massacres, massive deportations, and
all kinds of destruction wrought on the cityscape. But, a bit
like Lexington and Concord during the American Revolution,
the name of Babylon became a rallying cry for insurgents
against Assyrian domination.
By the seventh century B.C.E., the Assyrian Empire was
overextended and running low on resources. A leader by the
name of Nabopolasar took advantage of that situation to
mastermind a Babylonian revival. As a result, Babylon
became an independent metropolis and reached the height
of its power under the fabled king Nebuchadnezzar (605
562 B.C.E.). Nebuchadnezzar was heavily invested in restoring
the ziggurats and other monumental architecture of ancient
Mesopotamia. Among his achievements was one of the
ancient wonders of the worldthe Hanging Gardens of
Babylon.
Lesson 1
35
Greece and Beyond
Early in the first millennium, at about the same time as the
Sea People were terrorizing the Mediterranean, migrants from
the north poured into Greece, devastating the literate culture
that may have been the heirs of Mycenaean culture. Emerging
from a dark age, when most Greeks were goat herders and
marginal farmers, city states arose at places like Athens and
Corinth.
These cities became centers of seaborne commerce based
largely on olives and olive oil that gave rise to a swarm of
trade-based Greek colonies around the Aegean Sea, the Black
Sea, and across the Mediterranean in parts of Italy, Spain,
Southern France, Cyprus, and the North Africa. Wherever
there was Greek colonialism, there were ports and settle-
ments devoted to seaborne trade and impacts from Greek art,
science, and culture.
Map 5.1 on pages 100101 of your textbook depicts the Middle
East and the Mediterranean between 1000 and 500 B.C.E.
Spend as much time as you need with this map. Locate the
extents of successive empires and the locations of colonies.
Pay attention to trade routes. Note the locations of copper, tin,
and gold mining.
Early Greek Society
Ancient Greeks declared their culture, their values and cus-
toms, to be unique. Outsiders were barbarians. However,
Greek culture was assembled from influences and outright
imitation of Near Eastern (Asian) and even Egyptian cultures.
As a result, the long-treasured idea that the Greeks gave
birth to Western civilization is looked on more skeptically
than once it was.
In part, this is the case because the West would be influ-
enced by Greek intellectual elites, such as Plato and
Aristotle, when in fact the ideals and customs of ordinary
Greeks were largely formed by myths and superstitions sur-
rounding their ideas of the gods as little better than
squabbling, jealous, vengeful sociopaths. Further, even
though Athenians did invent governance based on their idea
of democracy, Greek society was anything but liberal. Only
World History
36
privileged males were citizens. The 40 percent of Athenians
who were slaves had no voice and no choice. At the same
time, women were typically treated as little more than compli-
ant property.
The Spread of State-Building and City-Building
Historical interpretation tends to be selective. Schoolchildren
learn all about the Greeks but little or nothing about the peoples
of Thrace, Illyria, or North Africa or the fertile civilization of
the Etruscans of central Italy. On pages 106 and 108109 of
your textbook, youll learn a bit about these vibrant cultural
areas. But the main point is this: Phoenician and Greek colo-
nization turned the entire Mediterranean into a highway of
cultural exchange. The sharing of wealth and ideas led to
state- and city-building all around the Mediterranean.
Study the Making Connections feature on page 103 of your
textbook to compare conditions in Phoenicia, Assyria, and
Babylon that led to recovery in the Middle East and the
Mediterranean.
Empires and Recovery in China and South Asia
In China, the realm of the Zhou became increasingly decen-
tralized. Zhou rulers seemed more concerned with rituals and
appeasing the gods than practical governance. As a result,
subordinate kingdoms became insubordinate. Meanwhile,
nomadic peoples from the western mountains continued to
apply pressure along Chinas northern borders.
The Zhou moved eastward into a region that had been central
to the Shang, but which now consisted of 148 Zhou relatives
or nominees. That number was reduced by reconfiguration
and consolidation, but by the sixth century B.C.E. China was
a hodge-podge of jostling states in an era called the Period of
Warring States. Those years were marked by more or less
continuous warfare between states and violent political
instability within states.
In India, along the lush valley of the Ganges River, a distinc-
tive cultural region was present from about 1000 B.C.E. These
people seem not to have been successors of Harappan civi-
lization. Yet, as in the case of Harappa, very little is known
Lesson 1
37
about the customs, economy, or governance of the Ganges
peoples. However, the surviving literature of sages is both
abundant and remarkable. Among these, the Upanishads
recount oral traditions and wisdom that appears to have
originated long before they were written down around the
beginning of the first millennium.
The religious and philosophical ideas found in the Upanishads
are unique. They include a mystical belief in the unity of all
being under a primary deity called Brahman, the illusory
nature of sensory reality, and a doctrine of reincarnation.
The concept of reincarnation is that people progress through
many lifetimes to acquire virtues, gain enlightenment, and
return to the embrace of Brahman.
To the south on the Indian Ocean, a people called the
Sinhalese established an urbanized trade and shipbuilding
society covering much of the present-day island nation of Sri
Lanka.
Study Map 5.2 on page 110 of your textbook to locate China
during the Warring States Period, the area of barbarian intru-
sions, the settled area of the Ganges Valley in India, and the
ancient Sinhalese cultural area on Sri Lanka.
The Frustrations of Isolation
Technological and cultural innovations spread across Eurasia
and around the Mediterranean because trade and cultural
contact was facilitated by similar kinds of climates, estab-
lished trade routes, and navigable sea lanes and rivers. By
contrast, radical variations in climate zones, natural barriers
such as the Sahara Desert, and sheer distance greatly
restricted cultural exchange in the Americas, much of Africa,
and Australia.
Innovative cultural developments in North America were
found among the ancient Dorset culture that thrived in
Arctic environments, the Poverty Point people of the lower
Mississippi, the mound-builder culture of the Ohio Valley,
and in the present-day American Southwest, where maize-
bean-squash farming techniques of the Olmec were adopted.
World History
38
In Africa, we find several ancient areas of cultural innovation
and development. Nubia, located in the present-day Sudan,
abandoned the Egyptian language, adopting a Nubian lan-
guage around 750 B.C.E. The Nubian cities of Meroe and
Napata were centers of independently developed and sophisti-
cated iron smelting and fabrication.
By 1000 B.C.E., the Bantu-speaking farmers of central Africa
were expanding southward. Their agricultural productivity
provided surpluses that allowed them to trade with Nubia.
Hard-iron technology appears in West Africa (along the Niger
River) and then in the great lakes regions of Kenya around
500 B.C.E. At that time trade routes existed across the Sahara
from the slave-trading Garamantes of Libya to West Africa,
possibly laying a cultural framework for the development of
medieval African states like Mali and Songhai that were first
encountered by Europeans more than a thousand years later.
The monsoon cycles of the Indian Ocean have encouraged
trade for a long time. Important trading centers were present
in the fertile regions of the Arabian Peninsula (in todays
Yemen, Oman, and Bahrain) during the period of Assyrian
and Babylonian power. Trade routes across the Indian Ocean
to South Asia developed and evidence indicates that trade-
centered cities were present in the Horn of Africa.
Be sure to study the Making Connections feature on
page 118 of your textbook. It will help you to compare and con-
trast cultural developments and achievements in the Americas,
Africa, and across Eurasia. And, of course, spend some time
with the Chronology sidebar on page 119 of your textbook.
Now, review the material youve learned in this study
guide as well as the assigned pages in your textbook for
Assignments 15. Once you feel you understand the material,
complete Self-Check 2. Then check your answers with those
provided at the end of this study guide. If youve missed
any answers, or you feel unsure of the material, review the
assigned pages in your textbook and this study guide. When
youre sure that you completely understand the information
presented in Assignments 15, complete your examination
for Lesson 1.
Lesson 1
39
Self-Check 2
Indicate whether each of the following statements is True or False.
______ 1. In the fourth century B.C.E., Aristotle speculated that the state arose from a voluntary
alliance of families.
______ 2. Most Andean experiments in civilization didnt last very long.
______ 3. The rise of Mycenaean civilization preceded the rise of Minoan civilization on the island
of Crete.
______ 4. Through most of human history, societies were organized into social classes.
______ 5. The early Yellow Empire in China was capable of sustaining a rice-growing society.
______ 6. The common people of Egypt lived mainly on a diet of bread and grain-based beer that
provided nutrition levels only modestly above subsistence level.
______ 7. In the sixth century B.C.E., if you were a woman you would experience more advan-
tages in life if you were an Etruscan as opposed to a Greek or a Roman.
______ 8. The Harappan civilization was located in the region of modern-day Pakistan.
Fill in the blank with the correct term.
9. If legend can be credited, the first major _______ colony was at Carthage in North Africa.
10. The most famous relic of _______ literature is the Epic of Gilgamesh.
11. The Upanishads record how lesser gods challenged the supreme god called _______.
12. In the first millennium B.C.E., Phoenician and _______ colonization and trade turned the
Mediterranean into a highway of cultural exchange.
13. The Hyksos invasion of _______ around 1500 B.C.E. preceded the invasion of the mysterious
Sea Peoples around 1190 B.C.E.
(Continued)
World History
40
Self-Check 2
Answer each question in not more than four complete sentences.
14. How did the Hittite kingdom manage to thrive in the rugged terrain of Anatolia?
__________________________________________________________
__________________________________________________________
__________________________________________________________
__________________________________________________________
15. What are logograms?
__________________________________________________________
__________________________________________________________
__________________________________________________________
__________________________________________________________
16. Describe the nature of oracles used in ancient Shang China.
__________________________________________________________
__________________________________________________________
__________________________________________________________
__________________________________________________________
Check your answers with those on page 215.
The Axial Age and Fitful
Transitions
THE AXIAL AGE
The first part of Lesson 2 covers Chapter 6, The Great
Schools, and Chapter 7, The Great Empires.
ASSIGNMENT 6: THE GREAT
SCHOOLS
Read this assignment. Then read the Part 3 opening spread and
Chapter 6, pages 122149, in your textbook.
The axial age unfolded over a period of roughly 500 years.
Its called axial because it marked a profound shift in human
perspectives on the nature of reality. Axial-age thought
is associated with highly influential and innovative ideas
expressed though religious doctrines, innovative philosophies,
and approaches to scientific-analytical thought. The major
players during the axial age lived in China, India, Greece, and
southwest Asia (Tibet). They were linked to each other through
trade and cultural exchange that included the exchange of
ideas.
During the axial age, religious and creeds and practices
shifted from a focus on survival to beliefs and practices
focused on salvation and acquiring spiritual virtues, such as
selflessness and compassion. There was less focus on this
world and more concern for transcending the limitations of
mortality.
The Thinkers of the Axial Age
The key religious thinkers of the axial age are discussed on
pages 126127 and 130131 of your textbook. Youll learn
about Zoroaster and Zoroastrianism, Gautama Siddhartha
41
L
e
s
s
o
n

2
L
e
s
s
o
n

2
World History
42
and Buddhism, Confucius and Confucianism, Laozi (often
written as Lao-Tzu) and Daoism (often written as Taoism),
and Jesus and Christianity.
Innovators of what we would now call secular thought, thought
based on critical reason and analysis, appeared in all the regions
of the axial age, including China and India. However, for
Western Civilization, the cradle of philosophyand of scienceis
ancient Greece. Among the many innovative Greek thinkers,
the most imposing figures are Aristotle and his teacher Plato.
The quip that all Western philosophy amounts to footnotes
on Plato contains more than a grain of truth. However,
Aristotles
writings, when recovered in Western Europe centuries later,
would have the greater initial impact on European and
Western thought.
The Thoughts of the Axial Age
Religious Thinking
The axial age produced influential ideas about the concept of
divinity. Your textbook details the idea of a divine creator, the
idea of a single god, and the idea of an involved god, who was
engaged in the world.
n Creation: Early cosmic myths, like those of many tribal
peoples, tended to explain how the world came to be as
it is. The axial age abounded in mythological narrative
attempting to explain the creation of the world. The Genesis
story, wherein the Creator said, Let there be light, pro-
posed a divine entity who had always existed and who
produced the universe by an act of divine mind.
n Monotheism: The idea that theres a primary, unique, and
eternal God existed in many places. However, Judaism was
rather unique in proposing a single god, Yahweh, which
basically means the unnamable god. The monotheistic
idea would be absorbed into all the so-called religions of
the bookJudaism, Christianity, and Islam.
Lesson 2
43
Look at Figure 6.1 on page 133 to think about the Abrahamic
tradition as it originated and was then adopted by Islam and
Christianity.
n Divine love: The image of divine loveGods love for his
human creatureswas formulated late in the axial age
by Judaic thinkers, mainly based on the assumption that
God made man in his own image to serve as stewards of
His Creation. This idea would impact the theologies of
Christianity and Islam. Similar ideas would appear in
China and by way of Greek philosophy. However, the
Jain sect of India declared all life sacred and all crea-
tures brethren, and for Buddhists, respect for all life
didnt involve a god concept.
New Political Thinking
All through history there have been two opposing views of
human nature. Some thinkers have viewed humankind as
inherently evil. Others have proposed, like Anne Frank, that
people are good at heart. And, of course, these two opposed
points of view encourage rather distinct ideas about
the role of governance and the proper ordering of society.
Justifications for both points of view were sharpened during
the axial age. Pages 134137 of the textbook discusses the
political developments in light of these two viewspolitical
pessimism and political optimismin the axial age.
Challenging Illusion
The most striking feature of axial age thought all across
Eurasia was the recognition that what we know through our
physical senses enslaves us by way of false assumptions.
Youve already seen that the sacred literature of India
declared the world a dance of illusion. In the West, Plato
expressed this idea in The Republic with his famous parable
of the cave. Shackled in a cave of sensory illusion, we know
not the source of true light beyond our cave. We imagine
reality to consist of the play of shadows across the cave
walls. So, in effect, reality is hidden behind our illusory
imaginings.
World History
44
Mathematics
Early concepts of numeration and ratios seems first to have
developed as ancient sages learned to measure the periodici-
ties of celestial bodiesparticularly the seasonal cycles of the
sun and roughly monthly cycles of the moon. During the
axial age, however, sages began to explore the concepts of
number and ratio as clues to the nature of the cosmos itself.
Hindu sages devised the idea of cosmic cycles covering
thousands of years. In the West, during the sixth century
B.C.E. the Greek genius Pythagoras established basic con-
cepts of geometry and proposed that numbers and ratios are
real in themselves as well as keys to the structure of the cos-
mos. In short, the language of the invisible cosmos was to
be found in mathematics.
Reason
A major outcome of thinking about numbers and ratios was
rationalism. Rationalism is the doctrine that unaided reason
can identify and establish truths. This section is a mind-
bender. Youll need to study and reflect on the ideas you
encounter here. The Greek sage Parmenides (fifth century
B.C.E.) proposed that just as the idea of a perfect triangle can
exist only in the mind, the same principle applies to anything
at all perceived by the senses. What is real for us is actually
an interpretive construction of the mind.
Greek, South Indian, and Chinese sages proposed concepts
of logic. Logic is the proper use of reason. All sound arguments,
such as those presented by prosecutor in a court of law,
depend on logic. For the West, Aristotles concept of argu-
ment through syllogisms is the paramount example of logical
rigor. Reflect on this famous syllogism:
All men are mortal
Socrates is a man.
Therefore, Socrates is mortal.
Try creating a few syllogisms from any first premises you like.
Can you come up with an invalid syllogismone in which the
conclusion doesnt follow from the primary or derivative prem-
ise?
Lesson 2
45
Science
An interesting development during the axial age was a new
tendency to distinguish between the natural and the super-
natural. Prior to that sort of thinking, what we might think
of as natural or healing sciences were seen as magical in
nature. The new distinction plus the ideas that reason is
fooled by the senses and that truth can result from careful
observation and numerically accurate measurement led to
the advent of science.
In China, a prototype of scientific thinking appears to have
derived from Taoist teachings. Chinese science tended to be
weak on theory while being strong on practical technology.
In the West, the rise of science was a remarkable counter-
current to popular culture that placated whimsical gods and
assumed the existence of sprites, trolls, and nature spirits. In
this setting, Aristotle set standards that would guide Western
science for a very long time. For Aristotle, science was all
about gathering facts, connecting dots, and applying reason
in order to understand what can be observed.
Medicine
The notions that diseases, especially mental disorders,
resulted from demonic possession, witchcraft, and the
like would persist well into the eighteenth century, C.E.
Nevertheless, the scientific mode of thought gained ground
in the sphere of medicine during the axial age. In Greece in
the late fifth century B.C.E., followers of Hippocrates (called
Hippocratics) proposed that human disease and illness result
from an imbalance of the vital fluids. The theory was wrong,
but at least it was based on empirical observation. In India,
Susutra (sixth century B.C.E.), developed a system of medicine
based on diet and drugs. In China, Xunzi (d. 235 B.C.E.) main-
tained that illnesses and diseases resulted from naturalistic
causes.
World History
46
Skepticism
An outgrowth of scientific thinking led to the markedly
uncomfortable idea that the world and the universe itself are
meaningless; theres no cosmic purpose. Several examples of
sages who espoused such views include the Greek philoso-
pher Epicurus (d. 270 B.C.E.) and the first-century Chinese
sage Wangchong. Epicurus was a strict materialist. The world
as he saw it was composed of nothing but impermanent parts
and pieces (atoms) that move about in random ways. Such
a cosmos has no place for eternal spirits or immortal souls,
much less divine purpose. For Wangchong, humans are para-
sitic vermin who imagine a supernatural realm to give
themselves a cosmic significance that doesnt exist.
Skeptical philosophies that werent based in strict material-
ism included those emerging from Buddhism and Taoism in
the East and from the Greek philosopher Zeno of Citium who
lived and taught the Stoic philosophy in Athens in the fourth
century B.C.E. All of these doctrines proposed that some sense
of grace and integrity required detachment or indifference to
the slings and arrows of mortal life. Stoicism was widely
adopted in Roman times. A basic Stoic idea is that nature is
morally neutral. Only human acts can be thought of as either
good or evil. How we act determines who we are. And those
ideas would continue to influence Western elites to this day.
Axial Age-Axial Area: the Structures
of the Axial Age
Schools and sages of the axial age fell into four general, often
overlapping, categories:
n Professional intellectuals who sold their services as
teachers, often in the context of aspiring to public or pro-
fessional positions, but also to offer comfort or advice to
people in search of wealth, health and happiness
n Intellectuals who sought the patronage of monarchs and
elites, often in the capacity of political advisors (for
example, Aristotle)
Lesson 2
47
n Prophets or holy men who emerged from ascetic lives
with inspired messages for society
n Charismatic sages, who sought advocates for a visionary
political agenda that could either attract followers or be
imposed on a populace
Probably the main idea youll want to draw from this section is
that innovative thought happened in a dynamic context. Sages
disputed with each other even as their disputes inspired new
insights. Teachers gathered students into schools; students
questioned teachers. Some graduates formed new schools.
Intellectual advisors to kings and princes crafted clever argu-
ments to sway the thought of ambassadors, wealthy merchants,
and royal ministers. Its instructive that, for the most part,
Plato expressed his insights and ideas through writing out
dialogues. Platonic dialogues like the Symposium read a bit
like a script for a stage play.
We tend to think of schools as buildings. Actually, its best
to think of schools of thought as virtual pools of discourse
chat rooms without computers. Wealthy thinkers like Plato
founded academies that included boarding for students. But
more often than not, ideas were bought, sold, or repudiated
at dinner parties, in marketplaces, or in a public commons
such as the famed Agora of Athens. The word college comes
from the Latin collegium, referring to an association of academic
colleagues.
Wrap up your study of this chapter with a careful study of
the Making Connections feature on page 146 and the
Chronology sidebar on page 147 of your textbook.
World History
48
ASSIGNMENT 7: THE GREAT
EMPIRES
Read this assignment. Then read Chapter 7, pages 150177, in
your textbook.
Routes that Drew the
Old World Together
Sea lanes of the Mediterranean were relatively short, and
shore-hopping was relatively easy. But the Greeks and the
Phoenicians had to learn how to make headway by sailing
against the wind in order to navigate the full length of the
Mediterranean. By contrast, the alternating monsoonal wind
currents of the Indian Ocean facilitated long-range naviga-
tion. Mariners could sail to their destination and then get
back home again. For this reason, during the period of the
Great Empires, the Indian Ocean was a vital highway of trade
and commerce from Arabia and Africa all the way to China
and ports of call in Southeast Asia.
In 111 B.C.E., a Chinese garrison established an outpost
beyond Chinas western borders. Poetically, this region was
referred to as the throat of China, where trade routes to the
west gathered like veins gather in the neck of a beast or a
person. Collectively, roads branching out from this region
were referred to as the Silk Roads. Trade along these routes
generally passed through series of middlemen in market
centers, like Baghdad or the Persian capital at Susa, finally
arriving in regions of the Near East and all around the
Mediterranean. The reference to Silk Roads reminds us that
evidence of long-distance trade included the appearance of
silk in places like Egypt, Greece, and Roman Britain. The
quantity and bulk of goods transported by land was far less
than what could be transported by sea. Nevertheless, even
though sea routes were more important to global history,
land routes were also vital to trade and cultural exchange
during the axial age.
Lesson 2
49
The key to this section, beyond enjoying the textbook discus-
sion, is Map 7.1 on pages 154155 of your textbook. Study it
carefully to understand prevailing wind currents and sea and
land trade routes. Also, be sure to study the icons on the map
that represent important trade goods. Use the map in conjunc-
tion with the Making Connections feature on page 158 to get
a good overview of trade routed and their connections.
The First Eurasian Empire: Persia
The founding of the Persian Empire is credited to Cyrus the
Great. In around the middle of the sixth century B.C.E., Cyrus
laid the foundation of what would become the Persian Empire
by conquering the land of the Medes, one of the larger suc-
cessor kingdoms of the Assyrian Empire. Thereafter, the
empire was built by way of conquest and assimilation, taking
advantage of Persias central location along the highways of
trade that connected east and west.
At its greatest extent, the Persian Empire, joined with
Mesopotamia, extended to the coast of the Aegean in the
west, across much of Egypt in the south, and onward into
parts of India beyond the Indus to the southeast. As with all
empires, the lifeblood of Persia was trade and tribute. And
in that regard, the Persians werent fools. Their 1,700 miles
of roads and practices of wise governance tended to keep
conquered states relatively comfortable with their Persian
overlords. For example, Cyrus the Great rebuilt the Jewish
temple at Jerusalem, garnering the gratitude of the Hebrews.
The Greeks versus the Persians: The
Rise of Alexander the Great and the
Hellenic Empire
The Greeks looked down on the Persians, expressing con-
tempt for their respect for women and the luxurious lifestyle
of Persian elites. The Greeks were given to constant squab-
bling among rival city states. However, they managed to unite
long enough to defeat Persian efforts to conquer Greece in
490 and 489 B.C.E. The latter war featured the famous Stand
World History
50
of the 100, wherein a handful of Spartans delayed the
Persian advance out of Asia long enough for the Greeks
to rally sufficient strength to defeat the Persians.
During the period of the Persian wars, Athens was, more or
less, the dominant political player in Greek affairs. That
ended in 338 B.C.E. when King Phillip of Macedon united
northeastern Greece, including Athens, through conquest.
Phillips next plan involved pushing the Persians out of Asia,
but when Phillip was assassinated and his 19-year-old son
Alexander ascended to power, the Persian agenda under-
went radical change.
Alexander of Macedon is, without doubt, one of the most
colorful and astonishing characters of the axial age. His
ambition had no bounds. Alexander and his Greeks
conquered Persia in three short years. Wisely, he left the
efficient, well-run state to continue to operate as it had.
Persia was, in effect, assimilated, but under the rule of
Alexanders lieutenants. Alexander then moved on to conquer
Lower Egypt, Bactria, and various states and tribes in and
around modern-day Iran and Afghanistan. His efforts finally
came to a halt in India, at which point the Greeks were just
about ready to hang it up and go home. Also at that point,
Alexander was suffering from delusions of grandeur, and his
rule had become cruel and arbitrary. Be that as it may, at
age 32 Alexander was dead of unknown causes. Thereafter,
Alexanders lieutenants would rule a loosely joined Hellenic
Empire of mixed Asian and Greek influences. In Egypt, Hellenic
and Egyptian influences fused under the long rule of Ptolemy
(one of Alexanders lieutenants) and his successors, the last
of which was Cleopatra.
The Rise of Rome
The history of the Roman Empire is, in effect, the history
of the foundations of Western civilization. However, because
the Romans annexed Greece and were heavily influenced by
Greek thought, arts, and architecture, scholars often refer
to Greco-Roman civilization. In any case, because Western
civilization would come to dominate the globe, a student who
wants to understand our world will make an effort to explore
all the things that made up the glory that was Rome.
Lesson 2
51
Rome started out as a provincial region of dirt farmers and
goat herders on the banks of the Tiber River, scattered over
the seven hills that would one day be covered by the monu-
mental architecture that characterizes Rome to this day.
Early Roman society no doubt borrowed some cultural
elements from the Etruscans, but Roman culture had a
distinct character based on martial virtues. The required
length of military commitment to Rome ranged from 16 to
25 years. In fact, as the Empire expanded to make the
Mediterranean a Roman pond, the retirement cities of
Roman legionaries (soldiers), scattered from Britain to
Anatolia to Egypt to Palestine, became the templates for
architecture and Roman (or Greek) ideas over the lands
ruled directly or indirectly from the eternal city (Rome).
Across the empire, the expression all roads lead to Rome
held more than a grain of truth.
A major tool of Roman expansion was civil engineering. The
Romans invented cement. Cement permitted the construction
of such wonders as aqueducts, the ruins of which are yet seen
all around the Mediterranean and as far west and north as the
lands Romans called Gaul (France) and Britannia. The Roman
legionaries spent as much time building roads and bridges as
they did striking down foes on battlefields. Another major tool
of Roman expansion was managed assimilation. Provinces were
allowed to follow their own customs and practice their local
religions, under the sway of local Roman governors. Also,
encouraging loyalty to Rome, people in conquered areas were
granted the rights and privileges of Roman citizenship.
Roman history can be roughly divided between republican
and imperial eras. The Republic featured elected consuls,
an aristocratic assembly called the senate, and tribunes who
represented ordinary citizens. However, Rome was an empire
based on conquest. Thus, central government by temporary
wartime leaders called dictators resulted from the need for
central command of the Roman legions. The dividing line
between the Republic and the Empire was the election
Augustus in 27 B.C.E. He was to rule as princeps (chief) of
Rome for life. In effect, Augustus was Romes first emperor.
World History
52
The chronic insecurity of Rome was generated by its long and
barely manageable northern and eastern borders. Under
these conditions, the Romanizing of barbarian peoples was
a chancy game. After fierce resistance, the Celtic peoples of
Gaul became enthusiastic participants in the expansion of
the Roman Empire. By contrast, the Germanic peoples living
beyond the Rhine River were left to fume, propagate, and
eventually pose a lethal threat to the empire.
Study Map 7.3 on page 163 of your textbook in conjunction
with the Roman Expansion chronology sidebar on page 165
to get an overall sense of Roman wars and conquests over the
period of imperial expansion.
The Beginning of Imperialism in India
As a response to the shock of Alexanders incursions into
the Indus valleys, the states of the Ganges valley began to
organize into a defensive confederation. Available sources
name a leader called Candragupta in regard to this political
development. However, little is known about what was going
on in India up to the time that Asoka founded the Mauryan
Empire in the 260s B.C.E.
Asokas roughly 45-year reign was remarkable by its results
and by its unique character. Like other imperial monarchs,
he expanded his empire over nearly most of the Indian sub-
continent by way of conquest. Imperial governance included
efficiently organized trade, an extensive road network, and
effective irrigation and land management. His decrees were
inscribed on stone throughout the Ganges valley and on
stone pillars in the Deccan. (The Deccan refers to the south-
ern part of India.)
Empire management requires literate communication, typically
organized within state bureaucracies. In India, literacy was
more or less the sole province of Buddhists priests and
monks. Therefore, Asoka had to rely on Buddhist scribes
and clerks to get things done. But Asoka did more than
exploit Buddhist literacy. He also, and increasingly, relied
on Buddhist teachings to impose moral and civil order over
his domain. This tendency led Asoka to the remarkable
conclusion that further expansion of the empire by conquest
Lesson 2
53
was immoral. Instead, conquest of hearts and minds should
proceed by spreading Buddhist teachings across the subcon-
tinent and beyondeven unto foreign lands.
Paradoxically, Asokas moral sentiments would weaken the
empire. Empires have to expand or die. Thats the general
rule. With the political boundaries of the Mauryan Empire
closed, political and military factions turned on each other,
and about 25 years after Asoka passed from the scene in
232 B.C.E. India broke up into rival states. Be that as it may,
Mauryan habits of environmental adaptation and much of
Mauryan infrastructure remained to be revitalized here and
there over time. At the same time, evidence suggests that
Indias oppressive caste system was being etched into the
fabric of Indian society during this period.
Chinese Unity and Imperialism
After 500 years of division and strife among warring states,
there was still a Chinese ideal of imperial unity. With that
ideal in mind, a marginal society called the Qin, a bit like the
provincial herders and farmers of early Rome, began an
agenda aimed at actually doing away with any notion of
empire. Qin rulers discontinued all the old imperial rites,
thus seeming to dissolve the imperial ideal. However, a bit
later, a new king of Qin, Shi Huangdi, declared that the
dismantled empire should be replaced. To that end, he led a
25-year campaign of isolating and conquering all the rival
states. Shi Huangdi then declared himself first emperor and
sole ruler of a new, unified, China. Shi Huangdi intended to
get rid of the Chinese aristocracy, abolish slavery, and outlaw
inheritance practices that concentrated wealth in the hands
of the few at the expense of the many. Now China was to be
ruled through a uniform system of civil and military districts
under his control. Uniformity was the key theme. Laws,
coinage, and standards of measurement were to be the same
all over China.
World History
54
Shi Huangdi was responsible for piecing together the Great
Wall. His funerary tomb included a buried army of terracotta
warriors, which today is a major Chinese tourist attraction.
At the same time, the first emperors reign was oppressive.
The Chinese social motto might have been conform or burn.
By the time Shi Huangdi died in 210 B.C.E., his son and heir was
deposed by rebels. The power vacuum was all but immediately
filled with what amounted to a revival of the Period of Warring
States. However, one of the rebel princes, Liu Bang, emerged
from this winner-take-all political storm as the first ruler of the
Han dynasty. Chinese expansion under the Han (206 B.C.E. to
220 C.E.) would unify the Chinese people. Indeed, they began to
call themselves Han. During this roughly 400-year period, the
Chinese population trebled and the central axis of China
shifted from the Yellow River, south to the valley of the
Yangtze.
A major threat to the relative peace and prosperity of Han
China was the Xiongnu people who lived in the steppes north
of the Great Wall and just beyond where the Silk Roads
gathered (in the throat of China). The Xiongnu were all
but born on horseback and were renowned for their skills
as mounted archers. The Han policy toward these steppe
nomads, as advised by Confucian sages, was mainly one
of appeasement. That worked to some extent. But at last,
around 120 B.C.E., the Han mounted operations to secure
the western flank of the Great Wall and to fortify the Silk
Roads. After that, due in part to internal factions and civil
war among these steppe nomads, the threat posed by the
Xiongnu faded away.
Beyond the Empires
Significantly advanced societies hovered around the border
of the Great Empires. Your textbook discusses developments
in Japan and Korea, the western Eurasian steppe, and
Mesoamerica on pages 172174.
As you conclude this chapter, note that the axial age left three
enduring legacies: a durable heritage of ideas; lengthening,
although insecure, trade routes; and a group of empires.
Please complete Self-Check 3 now.
Lesson 2
55
Self-Check 3
Indicate whether each of the following statements is True or False.
______ 1. Jews were expected to convert non-Jews to believe in and worship their god, Yahweh.
______ 2. Stoicism was a religion that appealed to Roman elites.
______ 3. Historians strongly agree that Alexander set out to conquer the world to avenge his
fathers assassination.
______ 4. The Romans recognized the Celtic people they encountered in Gaul as being civilized.
______ 5. Mozi, an opponent of Confucius, preached a secular doctrine of universal love
400 years before the time of Christ.
______ 6. The Romans undertook the Punic Wars against Carthage because they were
enthusiastic sailors and navigators.
______ 7. In India, Asoka rejected the teachings of Buddhism in the latter part of his reign.
______ 8. According to Hippocrates, the balance of four vital fluidsblood, phlegm, black bile,
and yellow biledetermined health or illness.
Fill in the blank with the correct term.
9. Among axial-age sages, those who embraced the idea that truth could be found by unaided
reason are called _______.
10. A major factor in early Greek history was their wars with the ________ Empire.
11. In India, the spiritual leader popularly called Mahavira founded a religion that called for
detachment from the world, chastity, truth, and charity called _______.
12. The Academy of Athens, founded in 380 B.C.E., was established by _______.
(Continued)
World History
56
Self-Check 3
13. Posing a threat to imperial China in the north, the Xiongnu were natives of the Asian
_______.
14. In China, the _______ opposed the teaching of ethics and insisted that society should be
based only on obedience.
15. With their invention of _______, the Romans were able to excel as builders and engineers.
Answer each question in not more than four complete sentences.
16. Using an example, briefly explain the concept of a syllogism.
__________________________________________________________
__________________________________________________________
__________________________________________________________
__________________________________________________________
17. Why was the Indian Ocean such an important highway of trade?
__________________________________________________________
__________________________________________________________
__________________________________________________________
__________________________________________________________
Check your answers with those on page 216.
Lesson 2
57
FITFUL TRANSITIONS
The second part of Lesson 2 covers Chapter 8, Post-
Imperial Worlds: Problems of Empires in Eurasia and
Africa, ca. 200 to ca. 700 C.E.; Chapter 9, The Rise of World
Religions: Christianity, Islam, and Buddhism; and Chapter
10, Remaking the World: Innovation and Renewal on
Environmental Frontiers in the Late First Millennium.
ASSIGNMENT 8: POST-
IMPERIAL WORLDS: PROBLEMS
OF EMPIRE IN EURASIA
AND AFRICA, CA. 200 TO
CA. 700 C.E.
Read this assignment. Then read the Part 4 opening spread and
Chapter 8, pages 178209, in your textbook.
During the axial age, the Maya civilization occupied Guatemala
and the Yucatan Peninsula. In the same period of the axial
age, in the highlands around present-day Mexico City,
Teotihuacan developed an imperial state. It lasted into about
the mid-800s C.E., at which time fire marked a catastrophe
that shattered the empire. In this section, think about
the relationships between the Maya and the peoples of
Teotihuacan. In particular, consider how the rise and fall
of Teotihuacan echoed similar patterns of rise and fall in
the Old World.
The Western Roman Empire
and Its Invaders
We often speak of the fall of the Roman Empire as though
it were a singular event. In fact, the gradual decline of the
empire was a complicated process that extended over several
hundred years.
World History
58
The reign of the Emperor Marcus Aurelius (161 to 180 C.E.)
marked the high tide of imperial expansion. But before and
after Marcus Aurelius, the Roman Empire faced many chal-
lenges. Relations with the powerful Persian Empire were more
often hostile than not. Not infrequently, the Roman legions
were inclined to fight each other, backing successive imperial
candidates they supported. The long boundaries of the
empire were porous, and problems with foreign incursions or
migrations were a constant imperial headache. Indeed, dur-
ing the third century C.E. Germanic invasions threatened the
survival of the Roman Empire.
Another Roman headache was the spread of Christianity.
Conservative Romans viewed the Christians with distrust at best
and lethal hostility at worst. Most Romans still worshiped the
old ways and the old gods. The Christian problem would be
resolved in the fourth century under the Emperor Constantine.
At that point, the Empire established a pattern of co-emperors:
one in the West at Rome and one in the East with a capital
founded by Constantine and named Constantinople (modern-
day Istanbul). Aiming to unite the empire around a common
core of values and beliefs, Constantine and his co-ruler at
Rome drafted a letter of toleration called the Edict of Milan in
328 C.E. Christianity was to be tolerated. However by 380,
Christianity was declared the official imperial religion, and,
thereafter, for centuries all of Western Europe was known as
Christendom.
Map 8.2 on pages 186187 of your textbook gives you an
overview of the Western Roman Empire and its invaders.
Study it long enough to sort out the paths of incursions by
various barbarian migrants, including Huns from the steppes
and Germanic peoples that included the Ostrogoths, Visigoths,
Angles, Saxons, and Jutes. The Franks were Celts from Gaul.
The Visigoths would sack Rome in 410.
Steppelanders and Their Victims
Environmental crises, including climate change, drought, and
plague, coupled with migrations related to these problems,
attracted displaced peoples of Eurasia to the borders of the
great empires.
Lesson 2
59
In China, by the end of the Han dynasty in 220 C.E., steppe
peoples were extending their influence south of the Yellow
River valley. In that context, plagued, as always, by inade-
quate procedures for royal succession, China reverted to the
old pattern of warring states. Particularly in the north, the
land was troubled by strife and warfare as a large number of
petty states and empires rose and fell. However, near the end
of the 400s C.E. an ascendant ruler of the barbarians intro-
duced Chinese rites and adopted the state cult of Confucius.
In short, just as Germanic invaders become more or less
Romanized, the invaders of China adopted and were
gradually absorbed into Chinese culture. That was Chinas
strength. Subject peoples tended to embrace Chinese iden-
tity. At the same time, Chinas enormous internal economy
made the empire self-sufficient. As new people became part of
China, they simply tended to increase internal trade and cre-
ate wealth.
In India, Hun invasions and incursions into northwest India
were resisted by the efforts of a ruler who called himself
Chandra Gupta. Gupta founded a dynasty aimed at unifying
the Indian subcontinent. The Gupta Empire covered much of
India, but governance was not well organized, and emperors
of the dynasty favored brutality and a rigid class system that
didnt do much to win the hearts and minds of the people. In
fact, public sentiment often favored the invaders.
Map 8.3 on page 190 of your textbook deserves your attention.
So does the Making Connections feature on page 191. It will
help you compare and contrast China and Rome with respect
to their geography, cultures, and economies.
New Frontiers in Asia
While barbarian incursions brought tumult and trouble to
the major empires of India and China, the times also pre-
sented opportunities for developing states both within and
along the margins of imperial boundaries. India became a
hodge-podge of diverse states, including one that was home
to Huns. North of China the state of Koguryo was emerging
in eastern Siberia and in the north of the Korean peninsula.
Koguryo became a refuge for Chinese people fleeing the
World History
60
barbarian invasions. In areas around the Gulf of Thailand,
a state called Funan prospered. Their culture seems to have
been borrowed from India, and Chinese visitors found it to be
a place of learning, refined tastes, and considerable potential
as a trading partner.
The Rise of Ethiopia
Ethiopia was an African state that prospered at the far west-
ern end of the monsoon-driven trade routes of the Indian
Ocean. The magnificent capital of the Ethiopian Kingdom,
Axum, was some 7,000 feet above sea level, making it easy
to defend. Also, the local soil was rich and easy to farm, and
the temperate climate at that altitude gave the kingdom
self-sufficiency in food production. The primary interest of
Ethiopians was farming. However, trade was a profitable
sideline that made Axum a wealthy and cosmopolitan city. In
time, the attractions of trade impelled imperial ambitions.
Early in the sixth century, King Caleb launched an expedi-
tion to conquer southern Arabia, the location of major Indian
Ocean trading ports. The invasion succeeded and Ethiopia
would occupy that region for the balance of the century.
Interestingly, Ethiopia adopted Christianity during the 340s.
So, in fact, Ethiopia was a Christian state before Europe
declared itself Christendom.
The Crises of the Sixth and
Seventh Centuries
Assorted crises swept Eurasia in the sixth and seventh cen-
turies. In 535 C.E., a monster volcanic eruption separated the
islands of Sumatra and Borneo. Untold tons of volcanic ash
swept around the globe, lowering temperatures and dimming
sunlight. All over Eurasia, growing seasons were disrupted.
At about the same time, plague swept across various regions.
Crop failures, plague, and resulting emigrations were primary
factors that ended Ethiopian domination of southern Arabia.
Elsewhere, plagues ravaged Constantinople and Japan.
Theres also evidence of a decline in health in Mesoamerica
around this time.
Lesson 2
61
However, these crises offered opportunities for rallying and
reforming weakened states. In the 600s, that was the agenda
of Indias King Harsha. Over a 41-year reign, he managed to
unify much of the Ganges Basin while garnering tribute from
neighboring states like Punjab and Nepal. However, Harshas
reforms didnt survive his passing.
Justinian and the Eastern
Roman Empire
The Western Roman Empire was officially dead after 476.
Germanic and other barbarian peoples ruled the fragmented
empire. Meanwhile, the Eastern Empire (renamed the
Byzantine Empire by historians) would last into the eleventh
century. At the outset, the inhabitants of the Eastern Empire
considered themselves Romans even though the primary
language of Byzantium was Greek and eastern culture was
distinctly eastern.
The Byzantine Emperor Justinian had some success in
restoring territories that had been part of the Roman Empire.
Over his reign, from 527 to 565, Justinian also managed to
fend off most barbarian marauders, mediate disputes among
Christian Church fathers, build the magnificent Hagia Sophia
Cathedral (twice), and institute a legal code which would set
a template for European law.
Justinian was furiously energetic, impulsive, and overly
ambitious. Justinians rule was heavily influenced by his
controversial co-ruler, Theodora. She had once been an
actress and a performer in a live bear act (where she may
have grasped the nature of politics). Regardless, the jaded
and worldly Theodora had a level head, a sharp wit, and an
unbridled capacity to influence and guide imperial policies
through her spouse or on her own.
The New Barbarians
Barbarian invasions continued to plague Europe. The
Germanic Lombards invaded northern Italy in 568. To the
east, the peoples of the steppes called Bulgars crossed the
World History
62
Danube and set up a state that extended from the northern
Balkans almost all the way to Constantinople. During the
seventh and eighth centuries, Slavic peoples occupied much
of Eastern Europe, from the Baltic to southern Greece.
The Arabs
Before the seventh century, the nomadic Arabian peoples
were mainly known as nuisance marauders who troubled
trade routes, because Arabia sat astride trade routes between
the east and the Romanized west. During the 620s and 630s,
Arabia was transformed by the appearance and compelling
teachings of the Prophet Muhammad. He was a charismatic
leader whose influence touched every aspect of Arab life and
thought.
Muhammad was the founder of Islam, a word that means
submission (to the will of Allah). Claiming to have received
his teachings from the mouth of the Angel Gabriel, Muhammad
first spoke then scribed the holy book of Islam, the Quran.
Islam is a unique blend of Judaic and Christian ideas seasoned
with a touch of traditional Arabic paganism. The religion
recognizes Jesus as a prophet. The religion also sanctified
Arabian militarism, thus justifying the conquest of infidels
(nonbelievers).
The time of Muhammad was ripe for conquest. The Persian
Sasanian Empire and the Romans had exhausted each other
in warfare, leaving the Persian Empire open for Arabian con-
quest during the seventh century. By the eighth century, the
populous and most prosperous of erstwhile Roman colonies
in Syria, Palestine, Egypt, and North Africahad also fallen
to Arabian or Arabian-led conquest. By the mid-eighth cen-
tury, the Muslim world extended across North Africa all the
way to northern Spain
Map 8.4 on page 198 of your textbook shows the extent of the
Muslim Empire.
Lesson 2
63
The Muslim World
The Muslim Empire was too big for central unified rule.
After Muhammads passing, different regions were ruled by
caliphs. A caliph was considered a successor of Muhammad
who served both as a secular and spiritual leader. (As a
Westerner quipped, a caliph was both pope and king.)
However, there was a cleavage of attitudes about who could
be considered a proper successor of the Prophet. The Shia
maintained that the caliphate must reside in the hands of
the bloodline of Muhammad, through his son Ali and Alis
mother, Fatima. The Sunni held that any member of
Muhammads tribe could be made a caliph. The Shia-Sunni
divide would lead to rival caliphates and secessionist states.
This basic split remains. (Iran and southern Iraq are predom-
inantly Shia. Saudi Arabia, where the sacred cities of Mecca
and Medina are located, is Sunni.)
Muhammad aimed at establishing a religion that shaped every
aspect of daily life. The Muslim world was to be a theocracy.
Civil and religious life was to be woven into a single tapestry
reflecting the will of Allah. However, Muhammads efforts in
fulfilling that agenda were incomplete. As a result, Islamic
sages forged a body of sacred-secular law called Sharia,
which literally means the desert-dwellers way to water
(see page 198 in your textbook). The diversity of the Muslim
world today is largely a reflection of the extent to which
Sharia law is endorsed and applied.
Study the Making Connections feature on page 199 of your
textbook to compare the strengths and weaknesses of empires
in Mesoamerica, the Roman world, China, India, and Persia.
Recovery and Its Limits in China
With the barbarians held at bay and internal conflicts the main
problem, China recovered more easily from sixth-century
troubles than some of its neighbors. Seasoned by war against
invaders from the steppes, the military leader Yang Jian
(r. 581605) purged assorted princes from the empire hed
once served and set about re-creating Chinas empire. Jian
embraced a law-and-order agenda through brutal force,
hands-on administration, fits of temper, and frugality.
World History
64
That approach was probably best for restoring the empire.
However Jians successor, Yangdi, felt it best to restore
clemency over sharp-edge justice, restore Confucian learning,
and return China to revered ancient standards of civil order.
His great achievement was reintegrating the Yellow and
Yangtze River systems by constructing a Grand Canal, which
was, in fact, a network of canals. The potential glory of
Yangdis civil engineering legacy was shadowed by the high
taxes and forced labor needed to build the canals, coupled
with a bad case of overextension marked by a ruinous cam-
paign to conquer Korea.
In 617, as Yangdis reign ended, the most respected lineage
of China, the Li, mounted a rebellion, one of several that
cropped up at this time. By 624, under the rule of the Li
family patriarch, Li Yuan, the Yang dynasty was born at a
time when the exhausted Chinese longed for a period of
peace. The second emperor of the Yang dynasty, Taizong, was
a reformer. He opted for a scientific-skeptical worldview and a
measure of contempt for traditional ways. As a practical
ruler, he established a professional civil bureaucracy based
on a civil service exam, as opposed to political connections or
nepotism.
Taizongs reforms were successful in stabilizing the empire, in
part by working out a relatively painless formula for dynastic
succession. However, in 690 a remarkably cold-blooded and ego-
maniacal woman named Wu Zhao successfully employed
murderous wiles to become emperor. Empress Wu was what
modern psychologists would call a psychopath. (Psychopaths
are often highly intelligent. Wu was both smart and beauti-
ful.) She viewed torture and brutality as basic to effective
governance. The gradual result, even under Wus less brutal
successors, was a subtle weakening of the empire.
By the mid-eighth century, China was suffering significant
defeats by steppe nomads. The imperial court, seeking a
scapegoat, targeted a general named An Lushan. Lushan,
in turn, sired a rebellion. As a result, civil war broke out
that pushed China toward militarism and fragmentation.
Provinces attained relative independence, and the empire
lost effective control of all but the Yangtze Valley.
Lesson 2
65
In the Shadow of Tang:
Tibet and Japan
The mountainous highlands of Tibet have had a long
antagonistic relationship with China. In that context, Tibet
was a militaristic state from about the time in the fifth century,
when barley production began to provide food surpluses suffi-
cient to mount armies. By the seventh century, under the
king Songsten Gampo (r. 627650), Tibet was a power to be
reckoned with. In 640, the Tang were forced to pay tribute to
Tibet in exchange for honoring Chinas territorial boundaries.
For a period of 250 years, Tibetan aggression subjugated Nepal
and parts of central Asia. Tibetan cities grew rich and cosmopol-
itan through trade and cultural exchange. However, by the ninth
century, surrounded by enemies, Tibet was losing ground. The
last king of the Tibetan state was assassinated in 842.
Japan began falling under the influence of Korean and
Chinese culture around the year 400. Much of that influence
had to do with the arrival of Buddhist monks and scholars.
Around the year 475, the most prominent Japanese state was
Yamato. As a maritime state, its rulers were interested in
courting China as well as expanding their influence into Asia
and other parts of Japan. Direct Japanese contact with
China would begin in the 600s.
An interesting feature of Japanese governance was the influ-
ence of women. To ease the threat to stability posed by the
transfer of power between rulers, women were acknowledged
as fit to rule. Thus, the pool of eligible imperial candidates was
expanded, and many of Japans early emperors were women.
See Map 8.5 on page 201 of your textbook to consider the
extent of Tang China, Tibet, and Japan ca. 750 C.E. Check
out the Making Connections feature on page 205 to compare
developing frontier states. Finally, be sure to study the
Chronology sidebar on page 206.
World History
66
ASSIGNMENT 9: THE RISE OF
THE WORLD RELIGIONS:
CHRISTIANITY, ISLAM, AND
BUDDHISM
Read this assignment. Then read Chapter 9, pages 210235, in
your textbook.
Commerce and Conflict:
Carriers of Creeds
The word jihad can be translated as holy war, even though
Muhammad used the word in different contexts. In practice,
as the Islamic Empire grew, it tended to mean that Christians
and Jews could either convert or a pay a poll tax for continuing
their religious practices. In any case, given the social atmos-
phere of our time, it should be recalled that religious faiths
have long been bent and twisted to justify armed aggression
and conquest. That was the case as Charlemagne expanded
the Frankish Empire in the eighth and ninth centuries, and
in India, where Buddhist ideals were spun to justify armed
conquest by Asoka and his successors. For example, the
Buddhist and Hindu concept of dharma (duty to god) was
used in rather the same way as the Islamic concept of jihad.
In any case, as youll learn in your textbook, trade was at
least as important as war in spreading religious doctrines.
A doctrine called Manichaeism was adopted by a pastoral,
Turkic-speaking people of the steppes called the Uighurs.
It originated under a teacher named Mani in third-century
Persia. During the eighth century, it became a rival of Buddhism
in some places. Manichaeism resembles Zoroastrianism in
positing an eternal conflict between good and evil. It resem-
bled Buddhism in embracing the sanctity of life and the
merit of good deeds. The religion never caught on in a big
way, but its worth knowing about because some of its ideas
would influence early Christian church fathers and become
known as the Manichean heresy.
Lesson 2
67
Christians didnt have much of a presence along the Silk
Roads. However, followers of Nestorius, a fifth-century Bishop
of Constantinople, had some influence across the trading
routes of Eurasia. Nestorian Christians viewed Jesus as
being human, not divine. Thus, its but a ghostly footnote
in the history of Christianity.
While Buddhism dominated the land trade routes across the
steppes, Islam was spread at least as effectively by way of
Indian Ocean sea lanes and trade routes that crossed the
Sahara. Indeed, Islam would become a dominant religious
influence in West Africa.
Monarchs and Missionaries
Constantine
A pivotal figure in the rise of Western Christianity was the
Byzantine Emperor Constantine. In your textbook, youll
learn a bit about what seems likely to be myth. In Italy,
leading an army aimed at ousting a usurper to the Western
imperial throne, Constantine was supposed to have seen a
sign in the skya sunlit cross with the accompanying text,
By this sign conquer. In fact, Constantine was a Romanized
politician, not a religious ideologue. (He was actually a sun
worshiper who would only formally adopt the Christian faith
on his death bed, and even that may have been a political
act.) Many historians argue that Constantines patronage of
the Christian Church was more about stabilizing the empire
than devotion to the Christian faith. Constantine did, in fact,
get together with his Roman co-ruler Licinius in 313 to issue
a letter of religious toleration which would be called the Edict
of Milan. And Constantine did, in fact, preside over the delib-
erations of Christian Church fathers at the Council of Nicaea
in 325, from which would emerge the Nicene Creed and the
official doctrine of the Trinity. However, Christianity wouldnt
become the official state religion of the Empire until 380
under the Emperor Theodosius I.
World History
68
Here are a few tips for negotiating this rest of this section.
Notice two main points: (1) Religious allegiances tend to shift
along with political agendas. (2) Attaining the patronage
of monarchs is a common strategy among religious
missionaries.
Ezana
King Ezana presided over the royal court at the Ethiopian
capital of Axum. His conversion to Christianity in the 340s
was marked by radical changes in his approach to gover-
nance. Where once he had identified with a war god, he now
justified his reign with reference to the Lord of heaven and
the Christian Trinity.
Trdat
Trdat was a king of Armenia. His story is reminiscent of the
change of heart recorded in the New Testament when Saul
of Tarsus became Paul the Apostle. Somewhere around the
years 301314 he switched from cruel persecutor of Christianity
to pious advocate. Around 314, Constantine was acting as an
advocate of Christianity, so the latter date is more likely.
Diplomatic Conversions
Diplomatic conversions occurred here and there around the
Caucusus. The Georgian kingdoms of Iberia and Lazica,
influenced by priests from Constantinople, found it helpful
to ally with Christianity, and thereby Rome, as opposed to
Persia. The rulers of the Khazars shifted religious allegiances
depending on the winds of power politics, alternately adopt-
ing Judaism, Islam, and Christianity in efforts to maintain
their political independence.
Buddhist Politics
Buddhist politics impacted Chinese society in large part
because some Chinese emperors looked to Buddhist ideals
to justify their reign. Thus, even as Chinese loathing of for-
eign religious ideas flared up from time to time, Buddhist
Lesson 2
69
monasteries popped up all over China. As a result, Buddhism
managed to imprint cultural influences on the face of China
that would coexist with the teachings of Confucius.
Korea
In Korea, the kingdom of Koguryo absorbed Buddhist teach-
ings as Chinese refugees fled barbarian invasions in northern
China. Initially, because state formation in Korea was based
on extending uniform religious rites from one community to
another, the rest of Korea resisted Buddhism. Eventually,
however, Buddhism was accepted as a royal religion in the
southern Korean kingdoms of Paekche and Silla.
Japan
Korean missionaries introduced Buddhism to Japan. After
the usual period of resistance, the Japanese gradually
worked out a fascinating blend of a universal religion,
Buddhism, with the indigenous religion, Shinto. Shinto pre-
served imperial prerogatives and traditional rites; Buddhism
informed ideals of moral behavior and spiritual discipline.
Tibet
In Tibet, Buddhism arrived along with migrating Buddhist
monks out of India. However, Tibet wasnt a Buddhist
country. For some time, Buddhism contended with a rival
Tibetan religion called Bon. Oddly, Bon was very similar to
Buddhism in its teachings, even though it recognized Shen-
rab as the original Buddha.
In 792, a Tibetan king, Trisong Detsen, presided over a great
debate between Indian and Chinese Buddhist sages. The
Indians endorsed Theravada Buddhism, which taught that
the progress of the soul to perfection could only progress in
baby steps over many lifetimes. The Chinese, by contrast,
endorsed Mahayana Buddhism, which maintained that a
soul could reach perfection in a single lifetime. Mahayana
Buddhism would become dominant in China and Japan.
World History
70
India
In India, where Buddhism was founded, it became a minority
religion in favor of Hinduism. Hindu rites, teachings, and
doctrines were derived from ancient writings, such as the
Upanishads. Hindu social traditions tended to endorse con-
servative traditions, including the oppressive caste system
and unpleasant rites that included blood sacrifice and sati.
Sati is the practice of requiring a wife to join her husband
on his funeral pyre.
The Margins of Christendom
Along the margins of Christendom, royal conversions to
Christianity seem to have been all the rage in the late tenth
and early eleventh centuries. Poland, Denmark, and Norway
adopted Christianity by way of royal conversions in the late
900s. The Slavic Magyars became Christians with the
ascendance of Stephen of Hungary to the throne in 1001.
(Hungarians call themselves Magyars.) An interesting excep-
tion occurred in the year 1000 when the Norse people of
Iceland held a vote and Christianity won a narrow victory
over Norse paganism.
Vladimir and the Rus
Vladimir was the ruler of Kiev (in present day Ukraine) in the
late 900s. He ruled a people called the Rus. In fact, Vladimir
counted a Norse (Rus) prince among his ancestors. In any
case, he was a brutal sovereign who would later be styled a
saint. After sampling Christian rites among the Germans
and the Slavs, his eyes were opened, so to speak, when he
encountered Greek Byzantine Christianity in the Hagia
Sophia of Constantinople. Thus spiritually enlightened,
he went forth to convert the Rus to Christianity by force.
However, he did sugar the enforced conversion by translating
Greek liturgy into the Slavonic language of the Rus, and
thus was born the Russian Orthodox Church.
Lesson 2
71
Islam and the Turks
Countering the magnetism of Christianity along the margins
of Christendom, the Turks adopted Islam. Given the fierce,
warlike character of these people, its hard to say just why
this happened. But it did. Thereafter, Turkic states in
Anatolia and Afghanistan would lay down foundations of
considerable importance to later historical developments.
Trickle Down: Christianization
and Islamization
The trickle-down idea refers to the fact that the spread of
the universal religionsIslam, Buddhism, and Christianity
has been related to advocates among rulers, forced
conversions, and penalizing of nonbelievers.
A key to this section is the Making Connections feature on
page 225 of your text book. Study it to see how war, trade,
missionaries, and the influences of elites were factors in the
spread of Buddhism, Christianity, and Islam.
Religious Lives: The World of
Monks and Nuns
The spread of Christianity and Buddhism was greatly encour-
aged by people who adopted a monastic life. Monks and nuns
adopted solitary, ascetic lives associated with meditation, con-
templation, and mysticism. Many Christian saints were monks
or nuns. At the same time, both Christian and Buddhist monas-
teries were often centers of learning and scholarship that also
gathered and enhanced secular learning.
Monasticism wasnt endorsed by Islam. Nevertheless,
Islamic ascetics and hermits, perhaps inspired by Christian
ascetics, gathered in monastic colonies devoted to scholar-
ship, meditation, and the quest for deep, direct spiritual
experience (mysticism). These Islamic ascetics and mystics
were called Sufis. As it turned out, Sufi thought and poetry
served to spread Islamic ideas.
World History
72
ASSIGNMENT 10: REMAKING
THE WORLD: INNOVATION AND
RENEWAL ON ENVIRONMENTAL
FRONTIERS IN THE LATE FIRST
MILLENNIUM
Read this assignment. Then read Chapter 10, pages 236259,
in your textbook.
Where human populations have been concentrated within
states and empires, resources for sustaining their societies
have tended to exhaustion. Intensive farming depletes soil
nutrients and amplifies water erosion. In times of draught,
wind erosion can turn a fertile prairie into a dust bowl. As
woodlands are stripped away for fuel or building materials,
erosion and soil loss become problematic.
During the last part of the first millennium C.E., between
about 700 and 1000, environmental problems of this sort
plagued much of the globe, especially across the dense popu-
lation belt of Eurasia. All of this was further aggravated as
nomadic peoples trickled or poured across imperial borders.
These environmental challenges could lead to developmental
decline, and in some places that was the case. Areas of
ancient habitation have, in many times and in various
places, been abandoned. However, change also inspires inno-
vation, and therein is a central theme of this chapter. Just as
plants and animals can adapt to new environments, so can
humans. People can envision goals, assess failures, and learn
from observation and experimentation. People can find better
ways to manage and benefit from their environment.
Isolation and Initiative: Sub-Saharan
Africa and the Americas
Africas geography presents formidable barriers to trade and
cultural exchange. The great expanse of the Sahara separates
sub-Saharan regions from North Africa and the Mediterranean.
Lesson 2
73
River access to interior regions of sub-Saharan Africa is lim-
ited by way of poor navigability on the one hand and malarial
tropical forests on the other.
The two culturally sophisticated areas of first-millennium
AfricaEthiopia and West Africaare at opposite ends of the
broad grasslands of the Sahel that extends east and west
across the continent south of the Sahara. However, avenues
of cultural exchange like those that crossed the Asian
steppes didnt develop between these two regions. Ethiopia,
like cities and states that arose in East African coastal
regions, was linked to Indian Ocean trade. The early states
that arose in West AfricaGao and Ghanadeveloped
independently in step with improvements in rice and millet
farming.
West African states were based on tribal and clan kinship
organized under the rule of divine kings. Trade routes linked
these resource-rich states to North Africa and other regions
dominated by Islamic influence. For this reason, West African
states would absorb Arab influences, with Islam becoming
the dominant religion of later West African states.
Study Map 10.1 on page 239 of your textbook. Notice how
villages and cities are joined by trade routes.
The geography of the Americas also restricted cultural exchange.
Nevertheless, fascinating and socially complex states arose
along the Andes ranges that define South Americas western
regions, as well as in Central American and Mexico. A desert
region between the Pacific and the Andean highlands in
northern Peru was the site of two civilizations that lasted
for several centuries, the Nazca and the Moche. The former
society is famous for its elaborate and fascinating works of
art. The so-called Nazca lines are spread across desert
plateaus and are recognizable only as spiders or humming-
birds, or what-have-you, from airplanes. The Nazca lines
remain one of historys mysteries.
Probably the most fascinating among Mesoamerican civilizations
were the ancient Maya, a people whose descendants are still
found in Guatemala and the Mexican Yucatan Peninsula. The
themes of all the cultures that either predated or postdated t
he Mayan are remarkably similar. We find sophisticated monu-
mental architecture, rituals of blood sacrifice associated with
World History
74
divine kings, and related forms of writing. Mayan writing and
mathematics served two purposes: tracking and predicting
astronomical cycles and recording the histories of royal
dynasties. Interestingly, the Mayan calendar was more
accurate than any known calendar up until modern times.
Map 10.2 on page 240 of your textbook offers information
about Mesoamerican and Andean societies between 300 C.E.
and 1000 C.E. Pay special attention to the inset focus maps to
note the significant extent of the Mayan cultural region as well
as regions of Huari, Nazca, and Tiahuanacan cultures in
regions of Peru and Boliviacultures that developed in the
high Andes.
As you study this assignment, recall that a basic theme of
this chapter is ecological innovation. The relatively fragile
Andean and Mesoamerican cultures coped through success-
ful experiments in farming. In the high Andes, for example,
hybrid varieties of potatoes were bred for microclimates at
different altitudes. In Mesoamerica, agricultural mounds
(called milpas), which were often linked to some kind of
irrigation system, produced rich harvests of beans, squashes,
maize, and peppers. In North America, new crops and tech-
nologies developed along the northwest coast, as well as in
areas of the Mississippi and Missouri valleys and along the
Ohio River valley. A vital frontier in these latter two cases
was maize development.
Study the Making Connections feature on page 246 of your
textbook to compare expanding states in the Americas from
200 to 800 C.E.
The Islamic World and the
Environment
According to a quip attributed to the Emperor Napoleon,
An army travels on its stomach, but Napoleons quip also
applies to states and civilizations. In that context, note that a
major aspect of the expanding Muslim world was the develop-
ment of new crops and the introduction of known crops into
new environments.
Lesson 2
75
Study Map 10.3 on page 247 of your textbook to understand
the enormous range of sources for new crops adopted and
adapted to different regions of the Muslim world. Study the
icons to consider the different kinds of crops. Notice how many
new crop plants came from India, Southeast Asia, and even
China.
Muslim rulers encouraged the search for new plants that
could add spice to the table or provide balm for different
kinds of ailments. Royal gardens employing professional
agronomists became favored projects.
Islamic law favored farmers. Landowners could use and
dispose their property in whatever way they preferred. As a
result, the most productive farmers tended to acquire more
land as food production increased.
Adapting new plants to new environments encouraged the
development of better ways of watering and cultivating crops,
including the use of fertilizers. As a result, farming began
to make use of marginal lands, and acreage devoted to food
or fiber crops expanded.
Frontier Growth in Japan
In Japan, various efforts were undertaken to expand food
production. In 711, an imperial decree permitted landed
aristocrats to apply to provincial governors for the right to
cultivate virgin lands. Later, farmers were granted rights of
ownership of their cultivated land for three generations if
their lands were irrigated by way of new ditches or ponds.
Then, by 1743, farmers were granted absolute ownership
of their land.
Significant efforts to pioneer new land were pursued by
freelance holy men, such as the Buddhist monk Gyoki. At
that point, the Japanese werent enthusiastic about Buddhist
practices, such as cremation of the dead, but they liked the
manner in which Gyoki organized work on digging ponds,
creating roads, and building bridges.
Colonization opened new lands for development on the major
Japanese island of Honshu. Of course, as seems always to
be the case, this was accomplished by displacing or otherwise
eliminating indigenous peoples.
World History
76
China and Southeast Asia
Even when imperial rule was shaky and uncertain, efforts
to make use of marginal lands to expand food production
took place in India and China. From 700 to 1000 C.E., states
developed all over mainland and island regions of Southeast
Asia.
In China, from the seventh century on, imperial policies,
such as the development of the Grand Canal system,
encouraged expanded food production. Population growth
stimulated new agricultural techniques, which, in turn,
stimulated population growth. A major aspect of Chinese
imperial policies was southward colonization into regions
that favored rice production. By the mideighth century,
one-third of Chinas burgeoning population inhabited the
Huai and Yangtze Valleys. By the eleventh century, half
of Chinas population would inhabit this same region.
A southward growth of population based on expanded
food production also gave rise to new states in mainland
Southeast Asia and beyond.
Study Map 10.4 on page 252 of your textbook. It will help
you to (1) understand the extent of the canal system in China,
(2) locate the densest population regions of China, (3) track
maritime trade routes, and (4) locate cultural regions identified
with Viet, Champa, Cambodian (Khmer), and Malayan-Sumatran
Srivijaya societies.
The Pacific
The most remarkable aspect of human colonization in the
Pacific is the Pacific Ocean itself. Archaeologists continue to
marvel at how humans reached the Solomon or the Caroline
islands over such ocean expanses. In that regard, the territo-
rial expansion of the Polynesian peoples from New Zealand
to Samoa to the Hawaiian Islands stands out. Polynesian
navigators were capable of managing thousand-mile voyages
by tracking the stars of the Southern hemisphere and liter-
ally feeling their way through changing ocean currents.
Lesson 2
77
The Expansion of Christendom
In Western Europe, the idea of the Roman Empire informed
all the former lands of the Western empire, now deemed
Christendom. However, by the eighth century European
boundaries were being expanded well beyond the boundaries
of the old Western Roman Empire.
In Western Europe, the key player was the Frankish king
Charlemagne. (The name Charlemagne was the medieval
French equivalent of Charles the Great.) Charlemagne did
what Julius Caesar dared not. In 1800, Pope Leo III crowned
Charlemagne Emperor and Augustus of the Germanic Holy
Roman Empire. By 1802, Charlemagne had completed his
conquest of the Germanic peoples of Saxony.
Meanwhile, as Christian monks found nests in the nether
realms of Scotland and Ireland, rival empires were emulating
Charlemagne in the east, contending for the title of Holy
Roman Emperor. In the early 800s, Mojmir I established a
Frankish-like Slavic state in Bohemia (a region of the pres-
ent-day Czech Republic). In 864, creating a buffer between
Europe and the Byzantine Empire, the Bulgars converted to
Christianity and entered the list as a rival claimant for
Roman imperial rule.
To the North, the peoples of Scandinavia were building their
Viking long ships and setting out on trade and conquest
ventures that would rattle all of Europe well into the eleventh
century. To the east, the Scandinavian Rus help establish a
dynasty that would rule people who would call themselves
Russians. To the west, Norse explorers would colonize Iceland
and found settlements that would deposit Iron
Age artifacts from Greenland to Newfoundland, which they
called Vinland. Initially, the Norse people werent exporting
Christianity. However, by the tenth century, Scandinavian
states were adopting Christianity.
World History
78
Now, review the material youve learned in this study
guide as well as the assigned pages in your textbook for
Assignments 610. Once you feel you understand the mate-
rial, complete Self-Check 4. Then check your answers with
those provided at the end of this study guide. If youve
missed any answers, or you feel unsure of the material,
review the assigned pages in your textbook and this study
guide. When youre sure that you completely understand
the information presented in Assignments 610, complete
your examination for Lesson 2.
Self-Check 4
Indicate whether each of the following statements is True or False.
______ 1. The Korean state of Koguryo benefitted from the influx of refugees from China who
were fleeing nomadic invaders.
______ 2. The literal meaning of the Arabic word jihad is service.
______ 3. In Japan, the conquest of new environments was the province of freelance holy men
as well as an agenda of state.
______ 4. When Buddhist leaders from India and China met in Tibet to debate the doctrines of
Buddhism, that mountainous kingdom had long been a Buddhist state.
______ 5. The expansion of the Islamic domain was marked by the gathering, cultivation, and
transplanting of a variety of new foods.
______ 6. The Mayan civilization depended on the unified, peaceful cooperation of city states.
______ 7. The body of Islamic law called sharia literally means the way to water.
______ 8. In Tibet, divine monarchs could be sacrificed when they were no longer useful.
(Continued)
Lesson 2
79
Self-Check 4
Fill in the blank with the correct term.
9. Followers of the female mystic Rabia al-Adawiyya, who established a mystical body of thought
and practice within the religion of Islam, were called _______.
10. The sixth-century Byzantine Emperor _______ was the last emperor to undertake a partly
successful campaign to restore control over previously Roman territories.
11. As a mediator over disputes among early Christian church fathers, the emperor ________
was instrumental in establishing Christianity in the Roman Empire.
12. In the Pacific, _______ navigators were able to pursue long voyages by detecting changes in
ocean currents.
13. Although it was the birthplace of Buddhism, _________ became the dominant religion in
India.
14. Influenced by Jewish monotheism, Christianity, and traditional pagan themes, the _______
became the sacred scripture of Islam.
15. Vladimir, ruler of Ukraine, impacted the history of the Slavs when he adopted Christianity
after visits to the _______ Empire.
Answer each question in not more than four complete sentences.
16. What is the difference between Theravada and Mahayana Buddhism?
__________________________________________________________
__________________________________________________________
__________________________________________________________
__________________________________________________________
(Continued)
World History
80
Self-Check 4
17. Why were eunuchs favored to serve in imperial administrations in China and Rome? Was this
policy always a good idea?
__________________________________________________________
__________________________________________________________
__________________________________________________________
__________________________________________________________
18. In what ways does geography isolate sub-Saharan Africa?
__________________________________________________________
__________________________________________________________
__________________________________________________________
__________________________________________________________
Check your answers with those on page 217.
Contacts, Conflicts, and
the Crucible
CONTACTS, CONFLICTS
The first part of Lesson 3 covers Chapter 11, Contending
with Isolation: c.a. 10001200, and Chapter 12, The
Nomadic Frontiers: The Islamic World, Byzantium, and
China, c.a. 10001200.
ASSIGNMENT 11: CONTENDING
WITH ISOLATION:
CA. 10002000
Read this assignment. Then read the Part 5 opening spread and
Chapter 11, pages 260285, in your textbook.
American Developments: From
the Arctic to Mesoamerica
Two fascinating cases of exploration and migration were tak-
ing place around the year 1200. Beginning around 1000, the
Thule Inuit, ancestors of the Inuit peoples who inhabit Arctic
areas all the way from Alaska to Greenland, managed a
transcontinental migration from Alaska to the northeastern
coast of Greenland and northern regions of Labrador. Their
technologies and social organization were pretty much what
Europeans would encounter as they made contact with the
people they called Eskimos hundreds of years later. The
Thule navigated open waters in canoes of walrus hide,
thriving on a fat-rich diet of Arctic caribou, walrus, seals,
and whales.
At around the same time, Norse explorers from Scandinavia
were voyaging across the North Atlantic, hopping off from
Iceland to make their way to Greenland and beyond. Like the
canoes of the Inuit, the Norse long ships had a shallow draft
81
L
e
s
s
o
n

3
L
e
s
s
o
n

3
World History
82
that allowed them to navigate near ocean shores and easily
make their way into inlets and rivers. The farthest reach of
the Norsemen into North America remains archaeologically
uncertain. But evidence confirms that Norse settlements
were established in Labrador and Newfoundland.
Study Map 11.1 on page 265 of your textbook to consider the
Thule Inuit and Norse Migrations. Note the extent of the Inuit terri-
tories, recognizing that Norse explorations in North American may
have put them in touch with Native Americans, including the
Thule Inuit.
In more southerly parts of North America, archaeologists
have revealed two outstanding instances of societies that took
on characteristics of states.
In the American Southwest, in areas of Colorado, New
Mexico, and Arizona, we find remains of a canyon culture,
with an urbanized center in the Chaco Canyon region. It
lasted for several hundred years, extending at its height to
cover some 57,000 square miles. The Chaco Canyon peoples
were organized under central, imperial rule. Evidence of that
includes a network of roads, up to 12 yards wide, suggesting
that they were avenues for armies. The demise of the canyon
culture appears to have been related to drought conditions
that increased in severity during and after the 1100s.
The Chaco Canyon state seems to have arisen in relative
cultural isolation. That wasnt the case for the Mississippian
cultures that arose in the present-day southeast United
States. Many of the cultural themes of Mesoamerica were
present in Mississippian societies. Seaborne trade trans-
ported ritual practices, ball games, social class stratification,
and modes of urban construction reminiscent of Mesoamerican
cultures. The most impressive Mississippian urban site was
Cahokia, a city that could have had a population of 10,000
people. The site, featuring lavish tombs, is located east of
present-day St. Louis.
Map 11.2 on page 267 of your textbook will help you locate the
Canyon and Mississippi cultures. Consider the impact of trade
routes emanating from Mesoamerica, including a connecting
route to the Mississippian peoples. Notice the location of
Cahokia on the northern boundary of Mississippian influence,
along the Missouri.
Lesson 3
83
Around the Indian Ocean: Ethiopia,
the Khmer, and India
Ethiopia
As youve learned, the East African Ethiopian Empire was
primarily reliant on indigenous farming. The magnificence of
Axum was a sort of cultural anomaly resulting from outside
contacts and trade. Be that as it may, Ethiopia was cosmo-
politan in the context of widespread cultural exchange. At the
same time, Ethiopians were fiercely dedicated to remaining
politically independent.
Toward the latter half of the 1200s, imperial propaganda
tended to grandiosity. It became fashionable for Ethiopian
elites to associate Ethiopia with the Biblical Queen of Sheba,
Solomons concubine, or with Solomon himself. In 1270,
Solomid aristocrats seized control of the imperial throne.
Khmer
India stood in the middle of the broad region of Indian Ocean
trade. Until the eleventh century, Indian culture had a per-
sistent and widespread influence in the societies of Asia and
Southeast Asia. Thus, its not surprising that the Khmer
Kingdom, located in present-day Cambodia, reflected
Buddhist and Hindu cultural influences.
Khmer power and influence in the zone of Indian Ocean trade
was directly related to fertile soils that produced rich rice
harvests. The magnificent city of Angkor was expanded and
embellished in step with increased wealth based on rice
exports. In that context, the rise to power of the Khmer King
Suryavarman II (r. 11131150) marked a new era of imperial
expansion and architectural achievement. The largest temple in
the world, Angkor Wat, was a product of Suryavarmans reign.
It was meant to celebrate the onset of a long era of prosperity
predicted in Hindu mythology. However, Suryavarmans
visionary exultation ended along with his reign in 1150.
Ambitious building programs and a remarkable public
World History
84
health system continued under subsequent kings. A signifi-
cant factor in Khmer history during the late twelfth century
was the adoption of Buddhism as the state religion.
India
After the eleventh century, Indias cultural impact beyond its
borders began to fade as political dissolution reduced the quality
of arts and learning. In the north, Hindu temples were attacked
by Muslim raiders from Afghanistan. Meanwhile, the subconti-
nent became a patchwork of squabbling rival states. In spite
of these troubles, the economy was booming in some parts
of India. Rajasthan became a bustling center of international
trade. Interestingly, merchant family dynasties enlisted
armies to aid their competitive struggles with Muslim rivals.
Although political chaos was the texture of much of central
and northern India, the situation in the south was another
matter. The southern Chola Kingdom (10701122) was
centered in the rice-producing heartland of the interior.
However, Chola power was based on a fusing of royal impe-
rial designs with the interests of merchant-warrior
dynasties centered in coastal ports. For the most part, Chola
imperialism amounted to raiding. Even so, Chola garrisons
had footholds in Sri Lanka, the Maldive Islands, and a pres-
ence in Malaysia significant enough to weaken Khmer control
of its tribute domains.
Map 11.3 on page 274 of your textbook will give you some
insights into the trade and cultural connections spread around
the boundaries of Indian Ocean navigation.
Eurasias Extremities: Japan
and Western Europe
During the eleventh and twelfth centuries, Western Europe
and Japan had benefitted sufficiently from cultural exchange
across Eurasia to begin to emerge from isolation.
Lesson 3
85
Japan
Between 838 and 1070, the doors of Japan slammed shut. All
trade with the outside world was forbidden. Buddhist monks
had to get imperial permission go on pilgrimages abroad.
Japanese uniqueness and isolation were celebrated as virtues.
In this social atmosphere, the center of political and literary
interest was the imperial court. Peasants oppressed by disease
and hunger were of no interest. Imperial courtiers appointed to
provincial governorships were viewed as having been dis-
graced. As a result, provincial governors were left to their
own devices. Among those devices was empowering thugs as
bodyguards and enforcers.
The relegation of the provinces to uncertain fate turned out
to be an imperial error with major consequences. By the
1070s, provincial governors began limited trade with China
and Korea, pursuing their self-interest. Out of this context,
direct relations with China were restored. As trade continued,
some provincial rulers began to acquire wealth and influence
that cast a deep shadow over the authority of the imperial
court. In 1160, the provincial Taira clan ascended to power
over rival provinces. In that guise, the ruler of the Taira clan
became the protector of the emperorthe shogun. The
Minamoto clan replaced the Taira in 1185, installing a new
shogun. Thereafter, the effective ruler of the empire was the
shogun and the power of the imperial court faded to near
irrelevance.
As you study this section, consider a paradox: During this
period, as Japan claimed to be a unique power unto itself,
the influence of Chinese culture and language was pervasive
among intellectuals and aristocrats.
Western Europe: Economics and Politics
After the fall of the Western Roman Empire in 476, Western
Europe entered a twilight of decline traditionally referred to
as the Dark Ages. Charlemagnes imperial achievements fell
apart as Europe became a jumbled patchwork of minor rival
states that were often little more than landed estates. Viewed
from the perspective of the Muslim world, Western Europe
was a backwater. However, as it turns out, the medieval
World History
86
period, in particular the period from 1000 to 1200, was
a period of adaptation and innovation. Consider a few
highlights.
By fits and starts, advances in agriculture initiated in the
Muslim world trickled into Europe. Farming became more
productive through the adoption of heavy iron plows, wind-
mills, waterwheels, and improved irrigation techniques. As
a result, the population of Europe doubled between 1000
and 1200.
Marginal lands were explored and developed both along the
margins of Europe as well as within Europe. Forests were
cleared, bogs were drained, and marginal lands were devel-
oped. Motivation for much of this activity was inspired by
efforts to spread Christianity to pagan lands within Europe
and the British Isles.
As populations increased and new cities and towns began
to dot the landscape, opportunities for trade were also
increased. The city of Lbeck, founded in 1143, became the
nexus of a federation of trading states and cities called the
Hanseatic League. A bit later, Mediterranean states and cities
sent their ships through the Straits of Gibraltar to expand
trade across the whole of Europe.
In Italian city-states, experiments in governance included the
concept of the commune. As a body representing the body of
ordinary citizens, the commune idea harkens back to the
Roman Senate. (Concepts from Roman law, including ideas
about citizenship and civic responsibility, would, in time,
become a major theme in Western political discourse.)
Western Europe: Religion and Culture
Efforts to purify the Roman Catholic Church were evoked by
the rise of dissenting heresies. As the burning of heretics
gained favor, the evangelical fervor of these efforts was amplified
during the reign of Pope Gregory VII (r. 10731085). Under the
Gregorian Reforms, more exacting standards were imposed on
clergy, even as clergy become more focused on converting the
common people.
Lesson 3
87
The teachings of medieval philosophers such as Abelard and
St. Anselm elevated the level of discourse among students
and scholars in favor of the power of reason over blind super-
stition. Later in this period, Europeans began to adopt the
learning and technologies of the Muslim world. Especially
from Moorish Spain, but also from the Near East, Europeans
gained access to classic texts of the ancient world, including
those by Aristotle and the Roman poets, which had been
translated into Arabic.
Other gifts to Europe, typically by way of the Muslim world,
included paper manufacture, improved techniques in metal-
lurgy, the compass, gunpowder, and algebra. Among
educated elites, attitudes toward the natural world were
shifting in favor of empirical observation and principles of
scientific analysis.
Study the Making Connections feature on page 280 of your
textbook to think about how different global regions coped with
isolation. Conclude your study of the chapter assignment with a
careful consideration of the Chronology sidebar on page 284.
ASSIGNMENT 12: THE NOMADIC
FRONTIERS: THE ISLAMIC
WORLD, BYZANTIUM, AND
CHINA, CA. 10001200
Read this assignment. Then read Chapter 12, pages 286311,
in your textbook.
The Islamic World and Its Neighbors
Threats to the Islamic world arrived from the Sahara and the
Asian steppes.
The Coming of the Steppelanders
In 1055, Seljuk Turks conquered Baghdad and turned the
reigning Caliph into a puppet. Mahmud of Ghazni, another
Turk, declared himself the self-appointed guardian of Islam
World History
88
in Afghanistan. Mahmuds minions raided northwest India,
gathering so many captives that slave market prices slumped.
This could have meant the end of Islamic reign in the Middle
East; but instead, beginning with the Seljuk Turks, the
invaders absorbed Islamic culture. The newly Islamic Turks
didnt abandon their warlike ways and were instrumental in
expanding the Islamic Empire through military conquests.
The Crusades
From the Western perspective, the Christian Crusades are
usually given a lot of attention. Europe was on the move,
making holy war beyond its borders to reclaim Christian
lands. From the Islamic perspective, however, the Crusades
were a minor nuisance. The Near Eastern Christian states
founded by the Crusaders were small, rather unstable, and
fairly easily contained by the fierce flickering of Turkic-
Saracen blades.
In the 1090s, Christian fanatics initiated Crusade fever
the Holy Lands should be returned to Christendom. However,
Pope Urban IIs primary political goal was to unite squabbling
medieval knights and petty lords under the leadership of the
Roman Catholic Church.
Culturally speaking, the Crusaders were babes in the woods.
They misunderstood both Islamic culture and the Greek-
Byzantine Christian cultures they encountered en route to
Syria and Palestine. In that context, they seriously upset
the relatively peaceful co-existence of Christians, Muslims,
and Jews already present in the Muslim world.
The Crusaders seized Jerusalem in 1099 and established
small Crusader-ruled states around ports along the eastern
Mediterranean, from Antioch to Tyre. Italian merchants
offered some support to the Crusaders, mainly in the interest
of benefitting from trade.
Around 1150, Zangi, a Turkish chief, declared jihad against
infidels and Shiites in the Near East. By 1170, Zangis Empire
was seized by Saladin, a Kurdish leader with formidable military
skills. Saladin would overthrow the Crusader Kingdom of
Jerusalem in 1187.
Lesson 3
89
To the Muslim world, the most significant conclusion of
twelfth-century conflict in the Near East wasnt ousting the
Crusaders. More important was the defeat of the Shiite
Fatimid Caliphate in favor of the Sunnis. The Crusaders
defeat in the Near East awakened a recognition that Western
Europe lagged behind the Muslim world in all kinds of ways.
The Making Connections feature on page 292 of your text-
book provides an overview of the Crusades in terms of their
historical background and their consequences in Europe and
the Eastern Mediterranean.
The Invaders from the Sahara
The Muslim situation in Spain in the eleventh country was
precarious. Muslim rulers had their hands full trying to con-
trol Christian populations. Warfare with the Christians was
chronic, and the outcomes were increasingly uncertain as the
so-called Spanish Caliphate dissolved into a patchwork of
rival kingdoms. Taking advantage of that situation, northern
Christian kingdoms began conquering Muslim territory. In
response, Muslim rulers called on the Almoravids of Africa for
help. Their request was honored.
The Muslim Almoravid peoples of North Africa were nomadic,
pastoral warrior-ascetics. Their domain was, basically, an
alliance of pastoral bands. By the 1100s, their domain extended
over the Sahara, extending from Morocco and parts of present-
day Algeria, deep into West Africa. Arriving in Spain, the
Almoravids succeeded in beating back the Christians. However,
in that process they also swept away and occupied the Spanish
Muslim kingdoms. In the 1140s, as the Almoravids were cor-
rupted by power, they would be displaced by another alliance
of warrior nomads from Africathe Almohads.
Examine Map 12.1 on pages 290291 of your textbook and
think about power politics around the Middle East and the
Mediterranean between 900 and 1100 C.E. Map 12.2 on
page 295 illustrates the extent of Almoravid power in West
Africa and southern Spain. It also shows the extent of
Almohad power in Spain. Note the location of the indigenous
African Kingdom of Ghana. Muslim influence in Ghana would
remain after that kingdom declined, and Islam remained the
dominant religion of West Africa throughout the Middle Ages.
World History
90
The Progress of Sufism
The essence of mysticism is that its a direct experience of
transcendent states of consciousness. Prior to the axial age,
shamanic mysticism employed trance techniques and drugs
to produce insight and visions. Within the universal religions,
selfless service to others, prayer, and meditation more often
characterized mystical spiritual ideals. One was to live in the
world without being of the world.
Sufism is, in effect, the mystical branch of Islam. Its rich
literature and poetry simply cant be summed up in an
overview course in global history. However, you can under-
stand why Sufism was popular with the common people even
as it was repudiated by power elites. Sufi doctrines extolled
compassion and mercy over legalism. Sufi teachings and
practices were indifferent to social station and tolerant of
different beliefs and cultures. Sufi ideas satisfied the common
peoples craving for saints. For reasons like these, Sufis
became Islams most effective missionaries.
The Byzantine Empire and
its Neighbors
Byzantium and the Barbarians
The Byzantine Empire was threatened by Arab expansion as
well as by Slavic expansion into the Balkans and the looming
presence of the Russians. Byzantium became increasingly
less Latin, more Greek, and a more distinct cultural region.
The main focus of the discussion here is on how the Byzantines
used diplomacy and intimidation mixed with a selective use
of military force to maintain its imperial identity. Note that
a major success of Byzantium in this era was effecting the
conversion of Russia to Christianity.
Basil II
Even as Byzantium maintained its influence over the Balkans,
Russia, and the Caucasus, there were clouds on the horizon.
The Muslim Arabs to the south and west were immune to
Lesson 3
91
Byzantine intimidation. Meanwhile, sandwiched as it was
between the Balkans and the Turks, Constantinople seemed
to be at the eye of a nomadic storm building up over the
steppes.
In 976, having arisen from less than advantaged circum-
stances, Basil II ascended to the imperial throne. Basils
idiosyncratic reign is hard to characterize. He made peace
with the Arabs, thus evading their power. He (its reported)
blinded 14,000 Bulgarian solders to send the Balkans a mes-
sage. Thereafter, adhering to no rules or conventions save his
own, he waged a campaign to endear himself to the domains
peasants while taxing the aristocratic landowners. He
attempted to weaken the aristocracy through oppressive taxa-
tion to finance his armies. While administering rough justice
wherever opposition reared its head, Basil also made nice to
subjugated Greek and Balkan states in order to retain their
allegiance.
The Era of Difficulties
Basils rough and impulsive style of rule centered on forcing
the aristocratic landowners to pay oppressive taxes in
exchange for providing military protection against Turks and
other invaders. However, after Basil died in 1025, things
began to fall apart. Byzantine decline was a gradual process
related to several factors.
Due to aristocratic squabbles, the imperial court suffered
from in-fighting that distracted them from affairs of state and
foreign policy. A prime example was the reign of Basils niece
Zoe and her jaded sister Theodora.
A schism (split) between church doctrines enlarged the rift
between the Latin-Roman Church and the Greek Byzantine
Church. Even as the Roman Pope attained to imperial
regency, the Greek Church refused to acknowledge the
ecclesiastical primacy of the Pope. Instead, the Patriarch
and bishops of the Byzantine Church were content to
preside under imperial power.
In the West, Turkish incursions into Armenia and Anatolia
cut into the Byzantine food supply. In the east, Normans
were menacing the Popes political independence and ousting
World History
92
the Byzantines from their last remaining territory in Italy.
(The Normans were descended from Norse people who settled
in the western France.) In 1071, the Byzantine Roman
emperor was defeated at the Battle of Manzikert.
To sustain its wealth and influence as borders shrank,
Byzantium turned increasingly to trade and commerce. In
that context, Constantinople became a rich and bustling
metropolis. And, as that happened, the Italian city state of
Venice became an overnight power as Venetians acquired
resources to build ships and enter into a trade relationship
with Constantinople.
Byzantium and the Crusaders
Toward the end of the 1190s, Europeans were clamoring for a
Fourth Crusade to recapture Jerusalem. The Venetians
offered to provide ships for transport, but the price was high.
The Byzantine prince Alexius IV, a pretender to the Byzantine
throne, sent an emissary to offer Crusader armies a land detour
through Constantinople in exchange for supporting Alexius bid
for the throne. Most of the Crusaders agreed to the deal. The
Fourth Crusade, launched in 1202, brought the Crusaders to
a Constantinople glittering with wealth and reeking of political
vulnerability. The Fourth Crusade to recapture Jerusalem ended
with the spilling of Christian blood at Christian hands as the
Crusaders sacked Constantinople in 1204. The remnants of
the Byzantine Empire in Europe were divided up among the
victors. In what was left of Byzantine Anatolia, rival dynasties
vied for the imperial throne. In this context, Venice acquired
a big chunk of Byzantine territory to acquire a virtual monopoly
of the Byzantine-European trade routes.
Map 12.3 on page 299 of your textbook shows the extent of the
Byzantine Empire and the maximum extent of Crusader
kingdoms.
Throughout most of this period, Byzantine art and learning
continued to flourish. Scholars and poets drew on the classic
texts of Greco-Roman civilization as well as the learning and
literature of the Islamic world. Indeed, the recovery of ancient
Lesson 3
93
arts and texts in Western Europe relied primarily on the flow
of cultural influences from Byzantium and from Moorish
(Muslim) Spain.
China and the Northern Barbarians
The End of the Tang Dynasty
The situation in China paralleled those of Byzantium and
the Muslim realm. All struggled to contain, evict, or manage
wave after wave of nomadic barbarian migrations and outright
invasions. All three realms attempted to use diplomacy, intimi-
dation, and bribery to manage the invaders. All attempted to
absorb the invaders into their culture.
The Rise of the Song and the
Barbarian Conquerors
In China in 879, the bandit leader Huang Chao seized the
Chinese imperial court to initiate a period of brutality and
carnage. In 907, Huangs successor, Zhu Wen, emerged from
this political chaos to claim power and effectively terminate
the Tang dynasty. Zhu Wens reign was buried, in turn,
under the assault of Turkic nomads in 923. Thereafter, the
Chinese Empire dissolved into several rival kingdoms.
However, the ideal of Chinese unity under a Mandate of
Heaven wasnt lost. China would survive even though much
of its northern territory was lost to nomadic peoples with
divergent political agendas. In 960, a general of one of the
mutinous armies was declared emperor. He would be the
first emperor of the Song dynasty, initiating a new imperial
era in China.
Map 12.4 on page 305 of your textbook shows the extent of the
Song Empire from 1050 to 1234 C.E.
A major point to keep in mind is that regions north of Song
control spawned a number of states rooted in the Asian
steppes. Among these, the Jurchen conquered and gained
control of pretty much all of China north of the Huai River.
The Jurchen also adopted essential aspects of Chinese
culture, such as a Confucius-based civil service and Chinese-
World History
94
style urban planning. In that light, the Song Empire was
forced to grant Jurchen sovereigns imperial rights under
the traditional Chinese notion of the Mandate of Heaven.
Economy, Society, and Arts and
Learning under the Song
As the Song were forced to coexist with Jurchen rule in the
north, the axis of Song China shifted to the Yangtze valley. At
that point, the population of China was around 100 million,
so the empire had labor resources. What it needed was more
territory for food and fiber farming. To that end, Chinese colo-
nization was aimed at the underpopulated lands of Sichuan
to the southwest. Natives were pacified as forests were cleared
for tea plantations and mulberry cultivation to support the silk
industry.
As internal colonization proceeded, bogs were drained, roads
and bridges were built, and cities sprouted across the land
like mushrooms (following patterns found in Europe). In the
later years of the Song dynasty, an enlarged China was brim-
ming with wealth.
The Song era left an enduring intellectual and artistic legacy.
Take note of Ouyang Xius philosophy of learning and morality.
Consider the ways by which Wang Anshis ideas impacted state
policies. Think about why Chinese painters of this era created
some of the worlds most admired and imitated images. Study
The Night Revels painting depicted on page 308.
Conclude your study of this chapter with a few moments of
serious attention to the Chronology sidebar on page 310
of your textbook.
Please complete Self-Check 5 now.
Lesson 3
95
Self-Check 5
Indicate whether each of the following statements is True or False.
______ 1. In regions were Christians and Muslims had coexisted cooperatively, the Crusaders
wanted the Muslims and Christians to demonize each other.
______ 2. Under the rule of Basil II, the Byzantine Empire made peace with the Arabs while first
terrorizing Bulgaria and then cooperating with Bulgarian elites.
______ 3. After the fall of the Tang dynasty, the Song dynasty made no concessions to invaders
from the steppes.
______ 4. A political network that covered 57,000 square miles of Colorado, New Mexico, and
Arizona probably wasnt associated with the Chaco Canyon society.
______ 5. The Norsemen who voyaged across the Atlantic were trying to escape poverty and lack
of social opportunity.
______ 6. Cahokia, which was located east of present-day St. Louis, represented a culture that
featured many aspects of Mesoamerican civilization.
______ 7. The Crusader kingdoms along the coast of the eastern Mediterranean got support from
Italian merchant communities that welcomed access to trade.
Fill in the blank with the correct term.
8. In the Khmer kingdom, the city of _______ was designed to express both Hindu and Buddhist
beliefs.
9. In _______, the Chola kings expanded their territories through ruthless exploitation coupled
with the ambitions of merchant communities along the coast.
10. The _______ of Japan were supposed to be guardians of the emperor and protectors of the
empire.
(Continued)
World History
96
Self-Check 5
11. In Italy, the term _______ referred to citizen bodies that would become institutions of civic
government in the late eleventh and early twelfth centuries.
12. The Hanseatic League was a network of ports along the North Sea and the Baltic that
collaborated to promote _______ with other parts of Europe.
Answer each question in not more than four complete sentences.
13. Summarize some of the technological advances that occurred in Europe from about 1200 to
the 1300s, differentiating those originating in Europe from those that were imported from
abroad.
__________________________________________________________
__________________________________________________________
__________________________________________________________
__________________________________________________________
14. Briefly outline major factors that divided Western Christianity from Eastern Christianity.
__________________________________________________________
__________________________________________________________
__________________________________________________________
__________________________________________________________
Check your answers with those on page 218.
Lesson 3
97
THE CRUCIBLE
The second part of Lesson 3 covers Chapter 13, The World
the Mongols Made; Chapter 14, The Revenge of Nature:
Plague, Cold, and the Limits of Disaster in the Fourteenth
Century; and Chapter 15, Expanding Worlds: Recovery in
the Late Fourteenth and Fifteenth Centuries.
ASSIGNMENT 13: THE WORLD
THE MONGOLS MADE
Read this assignment. Then read the Part 6 opening spread
and Chapter 13, pages 312341, in your textbook.
The Mongols Reshaping Eurasia
Mongol-speaking peoples of the steppes are first noted in sev-
enth-century Chinese historical records. But the story of the
Mongols is no footnote. As Mongol chiefs made contact with
their wealthier neighbors, mainly as raiders or mercenaries-
for-hire, chiefs got rich on loot. As a result, a gap between
rich and not-rich appeared in Mongolia, which led the
wealthier clan chiefs to seek control over other clans.
As youve already learned, the rowdy, violence-prone peoples
of the steppes had a long history of raiding and looting their
wealthier sedentary neighbors. What made the Mongols dif-
ferent was the sheer, mind-bending scale of their conquests.
In the year 1206, a Mongol chief named Temujin united the
Mongol clans and declared himself the ruler of all who dwell
in the felt tents. He would be known to history as Genghis
Khan. At its greatest extent, the realm of the Great Khan would
extend over more than 90 degrees of longitude, swallowing
Persia, Russia, China, and the Silk Roads.
Map 13.1 on pages 318319 of your textbook is a key resource
for getting the most from this chapter. Study it carefully to
track the Mongol campaigns of the thirteenth century.
World History
98
You can think of the Mongol Empire as proceeding through
three stages. At the outset, the Mongols under Genghis Khan
did what they were used to doing. They arrived on the hori-
zon like a plague of mounted locusts to loot, rape, slaughter,
and pillage. The second stage featured offering conquered
peasants a deal they couldnt refuseprotection in exchange
for taxation. The third stage saw Genghis Kahn transformed
into an enlightened lawgiver whose advisors from many lands
aided him in administering the Mongol lands. This latter
stage gave rise to a brief but genuine Mongol Peace, a bit
like the Pax Romana (Roman Peace). In effect, the Mongols
became highway police across the 3,000 miles of the empire.
Check out Map 13.2 on page 322 of your textbook to consider
European travelers of the Mongol roads between 1246 and 1295.
Take some time to study the typical Mongolian Passport shown
on page 323.
In the last part of this section, youll get some insight into
Mongol society. As you enjoy the tour, take note of several
significant points. The successors of Genghis Kahn tended to
be tolerant, pliable, and adaptable to new situations. For
example, rulers like Temujins grandsonthe fabled Chinese
emperor Kublai Khanheld a belief in one god expressed dif-
ferently in the different religions. Above all, recognize that the
Mongol Peace permitted the adventurous explorations of
Marco Polo and a historically crucial period of cultural
exchange.
The Mongol World Beyond the
Steppes: The Silk Roads, China,
Persia, and Russia
The Mongols were partial to merchants and encouraged trade
along the Silk Roads. However, these trade routes werent at
all like interstate highways. Geographic obstacles included
the 1,800 mile stretch of the Taklamakan Desert, scarce
freshwater sources, and rugged mountain terrain. Human
obstacles included getting lost and running afoul of bandits.
Marco Polos journey from Venice to China took three and a
half years.
Lesson 3
99
European journeys to the East were fairly common during
the Mongol Peace. However, few Chinese subjects had much
interest in journeying to the West. An interesting exception
was a Chinese subject by the name of Rabban Bar Sauma.
Bar Sauma was a Nestorian Christian who passed on fasci-
nating records of visits that took him as far as Rome and
Paris.
Map 13.3 on page 327 of your textbook traces the travels of
Rabban Bar Sauma from 1275 to 1288. Use the map to locate
the boundaries of the four major divisions of the Mongol
Empire after the reign of Genghis Khan. Note the lands of the
Khanate of the Golden Horde, which extended from central
Asia all the way to Ukraine and the Balkans.
China
The Mongol conquest of Song China took a while. China was
the most powerful state the Mongols encountered, and it was
highly defensible. It took the mighty resources of the Mongol
Empire to conquer China bit by bit. Armies were imported
from the steppes. Persian engineers built the siege engines
that helped capture cities in southern China. The final battle
was at Changzhao in 1275. For the Chinese, the Mongol con-
quest stank of death and despair. Suicides were common.
Kublai Khan (12141294) was Chinas emperor when Marco
Polo arrived there. Marco Polo noted that the emperors gov-
ernance was loathed and hated by the Chinese. The Khans
lords and enforcers were barbaric steppelanders who con-
sidered mares milk akin to the nectar of the gods. Also, most
of them were Muslims. In any case, the China Project
absorbed so much of Kublai Khans resources and attention
that his bid to assume his grandfathers role was honored
only in word, but not in deed, by Mongol rulers in the far
west who called themselves Il-Khans (subordinate Khans).
Persia
The Il-Khans of Persia gradually broke away from traditions
of the steppes, eventually rejecting Genghis (and Kublai)
Khans toleration of religious diversity. In the context of
absorbing Persian culture, they converted to Islam in 1295.
World History
100
That turned out to be problematic. The Muslims of Persia
(Iran) were Shiites. The bulk of Arabs were Sunnis. More
important, as Mongol Persia became Muslim, it adopted
the militarism and religious intolerance of the Muslim world.
Christians, Jews, Buddhists, and others were no longer wel-
come. In any case, Mongol rule in Persia came to an abrupt
halt in 1343 when the last Il-Khan died without appointing
an heir.
Russia
Mongol governance in Russia was exercised by terror and by
extracting tribute. The Russian princes were allowed to run
provincial and local affairs in traditional ways. However,
Russian princes had to show up at the Mongol imperial court
on the lower Volga to unload copious tribute and submit to
humiliation on a regular basis. The situation was barely
tolerated by the Russians. Revolts, which were suppressed
by mass slaughter, werent infrequent. In fact, the Russian
princes mainly tolerated Mongol rule because they were
caught between a rock and hard place. To the west, powerful
European monarchies in Poland, Sweden, and Latvia were
flashing swords and agendas of conquest along the Russian
border. It was all the Russians could do to keep these fanatical
European knights at bay. They couldnt do that and contend
with the Mongols at the same time.
The Limits of Conquest: Mamluk
Egypt and Muslim India
Saladins heirs ruled Egypt after 1192, more or less success-
fully contending with pastoral invaders out of North Africa.
They went about this with the armed assistance of a slave
army called the Mamluks. The Mamluk saga is historically
peculiar. The slaves were predominately young Turkic boys
who were captured and sold to Saladins domain by Mongol
rebels. The boys were raised in military barracks and trained
in martial skills. With no access to escape routes, they
became absorbed into a unique Mamluk warrior culture. For
a time, they served as an effective military arm of Saladins
sultanate.
Lesson 3
101
In Egypt in the 1250s, the Mamluks rebelled, taking effective
control of the Sultanate. In 1260, the Mamluks met and
defeated a Mongol army at the Battle of Ain Jalut in Syria. It
was the first time a Mongol army had met defeat. Later,
between 1268 and 1291, Mamluks wiped out the last of the
Crusader states in Syria and Palestine. However, the Battle of
Ain Jalut was more historically important because it kept the
Mongols out of Africa. As a result, Egypt became a Muslim
threshold into Africa. Muslim influence would seep into the
Sudan and trickle all the way west to Nigeria.
Muslim India: The Delhi Sultanate
In the 1190s, Turkic Muslims of the Ghaznavid Empire were
invading Hindu India to extract tribute and set up military
garrisons. One of their strongest garrison outposts was at
Delhi in northwest India. Iltutmish, a Muslim military adven-
turer, took command there in 1211. By avoiding war with
the Hindus, as instructed by the Ghaznavid rulers, and by
building up his local resources, he felt emboldened to declare
himself an independent ruler in 1216. Over the next 12
years, he manipulated the rivalries of Muslim commanders to
construct what would be called the Delhi Sultanate, which by
1236 extended across India, from the Indus to the Bay of
Bengal.
Study Map 13.4 on page 332 of your textbook to see the extent
of the Delhi Sultanate. Note its area of influence over nearly all
of the Deccan (southern India).
Over most of the thirteenth century, Mongols threatened
Indias northern frontiers. However, Indias Delhi Sultanate
held fast despite having to tolerate a Mongol state west of
Delhi.
Europe
The era of the Mongols posed a looming threat along
Christendoms eastern boundaries. Venice lost its Byzantine
windfall. Even as the Crusaders were driven from
Constantinople by the Byzantine emperor in the 1261,
the former Byzantine Empire was more nostalgia than
fact. Meanwhile, Latin Christendom was gaining ground.
World History
102
The pagan worlds along the Baltic, from Finland and Prussia
to Livonia and Estonia, were conquered by Christian armies.
Christians also gained control of Castile and Portugal, forcing
the Moors back into southern Spain.
Meanwhile, the European acquisition and adaptation of
technologies from the East accelerated. Paper, invented in
China, was being produced on a large scale by the thirteenth
century. Gunpowder and blast furnaces (also invented in
China) helped transform both warfare and metallurgy.
Although European maritime technology lagged behind
that of developed Eurasia, a framework for correcting that
situation was developing. Today we recognize that framework
with a single wordscience.
European science developed in unique ways that combined
the interplay of several factors. Scholars were retrieving
classic works of the Greco-Roman period as well as crafting
new manuscripts and ideas. In an era when nature was
increasingly seen as Gods beneficent architecture, thought-
ful people increasingly turned to empirical observation and
measurement, attempting to study Gods handiwork up
close and personal. A lot of the work of these new natural
philosophers took place in the context of universities crop-
ping up in Paris, London, Milan, and other places across
Western Europe.
An outstanding thirteenth-century example is Thomas Aquinas,
who wrote major works in both theology and secular thought.
Another major intellectual figure was St. Francis of Assisi.
Youll want to think about how and why the Franciscan
monks and nuns helped spread the ideas of St. Francis
of Assisi throughout Christendom.
Europe led the world in two technological areas: glass
manufacture and the development of clockwork. Efforts to
create stained glass for cathedrals (a project invited by inno-
vations in church architecture) led to other ideas, such as
fabricating ground lenses. In turn, glass lenses led to the
development of microscopes and telescopes. In turn, the
marriage of magnification to precise measurement provided
by clocks led to startling discoveries, ranging from the
identification of microorganisms to mathematically precise
observations of heavenly bodies.
Lesson 3
103
All in all, Europe was birthing a new worldview. Over time,
that Western worldview would impact the entire planet.
Study Map 13.5 on page 334 of your textbook. Notice the sites
of universities and of churches featuring stained glass.
Conclude your study of this assignment by spending time with
the Making Connections feature on page 337. Use the table
to think about European innovations and transformations in
the thirteenth and early fourteenth centuries. Next, study Map
13.6 on pages 338339 to think about the importance of grass-
land environments, contrasting the Eurasian steppes with
grasslands on the other continents. Study the Chronology
sidebar on page 340.
ASSIGNMENT 14: THE REVENGE
OF NATURE: PLAGUE, COLD,
AND THE LIMITS OF DISASTER
IN THE FOURTEENTH CENTURY
Read this assignment. Then read Chapter 14, pages 342373,
in your textbook.
Climate Change
Humankind emerged and developed as the last major ice age
ended and the planet entered a relatively warm interglacial
period. Today, were still living in that interglacial period at a
time of accelerated climate and ecological change resulting
from human activity. However, from a historical perspective,
the most significant period of climate change is whats called
the Little Ice Age. As you read, note three things: First, the
cold periods werent full-blown ice ages; Earths temperature
variations in that period were within interglacial parameters.
Second, there were oscillations of colder and warmer periods
over these centuries. Third, although the Little Ice Age is
associated with severe setbacks, such as plague and drought,
it was also a period when people adapted to changed climates
in innovative ways.
World History
104
Study Figure 14.1 on page 344 of your textbook to track global
climate change through the Little Ice Age that extended from
the mid-1300s through the early 1800s. Map 14.1 on pages
346347 offers a global overview of climate change during the
fourteenth century. Notice the locations of major volcanic
eruptions.
The Coming of the Age of Plague
The long period of relative global cooling coincided with the
outbreak of plague in various parts of the world. Why this
was the case remains unknown or speculative. Some experts
suggest that the cooling patterns encouraged the adaptive
selection (mutation) of warm-era pathogens into new forms.
In any case, conventional ideas about the nature of the
plague, or plagues, are at best sketchy. Bubonic plague,
which was spread by infected rats, was no doubt a significant
factor in what came to be called the Black Death. However,
evidence suggests that several kinds of diseases were affect-
ing various populations.
Study the Making Connections feature on page 348 of your
textbook for a summary of climate change in Eurasia, Africa,
and the Americas.
The Course and Impact of the Plague
The era of plague began in China. However, its not clear how
disease outbreaks there were transferred to the west over the
Asian steppes. In this section, youll learn about various
plague experiences in different places. Note the astonishing
decline in populations in areas impacted by the Black Death.
Map 14.2 on pages 352353 of your textbook shows sites with
recorded outbreaks of the Black Death between 1320 and
1355.
Moral and Social Effects
When devastation strikes, people have a tendency to seek
out moralistic explanations and/or find someone to blame.
The most striking moralistic response to the Black Death
Lesson 3
105
in Europe was the widespread appearance of Christian flagel-
lants. Viewing the plague as divine punishment for the sins
of the people, flagellants would literally rip their backs to
bloody shreds with flails tipped by iron claws. These
extreme penitents saw themselves as recapitulating Christs
crucifixion in order to heal the sins of the world. However,
both the French king and the Pope outlawed these practices.
A relatively positive outcome of the era of plague was the
application of science and reason to understanding the dis-
ease(s) and seeking cures. In contrast, a negative outcome
was the brutal persecution of the Jews.
The Jewish people of medieval times tended to inhabit urban
areas and were often engaged in trade and banking. The lat-
ter occupational tendency was related to medieval strictures
on lending money at interest, a practice called usury. In
effect, Jews served a vital economic function because loans
were in high demand and many Christians didnt want to
dirty their hands by engaging in commerce. (In that respect,
medieval Christians reflected similar Confucian attitudes.) In
any case, as the Black Death spread, Europeans turned
against the Jews, typically citing that age-old ruse that the
Jews killed Christ. Because medieval law usually forced Jews
into walled ghettos, they were easy targets.
Map 14.3 on page 355 of your textbook shows the major cen-
ters of Jewish settlement and the path of the Diaspora, the
dispersion of the Jews from their original Semitic homelands.
The map covers the period from 1100 and 1400. Suggesting
an eerie foretaste of modern history, note the virulence of
anti-Semitism in Germany and Austria, as indicated by
massacres of Jews during this period.
The Limits of Disaster: Beyond the
Plague Zone
India
Areas beyond the Eurasian plague zone benefitted from rela-
tive advantage. In India, the Sultanate of Delhi got a boost
from the Mongol decline. Sultan Muhammad Ibn Tughluq
World History
106
(r. 13251351) pursued an agenda of conquest that took in
just about the entire subcontinent. At first, he wisely prac-
ticed religious tolerance, which was good idea in a land that
was mostly Hindu. However, his basic policy of conquest and
plunder didnt make for a stable empire. For one, he was
dependent on Turkic fighters, which was expensive. When he
began to default on his payments, his steppelanders began to
secede. Meanwhile, hard-line Islamists kept badgering him to
enforce Islam by force. Eventually, Ibn Tughluq declared
Hindus to be infidels. As a result, a Hindu empire arose in
the south that resisted further Muslim advance. The Sultanate
was weakened, and provincial states began dropping out of
the empire.
Southeast Asia
Recall that the island of Java had been high on the list of
imperial objectives for Kublai Khan. There was a reason for
that. The strait running between Malaya, Java, and Sumatra
was vital to Chinese trade in the region. The region was
dominated by a powerful state centered around the city of
Majapahit on the island of Java.
As the Mongol threat eased due to the Black Death, the influ-
ence of Majapahit expanded. In the 1340s, under King Hayan
Wuruk, Majapahits garrisons appeared on the islands of Bali
and Sumatra, as well as on the Malay Peninsula. Records
indicate that Majapahits influence was felt in ports across
mainland Southeast Asia. The basis of Majapahits influence
was the rich variety of spices and other trade goods produced
in this part of the world.
Study Map 14.4 on page 359 of your textbook. In India, note
the extent of the influence of the Delhi Sultanate. In Southeast
Asia, note the regions where Majapahit claimed tribute.
Japan
During the fourteenth century, Japan was outside the zone of
the Black Death. Thus, with the easing of the Mongol threat,
Japanese society was free to develop in relative isolation.
Lesson 3
107
The shoguns role was supposedly to defend the rule of
Japans emperor. In fact, as you may recall, real power was
in the hands of the shogun. Be that as it may, fourteenth-
century Japan was a violent tapestry of competing warlords.
The emperors actual power was limited. Emperor Godaigo
mounted efforts to restore central imperial governance in
1318.
A turning point in this political situation occurred when, in
1335, the powerful warlord Ashikaga Takauji claimed the
role of shogun against the wishes of the emperor. Godiagos
supporters rose up in his support only to be defeated by
Ashikaga at the Battle of Minato River in 1336. The Ashikaga
dynasty remained in power for the rest of the century, mainly
by avoiding excessive intervention in the affairs of major
warlords.
Because central governance was either weak or absent,
peasants were left to tend to their business under the osten-
sible rule of provincial aristocratic or warrior landlords. As a
result, Japanese peasants were able to improve their farming
techniques, often extending them to marginal lands in a
process of internal colonization.
Largely because of an influx of Chinese Buddhist Monks
fleeing hard times in China, a mystical branch of Buddhism,
called Zen, was introduced into Japan. Zen practice focuses
on approaching transcendence though the mind through
ruthless discipline and an attitude of detachment from the
slings and arrows of outrageous fortune. The Japanese
warrior class, the samurai, found this kind of discipline
amiable and appropriate to the business of war.
Japanese women lost ground during this period. Whereas
earlier customs gave women relative equality in marriages
based on love and sex, marriage now became a game of
political alliances between rival political and warring factions.
In that context, women became viewed as male property.
World History
108
Mali
According to legend, a heroic figure by the name of Sundiata
founded the independent Islamic kingdom of Mali in the early
1200s. His realm featured rich resources, armored cavalry,
and well-administered courts and cities. Malis great age of
conquest took place in the 1260s and 1270s.
A king of Mali was addressed as Mansa. The most famous
of the kings of Mali was Mansa Musa. Scholars and poets
from Spain and North Africa were counted among his
courtiers. Following the practice of his predecessors, Mansa
Musa made the sacred pilgrimage to Mecca. He spent three
months of his journey in Egypt; while there, he endowed
countless mosques and gave so many gold coins to the
Mamluk Sultan that the local value of gold currency was
deflated.
In foreign eyes, gold was the color of Malis glory. Whats
interesting is that Malis merchants and officials were basi-
cally intermediary handlers of gold from secret mines beyond
Malis borders. Its value in Mali proper, aside from glorifying
the realm, was as a commodity that was exchanged for salt.
Salt was a scarce resource in Mali. In any case, tales of gold
associated with Mali glittered in the imaginings of Europeans,
helping to prompt voyages of exploration. The sources of
African gold werent found until much later, but the voyages
would yield other rewards.
The Pacific: Societies of Isolation
This part of the chapter offers a brief tour of societies isolated
within and along the boundaries of the immense expanse of
the Pacific Ocean. As your read, consider the following
questions:
n How did the Polynesian peoples of remote Easter Island
manage a resource-limited environment in ways that
appear to have ended badly?
n How is it that the Polynesians who settled New Zealand
(known as the Maori) went through a process of social
evolution that converted hunters into pastoralists under
the increasing power of warrior chiefdoms?
Lesson 3
109
n How did the Ozette people of North Americas Pacific
Northwest develop such a sophisticated society in
conditions of serious isolation?
n Why was the relatively advanced expansionist state of
the Chim people of Perus coastal desert region so
culturally distinct from the Moche culture found in that
region in much earlier times?
In the concluding Perspective part of this chapter, study the
comparative populations of Europe, China, and the Islamic
World from 100 to 1500 C.E. in Figure 14.2 on page 371 of
your textbook.
ASSIGNMENT 15: EXPANDING
WORLDS: RECOVERY IN THE
LATE FOURTEENTH AND
FIFTEENTH CENTURIES
Read this assignment. Then read Chapter 15, pages 374407,
in your textbook.
Fragile Empires in Africa
East Africa
In East Africa, Ethiopia reemerged and expanded, benefitting
from its domination of the Great Rift Valley. In 1403, aiming
at expanding Indian Ocean trade, Ethiopia recaptured the
port of Massaweh. However, by 1469 efforts to expand the
borders of the empire ended, even as internal expansion
continued. As youve seen in other cases, internal expansion
was largely focused on developing marginal lands for food
and fiber production.
Further to the south, between the Zambezi and Limpopo
Rivers, archaeological evidence attests to participation in
Indian Ocean trade. Along this eastern flank of Africa, port
cities developed along the coast. However, the centers of
World History
110
wealth and power were located inland in fortified stone-
crafted administrative centers called zimbabwes. The most
impressive of these was the Great Zimbabwe. It would serve
as the center of a kingdom founded by a leader who called
himself the Mwene Mutapa, which means lord of the tribute
payers.
West Africa
Developments in West Africa included the emergence of sev-
eral new kingdoms. As the kingdom of Mali fell into decline,
Songhay emerged as the most powerful of the West African
States. Like Mali, Songhay benefitted from trans-Saharan
trade, but not to as great an extent as did Mali under Mensa
Musa. Other significant West African kingdoms included
Benin, which benefitted from the easily navigable Niger River,
and the Kingdom of Kongo in the delta region of the Congo
Basin. The latter society rapidly adopted the culture and
religion of visiting Portuguese traders.
To understand what was going on in Africa between 1400 and
1500, study Map 15.2 on page 380 of your textbook. After
locating the various states, note the trans-Saharan trade
routes as well as the Indian Ocean sea lanes between Africa
and India.
Ecological Imperialism in
the Americas
The Inca Empire
Beginning in about the mid-1400s, the Inca Empire
expanded in much the same way as had earlier Andean
states. Tribute domains were established to take advantage of
the many microclimates, ranging from high plateaus to
Pacific coastal regions and even into rainforest climes east of
the Andean ranges. The basic imperial strategy was estab-
lishing trade and tribute zones such that crop failures in
one microclimate could be offset by good harvests in other
microclimates.
Lesson 3
111
Establishing their capital at Cuzco (in Peru), the Inca expanded
in a rather haphazard way to embrace a long, thin imperial
region that extended from Ecuador to Chile. The high Inca,
invested with authority as the Great Sun, demanded sub-
servience and tribute through brutal militarism. Whole
populations were relocated or slaughtered in the name of
imperial expansion.
The Aztec Empire
Around the same time as the Inca Empire, and also based on
exploiting diverse climate zones, the Aztec Empire emerged.
The Aztec state was supported by conquest and tribute.
However, the many tribute communities were left to manage
their own internal affairs, which permitted a flow of trade
within and between imperial domains. Although the Aztec
civilization was literate and complex, people tend to remem-
ber their penchant for human sacrifice. One of the main
items of tribute was slaves. On ceremonial occasions, slaves
were offered as human sacrifice to the Aztecs rather bad-
tempered gods.
Study Map 15.3 on page 383 of your textbook to better under-
stand the Inca and Aztec Empires. In the case of the Inca, note
the long chain of towns and cities extending north and south
from Cuzco. The Aztec capital, Tenochtitlan, was built in and
around a lake (Texcoco). Today, the lake is mostly memory,
and the ruins of Tenochtitlan lie beneath modern-day Mexico
City. Recall that both the Inca and the Aztec saw themselves
as inheritors of the legacies of earlier civilizationsTeotihuacan
and Tula for the Aztecs and Tiahuanaco for the Incas.
New Eurasian Empires
The Russian Empire
In the 1400s, the princes of Moscow established a domain
called Muscovy. It would give rise to the first stable empire to
appear in this region of Eurasia, in large measure because it
controlled the trade artery of the Volga River. Under the reign
of Ivan the Great (r. 14621505), the Russian Empire
World History
112
extended from the Arctic Ocean to the Caspian, casting
shadows on Poland, Lithuania, and Sweden. With the con-
quest of Constantinople by the Turks, Ivan aspired to replace
Byzantium as the Third Rome. That was mostly hubris, but
Ivan did import Italian technicians and marry a Byzantine
princess to substantiate his claim.
Study Map 15.4 on page 387 of your textbook to consider the
broad extent of the Russian Empire.
Timurids and the Ottoman Empire
The best way to think about the Timur and Ottoman Empires
is by comparing them. For example, both were Islamic pow-
ers, and both inherited the traditions and worldview of the
steppes. However, there the commonalities end. Timur the
Lame (known to the West as Tamerlane) would emulate
Alexander the Great and Genghis Khan, aiming for no less
than world domination. At its height, the Timur Empire
extended from the boundaries of the Ottoman Empire,
across Iran (Persia) and Iraq and deep into northwest India.
However, after he died in 1406 en route to conquer China,
the Timur Empire fell apart. In India, Tamerlane had a
far-reaching impact on the Delhi Sultanate, with one of
Tamerlanes heirs founding Indias Mogul Empire.
The Ottomans started out with the steppe worldview.
However, they soon absorbed the cultures of the peoples they
conquered. They adopted gunpowder-based warfare, making
effective use of heavy artillery against city walls. They even
took to the sea, becoming a dominant maritime power in the
Mediterranean. In 1451, Mehmet conquered Constantinople.
Thereafter the Ottomans, like the Russians, aspired to
become the Third Rome. That didnt happen. However, suc-
cessful Ottoman invasions did manage to conquer Greece as
well as large chunks of the Balkans and areas along the
lower Danube, including much of Romania.
Study Map 15.5 on pages 388389 of your textbook and con-
sider the extent of the Timur and Ottoman Empires. Notice
their relationship to the Mamluk Sultanate. Also, use the
legend to track the major Timur campaigns and their dates.
Lesson 3
113
The Limitations of Chinese Imperialism
In the middle of the fourteenth century, Chinas peasants
bore the brunt of bad times. Plague continued to flare up,
and would do so as late as the 1350s, when people began to
acquire natural immunities. Major flooding the Yellow River
required exhausting peasant labor to restore the Great Canal
system. A sustained drought followed the flood, wreaking
havoc on farming.
In bad times, the common people tended to look heavenward in
search of antidotes to despair. Millennial prophecies appeared.
Often these popular mythologies proclaimed the imminent
end of the world and/or the arrival of a great hero who would
strike down oppressors and seed justice across the land.
The world was going to end or be reborn. In China, peasant
folklore looked backward to a supposed golden time under
the Song dynasty, before the arrival of Mongol overlords. A
favored myth of deliverance foretold the return of the last
Buddha, Lord Maitreya, who would make everything better.
In 1351, a pretender to the throne claiming descent from the
Song was murdered by the Mongols. His peasant followers
mounted a violent rebellion. The fever spread. Out of the
chaos of armed rivalries, Zhu Yuanzhang emerged as the
leader of the rebellion. By cleverly managing a coalition of
rivals that boosted him to power, he renounced the cult of
Maitreya to proclaim himself the first emperor of the Ming
dynasty. His propaganda declared that he himself was the
promised savior. The adoption of the word Ming flavored that
assertion, because Ming (meaning bright) was associated
with the Lord Maitreya.
Zhu Yuanzhang was a self-made, self-educated man. Although
he retained most of Chinas traditional administrative and
governing institutions, he scorned Confucian ideals. Confucians
maintained that proper governance should focus on the wel-
fare of the people and steer clear of foreign entanglements.
Zhu Yuanzhang was interested in imperial expansion. As a
result, his son, the Yongle Emperor (r. 14021424) initiated a
brief but consequential period of Chinese expansion. Its in
that context that youll learn about the amazing voyages and
ventures of the Muslim eunuch Admiral Zheng He. Beginning
in 1405, he led the first of a series of naval ventures that
World History
114
imposed Chinese influence all across the Indian Ocean trad-
ing zone. Under Zheng He, the largest tribute-collecting
vessels imaginable (before the era of oil tankers) plied the
Monsoonal currents as far west as Africa and as far south
and east as Java and Sumatra.
In the end, Chinas dalliance with maritime imperialism was
terminated. Zheng Hes next proposed voyage was cancelled,
and Chinese policies turned inward.
Study Map 15.6 on page 393 of your textbook to better under-
stand Ming China and the voyages of Zheng He.
The Beginnings of Oceanic
Imperialism
Europeans recognized the wealth and profits to be gained
through access to the great Indian Ocean trade zone. But the
Muslim world controlled overland access to Indian Ocean
ports. So, what to do?
Europes only feasible access to seaborne exploration and
trade was the Atlantic Ocean. Recognizing this fact, some
Europeans began a trial-and-error process of mounting voy-
ages into the Atlantic. John Cabot (an Italian) discovered a
relatively unreliable route across the North Atlantic. But it
was mainly the Portuguese, with help from Italian bankers,
who made the most initial progress. In the 1480s, Portuguese
voyagers had made trade contact along the coast of West
Africa, as far south as the Kingdom of Kongo. By 1500, Vasco
da Gama and Cabral had reached India.
Meanwhile, with the backing of Italian bankers and Spanish
monarchs (Isabella of Castile and Ferdinand of Aragon)
Columbus made his way to the Caribbean in 1492. Thereafter,
mainly flying the flag of Spain, Europeans learned that
although the New World wasnt the Indies, it offered brand
new shores for conquest and rich resources for export. Silver
and gold mines in Spanish-controlled regions of Mexico and
Bolivia flooded Europe with precious metals, thus providing
hard cash to fund trade, exploration, and conquest.
Lesson 3
115
Examine Map 15.7 on pages 396397 of your textbook. The
key to where one could sail and safely return depended on
recognizing wind and ocean currents. As you study this map,
imagine that you wanted to sail from Europe to the Caribbean
and back again. How would you chart your course? Map 15.8
on pages 400401 offers an overview of European oceanic
exploration up to 1500.
The European Outlook: Problems
and Promise
Renaissance means rebirth. Traditional notions of the
European Renaissance have tended to focus on flourishing
innovations in philosophy, literature, science, and the arts
that mainly started in Italy in the 1300s while Byzantium
was still a major cultural influence. Thereafter, similar inno-
vations spread northward to Flemish, Germanic, and English
realms well into the 1600s. Your textbook emphasizes a more
recent historical perspective; namely, that aspects of rebirth
were cropping up as early as the 1200s or even a bit earlier.
A major aspect of the Renaissance was a philosophical world-
view called humanism. Humanism can be thought of as a
worldview that places humankind and human experience at
the center of reflections and speculations on the natural
world and human destiny. Humanistic perspectives showed
up in various ways, impacting art, literature, philosophy,
science, and even music and drama. In the arts, for example,
ordinary people began to replace mythical gods and heroes as
subjects. At the same time, benefitting from improved under-
standings of perspective, artistic renderings became more
lifelike and realistic.
Chivalry was an imposing aspect of popular culture both
before and during the Renaissance. The code of chivalry
emerged in the context of romantic ballads, tales of unre-
quited Platonic love, and dragon-slaying heroes saving fair
maidens. In the fifteenth century, romantic chivalric lore
inspired adventurers to honor their idealized fair maidens
with daring feats.
World History
116
Now, review the material youve learned in this study
guide as well as the assigned pages in your textbook for
Assignments 1115. Once you feel you understand the
material, complete Self-Check 6. Then check your answers
with those provided at the end of this study guide. If youve
missed any answers, or you feel unsure of the material,
review the assigned pages in your textbook and this study
guide. When youre sure that you completely understand the
information presented in Assignments 1115, complete your
examination for Lesson 3.
Self-Check 6
Indicate whether each of the following statements is True or False.
______ 1. For Marco Polo and other Western travelers of the Silk Roads, the Taklamakan Desert
was a major geographic obstacle.
______ 2. Emperor Yongle sought contact beyond Chinas empire, which led to the amazing
voyages of Zheng He.
______ 3. Vasco da Gama managed to round Africa and complete a voyage to India shortly before
Columbus reached the New World in 1492.
______ 4. The rulers of the Aztec Empire mainly left communities alone as long as they paid
tribute.
______ 5. In 1260, the Turkic Mamluks, who had taken over rule of Egypt, delivered the first
significant defeat to the Mongols at the Battle of Ain Jalut.
(Continued)
Lesson 3
117
Self-Check 6
______ 6. The rulers of the African Kingdom of Kongo were repulsed by Portuguese visitors and
their strange Christian religion.
______ 7. The Empire of Genghis Kahn encompassed the steppes of central Asia, Russia, Persia,
China, and Japan.
______ 8. In Persia, the Il-Khans eventually adopted Buddhism instead of Islam.
Fill in the blank with the correct term.
9. The legendary leader Sundiata founded the kingdom of _______ in the 1300s.
10. Although global ________ change occurred along with the arrival of the age of plague in the
fourteenth century, its not clear how the two may have been linked.
11. During turbulent times, _______ religious prophecies proclaim an imminent major cata-
strophic change that will punish the sins of the rich and empower the poor.
12. Metaphorically, the Ottoman Empire was to a monsoon as the Timur Empire was to a
_______.
13. As a Franciscan friar and a professor at the University of Paris, Roger Bacon maintained that
empirical science should question traditional religious or secular beliefs. Bacon was influenced
by St. _______ of Assisi, who taught that the world made Gods creation manifest.
14. The idealistic and romantic code of _______ was even more important than humanism in
stimulating overseas exploration.
15. Muscovy was the original name of the _______ Empire.
16. In the thirteenth century, northern India was ruled by the Muslims under the _______
Sultanate.
(Continued)
World History
118
Self-Check 6
Answer each question in not more than four complete sentences.
17. Who was Rabban Bar Sauma, and why is he an interesting historical figure?
__________________________________________________________
__________________________________________________________
__________________________________________________________
__________________________________________________________
18. What were zimbabwes?
__________________________________________________________
__________________________________________________________
__________________________________________________________
__________________________________________________________
19. What was Zen, and why did it flourish in Japan during the fourteenth century?
__________________________________________________________
__________________________________________________________
__________________________________________________________
__________________________________________________________
Check your answers with those on page 219.
Convergence,
Divergence, and Global
Enlightenments
The first part of Lesson 4 covers Chapter 16, Imperial
Arenas: New Empires in the Sixteenth and Seventeenth
Centuries; Chapter 17, The Ecological Revolution of the
Sixteenth and Seventeenth Centuries; Chapter 18, Mental
Revolutions: Religion and Science in the Sixteenth and
Seventeenth Centuries; and Chapter 19, States and
Societies: Political and Social Change in the Sixteenth
and Seventeenth Centuries.
ASSIGNMENT 16: IMPERIAL
ARENAS: NEW EMPIRES IN THE
SIXTEENTH AND SEVENTEENTH
CENTURIES
Read this assignment. Then read the Part 7 opening spread and
Chapter 16, pages 408433, in your textbook.
Maritime Empires: Portugal, Japan,
and the Dutch
The economic purpose of early empire-building was gaining
control of lands and the peoples who produced valuable
resources. That was the case up until about 500 years ago.
Later, empire-building involved additional motives.
Study the Making Connections feature on page 413 of your
textbook to compare land and maritime empires. All of the
nations discussed in this section were engaged in maritime
imperialism.
119
L
e
s
s
o
n

4
L
e
s
s
o
n

4
World History
120
The Portuguese Example
As European shipbuilders gained new skills, access to the
Indian Ocean trade zone became firmly established. Portugal
had an advantage because it had Atlantic ports in southern
Europe. Portugal established direct trade relations with
pepper-growing regions of India by 1500. By 1510, it had
established a trade center at Goa on the west coast of India.
It established trading connections with regions all over East
Asia as far as Japan.
By the 1600s, Portugal proclaimed its own Indian state,
which was a string of trading ports along Indias western
coast. An interesting aspect of Portuguese colonialism was
intermarriage with Indian and African women. Although
pondering what that might have meant in terms of cultural
interactions, keep in mind that the European traders were
motivated by an immensely profitable access to textiles,
medicines, and spices that werent otherwise available to
Europeans.
Take a look at the illustration on pages 414415 of your
textbook for a European perspective on Goa.
Asian Examples
In Asia, Japans governance and foreign policies are well illus-
trated by Toyotomi Hideyoshi, the warlord who took over Japan
in 1585. Hideyoshi proclaimed himself the gods choice for ruler
of the world, sending intimidating messages to the Chinese and
the Spanish governor of the Philippines at Manila. His efforts
to subjugate Korea failed. However, Hideyoshi did succeed in
making Okinawa and the Ryukyu Islands Japanese dependen-
cies. By the mid-1600s, Japan was contending with Russia over
territorial claims in the region of Manchuria. In the end, that
uncertain contest led to Japanese policies of containment, even
prohibiting Japanese people from traveling abroad.
Map 16.1 on page 416 of your textbook will help you under-
stand maritime imperialism and trade routes over the period
1500 to 1700.
Lesson 4
121
An important economic and social aspect of this period was
Chinese and Japanese migration. As economic refugees
fleeing poverty and oppression in their native lands, they
streamed into many regions of Southeast Asia, including ter-
ritories under the political control of the Dutch and Spanish.
The Dutch Connection
After their arrival in 1498, for about 100 years the Portuguese
managed to fit into the Asian-dominated regions they settled
in the name of trade. From about the 1620s onward, as the
English, French, Dutch, and other Europeans were contending
against dug-in Portuguese interests, Asians grew impatient
and hostile. In particular, because Portuguese Christians
were intolerant of Muslims and Jews, Asians saw fit to
liquidate their influence by force or through intimidation.
In this maelstrom of European competition for trade advan-
tages, the Dutch would rise to regional ascendancy. Youll be
invited to think about how this happened.
Unfortunately, the story is complicated. At the outset, the
Netherlands was a patchwork of rival petty states that
identified themselves with the peculiar environment of the Low
Countries and a common resentment of their official monarch
the King of Spain. The rise of the Dutch, culminating in the
founding of the Dutch Republic, was all about gaining the
same kind of advantage that had initiated Portuguese mar-
itime ambitions while struggling to overcome the power of
Spain and managing internal civil war.
In the end, the Dutch supplanted the Portuguese and bested
other European rivals for at least four reasons. In navigational
terms, Dutch maritime routes were more direct and efficient.
The Dutch benefitted from the loss of Portuguese hegemony
in India and across Southeast Asia. Brutal military force was
employed selectively to gain or sustain political control. In
effect, the Dutch shifted from maritime imperialism to territorial
imperialism. Finally, the Dutch had a favored trading partner-
ship with the Japanese. At that time, Japan was a crucial
source of silver. The Dutch could buy silver cheap, buy mer-
chandise elsewhere at bargain rates, and reap serious profits.
World History
122
A key financial innovation also figured into the Dutch imperial
advantage. Based on a proposal drafted by Jan van Linschoten,
an archbishop at Goa, the merchants of Amsterdam organized
a joint stock company aimed at creating a Dutch monopoly of
Asian trade. Thus was born the Dutch East India Company.
Land Empires: Russia, China, Mughal
(Mogul) India, and the Ottomans
The word Czar is the Russian version of Caesar. The Russian
Czars quest for empire entailed interwoven aspects. Control of
the entire length of the Volga River from near Finland to the
Caspian Sea assured domination of trade routes. Conquest of
the trade routes assured exclusive Russian access to the track-
less expanse of Siberia and the highly profitable fur trade.
Russian adoption of Western military technology greatly
assisted the domination of lightly armed Siberian states and
tribes.
Russian expansion was checked as Russians encountered
Chinese armies along the Amur River. In 1689, Chinese and
Russian emissaries drafted the Treaty of Nerchinsk to formal-
ize a boundary that continues to this day.
Meanwhile, Chinese imperial expansion involved claiming
and settling Mongolia to the north and Sichuan to the south.
To a greater extent than was the case in Russia, Chinese
imperialism aimed at absorbing conquered territories into
Chinese culture.
In 1526 a warrior prince by the name of Babur abandoned
efforts to restore the Timur Empire and settled for conquering
Delhi in northwest India. Baburs state, with its capital at
Delhi, remained fragile until his grandson Akbar ascended to
power (r. 15561605). The Mughal Empire in India followed
the old-fashioned imperial recipe. Its main occupation was
war. To sustain its armed might, dominated states and
provinces were allowed to maintain their local customs, reli-
gions, and modes of governance as long as they surrendered
tribute and paid hefty taxes. Akbars reign was mottled by
rebellion. Nevertheless, Mughal power continued to spread
and under the emperor Aurangzeb (r. 16561707) covered
most of the subcontinent.
Lesson 4
123
The Ottomans heartland was the resource-rich Anatolian
plateau (Turkey), and they are considered among the most
effective empire-builders in history. The domains of their con-
quests were so diverse that citizens were exposed to as many
ten different languages. Their empire encompassed three
waterways: the eastern Mediterranean, the Black Sea,
and the TigrisEuphrates river system. Geographically,
particularly under the reign of Suleiman the Magnificent
(r. 15201566), Ottoman Sultans ruled Egypt, great stretches
of territory on either side of the Red Sea, North Africa as far
as the Barbary Coast, the Near East, Greece, and the
Balkans a bit beyond Belgrade.
New Land Empires in the Americas
From the late 1600s and into the 1700s, the French, the
English, and the Dutch were making inroads into North
America, setting up colonies near coasts or around water-
ways. The Portuguese and the Dutch were making inroads
along the coast of Brazil, but the earliest and most remark-
able tales of conquest were spoken in Spanish.
Columbian conquests of parts of the Caribbean took place
after 1492. Columbus conquest of Hispaniola (14951496)
was the opening round of a period of Spanish conquest that
would subjugate the most densely populated regions of the
New World by the mid-1500s.
One cant but wonder how virtual handfuls of Spanish
conquistadores could have pulled this off. Even though
Spanish terrorism against Native people in Mexico and Peru
gets a lot of press, the process of subjugating Native peoples
under the flag of Spain was mainly peaceful. Why? Spanish
weapons were a factor, but not the main one. Native vulnera-
bility to European disease pathogens was a factor, but not
the main one. It seems the main factor was being in the right
place at the right time. Both the Aztec Empire in Mexico and
the Andean Empire of the Inca were being torn asunder
by civil war when the Spanish arrived. From the Native
perspective, the Spanish were vital allies in overthrowing
hated Aztec and Inca overlords. In short, the Native peoples
were primed and ready for alliances with the Spanish that
World History
124
would aid their cause. Another interesting factor is the so-
called stranger effect. Among peoples like those ruled by the
Aztec and the Inca, it was customary to honor strangers
outsidersas a source of knowledge and as potential
intermediaries for settling their internal disputes.
Map 16.3 on page 427 of your textbook will help you locate
and identify Spanish, Portuguese, English, French, and Dutch
possessions in the New World. The Dutch made a deal with
the English to trade New Amsterdam for Surinam around
1664. At that point, New Amsterdam was renamed after the
Duke of York to become New York.
Making the New Empires Work
The focus of this section is on attitudes toward Native
Americans under differing imperial regimes. In the Spanish
colonies, the dense indigenous populations were the eco-
nomic foundation of the imperial economy. Thus, Spanish
policy dictated preserving the Native population. This was
not the case elsewhere. In Brazil and some parts of North
America, Native Americans were used as slave labor. But the
British domains illustrated the more dominant pattern.
Adopting the rationalization that indigenous peoples were
heathens and savages, they were massacred or driven from
their lands.
The Global Balance of Trade
Two topics make up this brief section. First, were reminded
that patterns of precolonial internal tradereferred to as
country tradescontinued. An example was the internal
trade in furs and pelts that linked Native American tribes
with French trappers. The second topic addresses how
seaborne trade in this period gave rise to the need for explo-
rations in search of new routes as well as the development of
new inland and port cities related to the global expansion of
trade.
The Chronology sidebar on page 431of your textbook can
help you identify key dates.
Lesson 4
125
ASSIGNMENT 17: THE
ECOLOGICAL REVOLUTION OF
THE SIXTEENTH AND
SEVENTEENTH CENTURIES
Read this assignment. Then read Chapter 17, pages 434461,
in your textbook.
The Ecological Exchange: Plants
and Animals
On a high plane of the Colombian Andes, 8,600 feet above
sea level, lies the city of Bogot, Colombias capital. As the
sun sets and the chill of high altitude makes the air sharp
and crisp, the scent of eucalyptus smoke teases the senses.
That wouldnt have been the case a thousand years ago for a
simple reason: Eucalyptus trees are native to Australia.
Over millions of years, natural selection differentiated species
around the globe. And then along came humankind with its
proclivity for selective selection. The ecological revolution of
the sixteenth and seventeenth centuries was the most signifi-
cant instance of human agency ever wrought by humans. An
ancient pattern was reversed as people began to exchange life
forms from continent to continent in what historians now call
the Columbian Exchange. On pages 436442 of your textbook,
youll learn about some of the more important plants and
animals that were part of this exchange and their impacts on
the larger human population.
Map 17.1 on pages 438439 of your textbook summarizes a lot
of information youll want to think about, including major routes of
ecological exchange and genetic differences among populations
related to immunities and vulnerabilities to microbial pathogens.
World History
126
The Microbial Exchange
Demographic Collapse in the New World
Ecological and microbial (germ) exchange were significant fac-
tors in the rise of a new global order. On the whole, ecological
exchange tended to make people healthier, but there were
definitely winners and losers. Within the latter category, the
horrific die-off of Native Americans is, without doubt, the
most shocking.
The peoples of the Americas had been separated from their
human cousins for thousands of years. When the Europeans
arrived, they had no immunity for diseases that were com-
mon among European populations. Smallpox seems to have
been the major killer in Mesoamerica. Influenza was unwit-
tingly spread to Native peoples by French visitors to the St.
Lawrence region of Canada. No one knows how many died.
Measles and smallpox combined to kill of very large numbers
of Native American who made contact with British colonists.
Modern mortality estimates suggest that between 60 to 90
percent of Native peoples of the New World succumbed to
European diseases.
Study the feature box on page 445 of your textbook to think
about the Colombian Exchange.
Plague and New Diseases in Eurasia
Microbial evolution brought about more virulent strains of
known diseases, like plague, as well as new diseases. In the
1680s, its estimated that 80 percent of the tribal populations
of eastern Siberia died from exposure to outsiders from the
developed world. Tuberculosis arose from somewhere and
became a major European killer in the sixteenth century.
Strains of venereal disease, possibly gifts from the New
World to the Old, appeared in Europe after contact with the
Americas. Plague continued to trouble regions of southern
Europe, such as Venice, during the sixteenth and seven-
teenth centuries. People continued to die from typhus and
dysentery due to a lack of understanding of public sanitation.
Lesson 4
127
As Europeans found their way into tropical lands, they met
with lethal tropical diseases such as cholera, yellow fever,
and malaria.
Labor: Human Transplantations
Colonies in the Americas suffered from nagging labor shortages.
Aside from their tendency to die from European diseases,
Native Americans werent well suited for work as field hands,
house servants, or miners. And even though Europeans were
imported as indentured labor, these efforts didnt provide as
many workers as were needed. Of course the fact that new
arrivals in the Americas tended not to live very long didnt
help matters.
As early as 1500, the Spanish colonies began importing labor
from a source what was, in terms of ocean navigation, rela-
tively close at handAfrica. By the 1570s, Spaniards were
importing about 2,500 African slaves a year. Over the next
century and a half, slave importation to Spanish and
Portuguese American ranged around 3,500 slaves per year.
For a while, Spanish and Portuguese control of the African
slave trade denied slaves to the other European powers. That
situation changed gradually as pirates had their way and as
agreements were reached with the Dutch, the English, and
the French. In the final quarter of the 1600s, some 24,000
slaves were being shipped from Africa every year.
African slaves became the fundamental labor force on
plantations in subtropical North America as well as in the
Caribbean and in South America. This was largely due to
the fact that sub-Saharan Africa had long been exposed to
European pathogens and because Africans could deal with
the climate.
World History
128
Wild Frontiers: Encroaching Settlement
Northern and Central Asia: The Waning
of Steppe Imperialism
The influence of the steppelanders declined after 1600.
Aspirations to emulate Genghis Khan simply didnt material-
ize, perhaps because Persians and other peoples bordering
the steppes adopted gunpowder and better-engineered
fortifications. This development is referred to as the military
revolution. In addition, nomadic pastoralists of the steppes
began to give up pastoral life in favor of life in settlements.
A paradoxical exception to Mongol decline occurred in China.
In the 1640s, a Manchu Army composed of steppelanders
was charged with serving the Chinese Empire as defenders
against Mongol intruders. However, when civil war broke out
in China, the Manchu stepped in to settle the matter by over-
throwing the last Ming emperor in 1644. The Manchu, who
declared themselves the Qing dynasty, would eventually
absorb Chinese culture and end up overthrowing Mongol rule
in Tibet. Qing rule would last until 1912.
Map 17.2 on page 447 of your textbook will help you track and
locate imperialist ventures and settlement in Eurasia from
1600 to 1725.
Pastoral Imperialism in Africa and the Americas
Pastoral ways of living based on managing herd animals con-
tinued in some regions of the globe, such as North Africa and
the African Sahel. At the same time, hunting and gathering
life-ways continued among Australian Aboriginals and among
tribes scattered from the Great Basin to California in what
would become the United States.
Pastoral imperialism appeared in North Africa when in 1588
the Moroccan Sultan Al-Mansur successfully conquered
the West African kingdom of Songhay. In the Americas,
something akin to pastoral imperialism appeared over
the sixteenth and seventeenth centuries as great herds of
cattle, sheep, and cattle were transported to Spanish and
Portuguese America and to the Pampas of the Argentine.
Lesson 4
129
To a degree, expanded ranching encroached on sedentary
indigenous natives. And, having adapted horses to their
way of life, pastoral indigenous peoples of the Pampas
even threatened European domination for a time.
Imperialism and Settlement in
Europe and Asia
When the Russians conquered Kazan, they gained access
to the rich black soils of the lower Volga River. Thereafter,
Russian colonization expanded farming regions, pushing
nomadic steppelanders out of the way. In the British Isles,
struggles to gain control of farmable lands pitted Scots
against Irish, Celts against Saxons, and Protestants against
Catholics in a merry mlange of bloodletting and cultural
exchange. The same sort of thing took place in Japan as settlers
expanded into arable lands, displacing the indigenous
peoples of Hokkaido.
China was a major player in the game of colonizing new
imperial territories, including the large island of Taiwan.
Overall, the pattern was about increasing access to arable
land that could be exploited for food production. However,
as youll see in your textbook assignment, the case of China
involved a complex interplay of conquests, fleeing migrants,
political maneuvering, and outbreaks of civil war.
Developments in India followed the same trends. Imperial
expansionextending as far as Burmafostered new settle-
ments that increased the range of farming and amplified the
exploitation of natural resources. However, in the Mughal
Empire official deforestation policies were a tool of conquest
that had major ecological repercussions.
New Exploitation in the Americas
The Spanish Empire
Clues to understanding the character of the enormous
Spanish colonial empire are revealed in any of the major
Mesoamerican cities that became pieces of the Spanish
World History
130
dominion. Again and again one finds Spanish cathedrals and
other public buildings superimposed on stone foundations
built by Aztecs, Incas, and other native societies.
Spanish elites replaced Native elites. Spanish technologies,
such as the iron plow, were adopted by Indian peasant farm-
ers. Spanish bureaucrats oversaw local indigenous economies
that continued to produce maize, beans, squash, potatoes,
cocoa, coca, and cotton. Catholic missionaries, priests, and
bishops superimposed Christianity on native populations,
resulting in a weird blend of heathen traditions, plaster
saints, and crucifixes. Meanwhile, in an often heavy-handed
fashion, Spanish overseers harnessed native labor to mine
the many tons of silver and gold that would literally flood
Spain and then the rest of Europe with precious metal
currencies.
Map 17.3 on pages 452453 of your textbook packs a wide
range of information. Study it carefully to better understand
European land exploitation in North and South America. Note
the enormous extent of the Spanish Empire. Also, think about
the opposing French and British territories in North America.
Brazil
The Portuguese presence in Brazil began with sugar cane
plantations and the importation of African slaves. Coastal Brazil
couldnt provide the Europeans an economic foundation like that
offered the Spanish in the Andes and Mexico. Also, Portuguese
expansion to the West was obstructed by the massive rainforests
of the Amazon Basin. Such expansion as did occur was in
response to two factors: Spanish domination in South America
and the promise of mineral wealth. The latter was revealed by
Spanish and the Portuguese explorations of the mighty Amazon
and its countless tributaries. In particular, discovery of gold and
diamonds in the Brazilian province of Minas Gerais in the 1680s
spurred Portuguese expansion westward.
Lesson 4
131
British North America
English colonies from the mid-Atlantic to the subtropical
south were mainly focused on cash crops for export. The
colonies of New England had little to export beyond harvests
from rich stands of timber. However, those hardwood forests
became raw materials for shipwrights, and therein lay the
seeds of colonial prosperity. The New Englanders took to
the seas in sturdy, state-of-the-art sailing ships, building a
maritime dominion that would eventually reach from the
Caribbean all the way to China. This East India trade
brought prosperity to American shores and laid the founda-
tion for importing the fruits of the Industrial Revolution.
Home Fronts in Europe and Asia
Beginning in the late 1500s, worldwide, imperial rulers and
monarchs became systematic about evaluating food production.
Lands were carefully surveyed. The productivity of farm acreages
was tallied. Commodity prices were calculated. In both Japan
and India, peasant farming practices were officially established
to maximize productivity, and uncooperative land owners were
summarily executed.
New energy resources were discovered as environments were
explored and exploited. Whale oil was harvested for lamps
and lotions. Peat from the bogs of Holland was harvested to
be burned as fuel. Timber was systematically harvested for
fuel all across Eurasia. However, especially in the British
Isles and parts of Western Europe, diminishing supplies of
wood resulted in ever increasing reliance on coal. By 1700,
Londoners were hard put to evade the cloying drift of ash and
dust from burning coal.
Land reclamation efforts were directly associated with the
need to protect and expand arable land for farms and village.
In Japan, the systematic draining of wetlands increased
Japanese food production by 82 percent between 1600 and
1720. The most striking example of land reclamation took
place in the Netherlands. Employing ever more efficient
systems of canals and windmills, the Dutch added some
370,000 acres of farmland to their realm between 1610
and 1700.
World History
132
Frontiers of the Hunt
This section on pages 456 and 457 of your textbook high-
lights the impact of European expansion and technology
on resource utilization and agriculture by Native Americans,
specifically the Iroquois.
See the Making Connections feature on page 458 of your
textbook to compare new ways of exploiting the natural
environment in different regions of the globe between 1500
and 1700. Spend some time reflecting on the Chronology
sidebar on page 459.
ASSIGNMENT 18: MENTAL
REVOLUTIONS: RELIGION AND
SCIENCE IN THE SIXTEENTH
AND SEVENTEENTH CENTURIES
Read this assignment. Then read Chapter 18, pages 462487,
in your textbook.
Christianity in Christendom
Emerging from the medieval period, many European back-
waters still clung to pagan ways. Even though Christianity
was the official religion of Europe, its actual presence over
the land was a bit like a thin veneer. Clergy were expected
to repudiate pagan superstitions such as fortune-telling, folk-
healing, and magic. These efforts werent entirely successful.
Many provincial people continued to cling to local folk
wisdom, leaving fertile ground for intermittent outbreaks of
witch-hysteria.
Over the sixteenth and seventeenth centuries, the Catholic
Church was heavily invested in controlling peoples sexual
behavior. The idea that sex = sin became a persistent reli-
gious standard.
Meanwhile, lay people were demanding greater access to
Holy Scriptures and knowledge about the sacraments. Local
priests tended to offer selective bits and pieces of scripture
Lesson 4
133
to laity, partly because so many of them were illiterate.
Martin Luther (14831546) got peoples attention because
he was good at explaining the Christian message to the
masses. He was also pretty blunt about pointing out the
corruption then present in the Catholic Church. As it
turned out, his agenda of reform ended up gaining its
most important support from Germanic princes who had a
vested interest in gaining secular control over the clergy.
Luther triggered the Protestant Reformation, which, in turn,
triggered major house-cleaning in the Catholic Church and a
fierce Counter-Reformation effectively led by the Jesuit Order
founded by Ignatius Loyola (14911556) in 1540.
Christianity beyond Christendom:
The Limits of Success
Wherever European imperialism went, Christian missionaries
followed. In this context, the sheer extent of Spanish dominions
meant that Catholicism had the greatest global impact. In
the Far East, Spanish Catholicism was the religion of the
Philippines. In Japan, Catholic Jesuits and Franciscans
enjoyed a season of missionary zeal. By the 1630s, about
100,000 Japanese had been baptized. However, Japans
rulers viewed Christianity with unqualified suspicion. And,
after a period of persecution, Christianity was banned in
Japan. In China, mandarins and scholars were interested
in the Jesuits scientific learning, but efforts to convert the
Chinese to Christianity fell flat.
The Missionary Worlds of Buddhism
and Islam
As youve already seen, Buddhism was supplanted by
Hinduism and Islam in most of India. However, it took
root in China and Japan as well as in areas of Central and
Southeast Asia. In China, Buddhism coexisted with Taoism
and Confucianism. Under Chinese emperors of the sixteenth
century and later, Buddhism became acceptable as a reli-
World History
134
gion one could practice at home. Something similar took
place in Japan, where Buddhist teachings took on a
Japanese twist and coexisted with Shinto.
The Buddhist influence in Mongol lands of Central Asia
was revitalized in the late sixteenth century at a time when
Mongolia was under the political sway of emperor Altan Khan
(15301583). At that point, Tibet was a Buddhist theocratic
state. The ruler of Tibet, the Dalai Lama, was invited to
Mongolia. It must have been quite a visit, because Mongolia
converted to Buddhism.
As youve seen, Islam was spread by jihad (holy war). Islamic
leaders tolerated infidel Christians and Jews simply to maintain
order within their domains. Meanwhile, Africa was heavily
influenced and impacted by the Islamic faith. Initially, much
of that process was mainly a matter of exposure as opposed
to armed intimidation. Islam accompanied Muslim traders
wherever they went, introducing the Quran and Islamic
life-ways by example and persuasion.
Map 18.1 on page 471 of your textbook shows the spread of
Christianity, Islam, and Buddhism in Asia by 1750. Map 18.2
on page 473 offers a geographic overview of the spread of
Islam in Africa around the year 1700.
The Resulting Mix: Global Religious
DiversityAmerican and Indian
Examples
Just as the Japanese shaped their versions of Buddhist prac-
tice to fit their culture, something similar happened in other
locations. However, the Japanese share a common, uniform
culture. In the Americas and in India, religious diversity has
been shaped by ethnic, racial, religious, and class conflicts,
as well as by historical factors.
The religious attitudes and practices of black Americans have
been, and still are, distinct from the religions of white people.
In Brazil, black Catholics drew on aspects of West African
culture to develop a distinctive subculture. Within that sub-
culture, black Catholic laypeople formed confraternities,
Lesson 4
135
which were characterized by charitable activities and cultlike
devotion to particular saints. In North America, black reli-
gious organizations eventually became hotbeds of political
dissent and activism.
White religions of the British colonies were characterized by
two themes: flight from religious persecution in the Old World
and apocalyptic fervor. The Puritans of New England were
Protestant extremists who felt destined to re-create a godly
city on a hill. Similar fervor characterized other Christian
splinter groups. The Anabaptists, for example, were intensely
dogmatic in their belief that only adults should be baptized.
For Christians, the term apocalypse refers to the end of days
and/or the dramatic redemption of the faithful. Prophecies of
looming apocalypse are said to embrace a millenarian doc-
trine. Millenarian ideas cropped up all over the place in the
sixteenth and seventeenth centuries in both Catholic and
Protestant regions.
India was rife with conflicts between Muslims and Hindus.
That would be the case well into the twentieth century and
beyond. However, in the 1600s, a new religionSikhism
arose that was tailored to accommodate both Muslim and
Hindu customs, practices, and beliefs. The founder of the
religion, Guru Nanak (14591539) proclaimed that the path
to God was neither Hinduism nor Islam, but a creative path
that consulted both sets of beliefs and practices.
The Renaissance Discovery
of the World
Two topics are discussed in this section: the spread of the
Italian Renaissance and European efforts to adapt to the
discovery of alien cultures and peoples in the Americas.
Present-day historians are inclined to see evidence of a
European renaissance (rebirth) modifying Western thought
as early as the thirteenth century, during the time of Thomas
Aquinas. Nevertheless, the traditional notion of a fifteenth-
and sixteenth-century Renaissance birthed in Italy has merit
for two reasons. First, the recovery of the classics of the
Greco-Roman world and the influences of humanism produced
unprecedented advances in art, architecture, literature, and
World History
136
science in Italy. Second, the rest of Western Europe was
heavily impacted by what was flowing out of Milan, Florence,
and Rome.
The second theme of this section encourages you to think
about the mind-blowing impact of what has been called the
Age of Discovery. Beginning with Columbus encounters with
the gentle Carib Indians, the Spaniards were confronted
with what psychologists call cognitive dissonancean inner
struggle with contradictory ideas. Are these people human or
subhuman? (That issue would be settled by a Papal decree
allowing that Native Americans were indeed human beings.)
If theyre subhuman or inferior, how could they have con-
structed temples and monuments that rivaled anything found
in Europe? And then there was the shock of encountering
chimpanzees and gorillas in Africa and orangutans in Southeast
Asia. A nagging question was born. What is it that makes
humans different from these apes?
The Rise of Western Science
In the context of the spreading Renaissance, key players in
the rise of Western science appear all over Western Europe
and the British Isles. And virtually all of them were wander-
ing about in a sort of mental spiritual limbo somewhere
between hidden magical secrets and the ever more sharply
defined universe that could be observed, weighed, and
measured. For example, both Sir Isaac Newton (16421727)
and Gottfried Wilhelm Leibnitz (16461716) invented calcu-
lus. Newtons Principia Mathematica (containing his laws of
motion and of gravity) would become the cornerstone of mod-
ern empirical science. Yet Newton was obsessed with alchemy
and never gave up a compulsive struggle to find empirical
evidence of Gods existence. Leibnitz was obsessed with
Kabalistic and Egyptian mysticism.
Following on the earlier discoveries of Polish astronomer
Nicolaus Copernicus (14731543), Johannes Kepler (16461716)
confirmed a heliocentric (sun-centered) understanding of
the solar system. With Earth demoted from center stage,
the impact of the work of Newton and Leibnitz would gradually
be distilled into the rationalist methodology of modern empir-
ical science. Key players in this process included Rene
Lesson 4
137
Descartes (15961650) and Francis Bacon (15611626).
Descartes declared an unbridgeable gap between mind and
matter, thus establishing an uncertainty principle into
deliberations on epistemology. (Epistemology is the study
of how we can know what we know.) Bacon established the
rules of scientific inductive reasoning, whereby scientists
gather multiple observations, hypothesize connections among
what they observed, and test their hypothesis by way of
empirical experiments. In time, whatever couldnt be weighed
and measured was no longer of any interest to science, thus
opening a divide between science and religion.
Western Science in the East
Prior to the ascendancy of Western science, China was the
global center of practical technological innovations. Blast fur-
naces, advanced metallurgy, gunpowder, papermaking, and
advanced shipbuilding technologies, such as the rudder, were
invented in China.
East Asians tended to view the West as beastly and provin-
cial. Initially, Western scientific thought found few places to
nest in the Orient. Eventually, however, Western science
would make its way to the East. In the case of China, the
main harbingers of new ideas, especially about the age of
astronomy, would be Jesuit missionaries.
Use the chapters concluding Chronology sidebar on
page 485 for a time line of the many events over this period.
ASSIGNMENT 19: STATES AND
SOCIETIES: POLITICAL AND
SOCIAL CHANGE IN THE
SIXTEENTH AND SEVENTEENTH
CENTURIES
Read this assignment. Then read Chapter 19, pages 488513,
in your textbook.
World History
138
Political Change in Europe
The European state system of the sixteenth and seventeenth
centuries halted dreams of unification. The possibility of
creating a new European Roman Empire ranged from zero
to nil. Your textbook offers three reasons for this situation.
First, European states were asserting their political independ-
ence at the same time that they were increasing their control
over citizens. As a last desperate effort at unification, King
Charles V was invited to rule a mainly Germanic empire to
resurrect the earlier Holy Roman Empire. That idea faded
after Charles V left the scene. (In any case, as historians like
to quip, the Holy Roman Empire was neither holy, nor Roman,
nor empire.)
Second, monarchs continued to gain and impose power over
rival political claimants and ordinary citizens. The Treaty of
Westphalia (1648) gave state rulers the right to impose a
state religion on their subjects. In this context, cities and
churches were forced to surrender their rights to self-
government.
Third, for various reasons, no European state was in a posi-
tion to pursue and agenda of imperial unification. Even
Spain, for all its wealth, couldnt suppress rebellion in the
Netherlands or gain a real toehold after invading France.
Map 19.1 on page 491 of your textbook will help you visualize
the scattered extent of the Hapsburg dominions of Charles V.
Map 19.2 on page 492 offers a global view, including a handy
time line, for grasping the extent of the Spanish Empire.
Western Political Thought
The concept of state sovereignty was formulated by the
French political philosopher Jean Bodin in 1576. His idea
was that a sovereign state had the sole right to make laws
and afford justice to its subjects.
Following on that idea were the radical notions of an Italian
office-seeker by the name of Niccol Machiavelli. His hand-
book for rulers, The Prince, was published in 1513. It gained
influence. Some would say it was a dreadful influence.
Machiavelli proposed that the sole considerations of a prince
Lesson 4
139
were serving his (or her) self-interests and holding onto
power. The Prince introduced rulers to the concept of
realpolitik; namely, that the state is intrinsically amoral.
Anything the state does in pursuit of self-interest is justified.
The notion of laws-of-nations was first identified in the
writings of Thomas Aquinas in the thirteenth century. They
were crystallized under the pen of the Dutch jurist Hugo
Grotius in 1625. In his view, nations were required under
natural law to respect the sovereignty of other nations.
Failure to abide by international treaties was just cause for
war.
Western Society
In the West, the modern social class system featuring an elite
upper class, a middle class, and a working class arose with the
Industrial Revolution. In sixteenth- and seventeenth-century
Europe, vague social class distinctions were superimposed on
those based on occupation, clerical status, and aristocratic
status. Tradesmen were organized under a guild system
requiring apprenticeships before one could ascend to the title
of journeyman or guild master. Merchants and bankers had
their own guilds. Priests, bishops, arch-bishops, and so on
identified with the church hierarchythe ecclesiastic estate.
Nobility (the blue-blood estate) was recognized as a social
status that lumped wealthy aristocrats with impoverished,
but titled, landlords.
As social change reshaped European society, the status
of the family changed. In the High Middle Ages, a family
dwelling amounted to a large common room sufficient to
shelter kin, retainers, cousins, and maybe a pet goat or a
baby pig. Private bedrooms? Forget about it. Gradually, fami-
lies began to have living rooms apart from the commons hall
and private chambers appeared. Gradually, the concept of
family began to refer to spouses and their offspring residing
in a separate dwelling.
World History
140
An east-west gap appeared in Europe. To the west, peasants
could manage their own labor and even negotiate commodity
prices. To the east, landlords tended to treat farm labor as
property attached to the land. In the west there were
peasants; in the east, serfs.
Womans status changed, sort of. Women became monarchs
in unprecedented numbers, despite voices of outrage here
and there. Queen Elizabeth I became an icon of British sover-
eignty and power. However, women were routinely beaten by
their husbands and deprived of justice by the courts.
See Figure 19.1 on page 493 of your textbook, which compares
social identities and concepts of social class in modern soci-
eties versus those of the sixteenth and seventeenth centuries.
Read the Making Connections feature on page 496 and think
about the factors that contributed to European state-building.
The Ottomans
The sparkling center of Ottoman power was the fortified
and sanctified Topkapi Palace in Constantinople, where the
ruling sultan autocratically dispensed laws and decrees at
will. The inner circles of power, often guided by concubines
and eunuchs, schemed and plotted to place their favorites
on the throne, because no adequate system for appointing
heirs was in place. Indeed, it wasnt uncommon for the ruling
sultan to murder or imprison his brothers to neutralize their
imperial aspirations.
You should already be familiar with some of what youll
encounter here. Recall that the Ottoman Empire was as
efficiently run and as technologically up-to-date as were
European powers of that period. Also, remember the elite
Janissaries who were recruited from Christian families
while still young boys.
The sultans grip on central control was limited. Provincial
governors were often corrupt, and rivalries between provinces
could be violent and destructive. Onerous tax burdens forced
military deserters and unhappy peasants into banditry.
Lesson 4
141
During and after the seventeenth century, the Ottoman
Empire ceased its expansion into European territories. This
didnt result from a decline in government efficiency so much
as it did from a shortage of people. As populations in China,
India, Japan, and Europe were increasing, Ottoman popula-
tions were shrinking.
Mughal India and Safavid Persia
Mughal India and Safavid Persia were basically tribute
empires. The Mughal justified their sovereignty by seeing
themselves as heirs of the Timur Empire and, ultimately,
Genghis Khan. The Safavid fiercely embraced the Shiite
branch of Islam to justify their rule, even thought the Safavid
aristocracy was quite lax in observing Islamic ideas about
drink and sex. The Safavids benefitted from the flow of trade
across Eurasia. The Mughal Empire retained power through
policies of continuous war and expansion.
Map 19.3 on page 500 of your textbook shows the Safavid
Empire sandwiched between the Ottoman and Mughal
Empires between 1501 and 1736. Spend some time with the
Chronology sidebar on page 501 for an overview of key
dates and events in the three Islamic empires: Ottoman,
Safavid Persian, and Mughal Indian.
China
In China, the emperors ostensible unlimited authority was
channeled through a bureaucracy composed of thousands
of scholarly mandarins who embraced Confucian doctrines.
They basically ran the administration of the empire. In that
context, mandarins extolled the emperors role in rituals and
ceremonies while insisting on policies of peace and stability.
Foreign adventuring was frowned upon, and mandarin con-
trol was such that the emperor was forbidden from leaving
the capital.
A grave crisis arose in 1587 when the emperor Wanli
(r. 15721620) decided to defy the mandarins. His ten-year
struggle ended in the mandarins victory. Thereafter, the
Ming dynasty was a paper tiger, unable to cope with ecolog-
World History
142
ical disasters, peasant revolts, and the dissolution of the land
into civil war. What followed was the rise to power of united
Manchu forces and the founding of the Qing dynasty in 1644.
Chinese society had no traditions of people power as repre-
sented in republics or democracies. Thus, peasant rebellions
mainly rendered China into a patchwork of peasant landholders.
Qing strategy in this situation favored Manchu aspirants to rank
and power in the military or within the imperial administration.
At the same time, the Qing rulers were diligent in appeasing
the mandarins and declaring the importance of Confucian
traditions.
Tokugawa Japan
With the doubling of food production and an expansion of
legal and illegal trade with China and Korea, Japan experi-
enced a period of peace and prosperity under the Tokugawa
shoguns residing in Edo (the original name for Tokyo). The
big winners were peasants and merchants offering food and
merchandise to an expanding internal market. The losers
were the traditional warrior class, the samurai. Many of these
out-of-work warriors became underpaid roninfreelance
mercenaries employed by petty landlords called daimyo.
Map 19.4 on page 504 of your textbook shows the daimyo
boundaries and the Tokugawa domains after 1614. The
Making Connections feature on page 505 summarizes
factors of stability and change in China and Japan.
The New World of the Americas
Colonies in the New World featured a mixture of transplanted
Old World norms and values with new ways of life required
for adapting to new frontiers. In Spanish America, Europeans
were far outnumbered by indigenous peoples. Not surprisingly,
Spaniards intermarried with Indians to create Creole conscious-
ness. To be Creole was to be not quite Old World and not quite
New World. Creole identity was pride in being something new.
Lesson 4
143
In Spanish Mexico, young Aztec boys, called ladinos,
attended mission schools to learn Spanish and Spanish
ways. Having gained the trust of the Spaniards, ladinos could
find a place in the new social order. Intermarriage between
Europeans and Mexicans produced generations of mestizos
who would be counted as citizens of what would one day be
called the Nation of Bronze.
Elsewhere in the Americas, the unprecedented extent of
slavery was another frontier factor. Especially in Brazil and,
a bit later, in North America, intermarriage would produce a
spectrum of racially mixed people. However, in Brazil, there
was a tolerance of mixed-heritage people that wouldnt be
found in the southern areas that would be part of the United
States.
An interesting development in the Americas was a proliferation
of runaway-slave communities. Calling themselves maroons,
they transplanted elements of West African culture to various
parts of Americas.
Map 19.5 on page 508 of your textbook will help you locate
maroon communities in the Americas. Notice the locations of
the Esmeraldas kingdom in Colombia and the backcountry
maroon state founded in Dutch Surinam.
Africa
This section touches on the social and political impact of the
slave trade in Africa. Several main points stand out. The
great majority of slaves delivered into the hands of European
slavers were rounded up and captured by black Africans. In
that context, some African tribes or states were actively
engaged in slaving for economic and political reasons. Among
these were Akwamu and Kongo.
Over 1.5 million black slaves had reached the Americas by
1700. Six million more arrived over the eighteenth century.
Indeed, some populations of the West African coast were
almost entirely depleted. At least 400,000 captured slaves
died (or were murdered in some cases) during the infamous
Middle Passage. They never reached the Americas.
World History
144
Spend some time studying Figure 19.2 and the date-event box
on page 509 of your textbook. Note the intense escalation of
slaving activity over the eighteenth century. Map 19.6 on
page 510 depicts West and West Central Africa around 1750.
Use it to locate the African states described in the textbook
and the Portuguese possessions and Dutch settlements related
to the slave trade.
Please complete Self-Check 7 now.
Self-Check 7
Indicate whether each of the following statements is True or False.
______ 1. In the context of ecological exchange, its an odd fact that the sweet potato had no
impact on China.
______ 2. The Dutch East India Company had a government-granted monopoly on trade between
Holland and Asia. However, the Chinese viewed the Dutch role in Batavia as a
subordinate one.
______ 3. In the context of the rise of Western science, epistemology is the doctrine that reality
is observable and verifiable through the senses.
______ 4. Efforts by the Roman Church to stamp out popular religious beliefs and superstitions
were aimed at weaning people away from a religion of worldly survival to one of
salvation in the afterlife.
______ 5. To a large extent, the Portuguese were accepted in Asia because their trade activities
and their work as shippers were good for local economies.
(Continued)
Lesson 4
145
Self-Check 7
______ 6. Peaceful conditions in Tokugawa Japan encouraged unemployed samurai to become
freelance mercenaries called ronin.
______ 7. Sixteenth-century demographic collapse in the New World was mainly the result of
armed conquest and only secondarily an effect of microbial exchange.
______ 8. In China, under emperors such as Zhu Hong and Han Shan, Buddhism was presented
as a religion that could be practiced at home without requiring the mediation of priests.
Fill in the blank with the correct term.
9. In British North America, _______, introduced to Virginia in 1614, was the first crop that
helped sustain the colonial economy.
10. Residents of communities and mini-states created by runaway slaves in the Americas were
referred to as _______.
11. In 1662, leading merchants in the Dutch city of _______ formed a joint stock company aimed
at monopolizing trade with Asia.
12. The most important crop in transoceanic trade between the Americas and Europe was
_______.
13. The _______ theologian Martin Luther was more successful in communicating his ideas about
Christianity to secular rulers than to fellow priests.
14. As derived from the writings of Machiavelli, the concept of ________ maintained that the
state wasnt subject to moral laws and that state policies should be dictated solely by self-
interest.
15. Within the arena of Indian Ocean trade, some local and regional exchanges never reached
Europe. These were referred to as _______ trades.
16. The religion called Sikhism blended _______ and Muslim traditions.
(Continued)
World History
146
Self-Check 7
Answer each question in not more than four complete sentences.
17. What is the stranger effect, and what did that have to do with the Spanish conquest of the
Aztecs?
__________________________________________________________
__________________________________________________________
__________________________________________________________
__________________________________________________________
18. Considering both Africa and the Americas, what is meant by the expression pastoral
imperialism?
__________________________________________________________
__________________________________________________________
__________________________________________________________
__________________________________________________________
19. What was Creole consciousness?
__________________________________________________________
__________________________________________________________
__________________________________________________________
__________________________________________________________
Check your answers with those on page 220.
Lesson 4
147
GLOBAL ENLIGHTENMENTS
The second part of Lesson 4 covers Chapter 20, Driven by
Growth: The Global Economy in the Eighteenth Century;
Chapter 21, The Age of Global Interaction: Expansion and
Intersection of Eighteenth-Century Empires; and Chapter 22,
The Exchange of Enlightenments: Eighteenth Century
Thought.
ASSIGNMENT 20: DRIVEN BY
GROWTH: THE GLOBAL
ECONOMY IN THE EIGHTEENTH
CENTURY
Read this assignment. Then read the Part 8 opening spread
and Chapter 20, pages 514537, in your textbook.
Population Trends
Population growth took place as people were dispersed onto
underexploited marginal lands or were concentrated in
urbanized areas. Putting aside considerations of overall
general population increase across the globe, towns, villages,
and cities sprouted in increasing numbers. China, India,
and Japan had the greatest numbers of new urban centers.
Among these, the proportion of people living in cities was
greatest in Japan.
In Western Europe, urban growth was concentrated in
several large cities. Eighteenth-century London had a
population of around one million. Paris had a population
of about 500,000, as did the port city of Naples in southern
Italy. Moscow, Vienna, Amsterdam, and St. Petersburg had
populations of around 200,000.
This section explores the possible causes of the eighteenth-
century population growth spurt. Historians have offered a
variety of explanations. Some believe that nutrition improved
as a result of global ecological exchange. However, nutrition
levels actually declined among the poor, many of whom
World History
148
hovered around the large urban centers. Improved hygiene
seems to have played a part, but sanitation in large cities of
this period was anything but ideal. In London, ditches filled
with raw sewage flowed into the Thames. Typhus outbreaks
and acute diarrhea were common in all large cities.
Improvements in scientific understandings of disease and
cures had a couple of significant impacts. Scurvy, long the
scourge of sailors, was basically eradicated once it was
demonstrated that sources of vitamin C (lemons, oranges)
offered protection. More important, knowledge of inoculations
against smallpox, which had already been developed in the
Far East, finally spread to Europe. By 1796, Edward Jenner
discovered that cowpox inoculations conferred immunity
against the disease, with few, if any, side effects.
However, for the most part, Western medicine remained glued
to outdated ideas that were either useless or actually harmful.
In the final analysis, it isnt clear why plague burned itself
out, or why, in general, people seem to have developed partial
or complete immunities to a variety of other diseases, such
as measles and chicken pox.
Map 20.1 on pages 520521of your textbook shows the com-
parative growth of populations in different regions across the
globe. Youll see that the greatest population gains are in
Europe and Asia. The inset graph, (The World) shows the
accelerating curve of human population growth from
10,000 B.C.E. to 1800 C.E.
Economic Trends: China, India,
and the Ottoman Empire
Economic vitality is tightly associated with increasing popula-
tions. Therefore, its not surprising that Chinas immense
population provided a huge internal market. For a time, Qing
China was top dog when it came to the manufacture of silk
fabrics, porcelain, tea, and paper. The West clamored for
access to Chinas trade goods, but China was more or less
uninterested. It boasted a full-employment economy and little
motivation to deal with Western barbarians.
Lesson 4
149
When the British East India Company imposed a monopoly
on Indias poppy production, Westerners finally had a com-
modity that generated high demand in China. Chinas
favorable balance of trade could start to be reversed. Of
course, Western expansion of alternative sources of desired
trade goods in India and Southeast Asia had its effects, but
the clincher was a Western productivity leap based on
mechanization. China was caught in what has been called a
high-level equilibrium trap. China had an economy based on
surplus labor meeting high demand with traditional technol-
ogy. Therefore, Chinese manufacturers had no motivation for
adopting mechanization.
Map 20.2 on page 525 of your textbook shows the extent of the
Qing Empire as of 1770.
The situation in India was similar to that in China in many
respects. The Indian economy was vibrant and richly produc-
tive. However, while China would remain a power to be
reckoned with after its global economic clout declined, the
Indian economy would simply collapse. India would end up
as a de facto tribute state of the British Empire. At least two
factors contributed to this turn of events. The decline of the
Mughal Empire accelerated due to Persian invasions, internal
conflicts, and rebellion. However, the main factor seems to
have been the clever manipulation of markets and commodi-
ties by agents of the British East India Company.
As for the Ottomans, their story seems to have amounted to a
lack of mechanical inventiveness and an inability to compete
with dynamic competition from Britain, France, and Italy.
Use the inset table, Global Population and Economic Trends,
on page 528 to review what youll have learned to this point in
the chapter.
The Wests Productivity Leap
The rise of industrialism was influenced by science. Youll see
a long list of the names of scientists and their innovations on
page 529 of your textbook. All of these people tended to think of
themselves as natural philosophers. And all of them represented
the spirit of an age of curiosity and discovery. Although science
didnt cause the Industrial Revolution, it certainly paralleled and
encouraged it.
World History
150
Most of this section focuses on Great Britainwith good
reason. The heart and center of the rise of the industrial
age was in England and Scotland. Youll want to think about
why that was the case. Several ideas are offered on pages
529532 of your textbook, although none of them are neces-
sarily a full explanation.
Map 20.3 on page 531 of your textbook will help you better
understand what was going on in Britain as the Industrial
Revolution gained steam. Keep on mind that industrialization
was an outgrowth of a general effort, expressed around the
globe, to maximize the uses of land and other natural
resources.
The Expansion of Resources
This section on pages 532536 of your textbook is rich with
fascinating detail, particularly with respect to developments
in the Pacific. Take time to enjoy the tour. Before you do, pre-
pare to digest a few central ideas, particularly the concepts of
global gardening and New Europes.
Global gardening was a major aspect of ecological exchange
in the eighteenth century. Immense, scientifically managed
gardens, such as the Royal Botanical Kew Gardens in London,
became plant-life research laboratories. Spin-offs of botani-
cal research in these places were complemented by field
experiments, such as those conducted by Pierre Poivre in
Frances Indian Ocean colonies.
Europeans wanted to establish colonies in parts of the world
where climatic and environmental conditions would permit
the transplantation of European-style economies and life-
ways. Two prominent examples of such New Europes in
the Pacific are New Zealand and Australia. Both of these, of
course, would become part of the British Commonwealth.
Use the inset table, Economic Revolution in the West, on
page 535 to review key events in the rise of the rise of global
Western influence. Then study the Chronology sidebar
on page 536 for an overview of key events between 1500
and 1800.
Lesson 4
151
ASSIGNMENT 21: THE AGE OF
GLOBAL INTERACTION:
EXPANSION AND INTERSECTION
OF EIGHTEENTH-CENTURY
EMPIRES
Read this assignment. Then read Chapter 21, pages 538561,
of your textbook.
Asian Imperialism in Arrest or
Decline: China, Persia, and the
Ottomans
China under the Mongol Qing dynasty was the fastest-
growing empire in East Asia. The Qing emperors over-
whelmed Tibet and conquered chunks of Mongolia, Russia,
Burma, and Vietnam.
In this same period of the eighteenth century, by way of
massive Chinese migrations to the south, Chinese traders,
merchants, and peasants became significant parts of the
many populations in Southeast Asia, including the Spanish
Philippines. However, by the last years of the reign of the
Chinese Emperor Qianlong (r. 17351796) Chinese imperial
expansion ground to a halt. The empire had reached the
limits of its resources and technologies.
Both the Persian and Ottoman empires were disadvantaged
by population decline relative to other regions of Eurasia. In
this context, the Safavid Persian Empire became a play-
ground for invading Afghan warlords and fell apart by 1722.
The Ottoman Empire retained some degree of control over its
heartlands in Turkey as its empire shrank. With the rise of
Wahhabism, Arabian states seceded, and the Ottomans lost
control of that region. Meanwhile, by 1774, the Ottomans
had ceded their holdings along the Black Sea to Russia, even
as the vitality of Ottoman trade was being drained away by
European competition. And, following Napoleons brief
World History
152
invasion of Egypt and Syria in 17981799, North African
states from Egypt to Algeria to Morocco essentially became
autonomous.
Map 21.1 on page 544 of your offers a geographic view of the
expansion of Wahhabi domination in the Arabian Peninsula.
Imperial Reversal in India: Mughal
Eclipse and British Rise to Power
Starting around 17191720, the Mughal emperor began
making concessions to petty Hindu princes called Marathas.
Thereafter, Mughal control was reduced to a nominal or
titular role, as one Muslim ruler declared himself ruler of an
independent central Indian state of Hyderabad and Hindu
Marathas princedoms more or less ignored imperial influ-
ences, even as they stopped short of a full-blown effort to
overthrow the ghostly remains of the Mughal Empire.
See Map 21.2 on page 545 of your textbook for a geographic
view of the Marathas impact.
The British East India Company was founded in 1600, at
about the same time as the Dutch East India Company.
However, although the Dutch werent averse to using force as
needed to control their colonial domains, the British policy
was ostensibly only about tradenot war. As late as 1750,
major British coastal colonies acted as administrative centers
of various presidencies at the ports of Bombay (Mumbai)
and Madras (now Chennai). However, armed French expedi-
tions into India changed the rules of the game. Robert Clive,
a clerk with the Presidency of Madras, led a punitive
expedition into Bengal in 1756. His aim was to neutralize a
French military incursion. Although heavily outnumbered,
Clive achieved a British victory, and his punitive expedition
became a bit like Pizarros conquest of Peru. The French were
bloodied, and Bengali resistance collapsed like a house of
cards. Abruptly, the richest province of the erstwhile Mughal
Empire fell into British hands.
Bengals wealth was sufficient to fund further military expedi-
tions, and the rest, as they say, is history. By the outset of
the nineteenth century, India was a major piece of the British
Empire.
Lesson 4
153
The Dutch East Indies
The Dutch, unlike the Spanish or the British, were short on
two kinds of resources. They had little or no control over
regions that produced silver and gold; that is, they lacked
hard cash. Also, they were short on manpower. In short, the
Dutch were in no position to launch campaigns of territorial
conquest. In the end, the Dutch used force and manipulation
of local princes and populations to monopolize the spice
trade. However, until the Dutch got serious about developing
coffee plantations on Java, their colonial trade ventures were
more loss than profit.
Examine Map 21.3 on page 546 of your textbook. It will help
you understand the impact of European colonial expansion in
South and Southeast Asia around the year 1800.
Africa, the Americas, and
the Slave Trade
At this point, youre already familiar with the impact of the
slavery on the global economy. In this section, aspects of the
slave trade and its impacts on European and African states
are fleshed out with a bit more detail.
Although simple principles of economics maintain that forced
labor is inefficient and not normally profitable, slavery in the
Americas may have been economically viable for a couple of
reasons.
In Spanish and French America, slaves were granted the
right of marriage and assurances that spouses wouldnt be
separated. Within the constraints of plantation life, even in
North America black slaves were able to sustain aspects of
their native cultures and maintain their own social worlds.
However, given the dreadful conditions of slavery, infant mor-
tality plus low birth rates tended to keep slave populations
smaller than they might otherwise have been.
The slave trade was very profitable for Europeans. Therefore,
with the exception of limited coastal incursions, mainly by
the Portuguese, Europeans werent interested in contending
with the natural obstacles of invading the African interior.
World History
154
African geography plus malaria kept Europeans out of most
of Africa before the virtues of quinine were discovered in the
nineteenth century. A notable exception was in South Africa.
There, Dutch settlers (the Boers) pushed the native Xhosa
northward in a series of wars that ended in 1795. Thereafter,
the Boers entered a period of internal conflict as the British
seized Cape Town from the Dutch East India Company.
In West Africa, an Islamic extremist, Usuman da Fodio,
claiming to be the forerunner of the Mahdi (the Muslim
messiah), founded and expanded a pastoral Fulani Empire
with its capital at Sokoto. It would last until the British
conquered it in 1906.
Study Map 21.4 on pages 550551 of your textbook. Its vital
to your understanding of the global impact of the world slave
trade around the year 1800. Map 21.5 on page 553 offers a
detailed look at the extent of the Sokoto Fulani Kingdom
around 1820.
Land Empires of the New World
The Araucanos and the Sioux
On the prairies of the Argentine Pampas, pastoralist natives
were able to draw on the military and organizational expertise
of the Araucano chiefdom to enter into trade with Chile. In
time, having developed governance under hereditary war
chiefs, they became powerful enough to threaten Buenos
Aires. Something roughly similar happened across the
American Prairies. The Sioux (Lakota), who had been gradu-
ally pushed west by European expansion, developed a unique
culture that lasted until around the 1870s, when the last of
the Plains tribes were pacified and confined to reservations.
Lakota plains culture was based on buffaloes, horses, and
the conquest of rival tribes.
Portugal in Brazil
As Portugal surrendered much of its earlier maritime colonial
empire elsewhere, it began to develop a rich and complex
society in Brazil based, in part, on the gold and diamonds of
Lesson 4
155
Minas Gerais. The Portuguese were able to push the Spanish
westward across the Amazon Basin to roughly the language-
division boundaries that exist today.
Spanish America
Spanish America expanded mainly by negotiating with Native
Americans, inviting them under the umbrella of the Spanish
monarchy. That this strategy was effective is marked by the
extent of Catholic missions from California through Central
America and Andean South America all the way to the South
American cone formed by Chile, Paraguay, Uruguay, and
most of the region that would become Argentina.
Your textbook discussion of Creole mentalities refers to the
fact that Latin American cultures began to adopt their own
regional identities. Colombians and Peruvians and others
began to see themselves as citizens of distinct New World
nations. At the same time, Creole consciousness asserted
that Latin American cultures and sciences were actually
superior to those of the Old World. A similar shift in cultural
and regional identities would transform the British colonies
of North America. The American colonials who rose up
against England were citizens of a new kind of society
mainly young, impulsive, and primed to affirm the vexations
and grievances that had driven them to the New World in the
first place.
Toward Independence
Were now in historical territory with which youre likely to
already have some knowledge, at least with respect to North
America. With respect to the causes of the American
Revolution of 17751783, you might want to rethink what
was called the French and Indian War that concluded in
1763. What was regional in North America was actually the
culmination of the long, exhausting worldwide Seven Years
War between France and England (17561763). The British
victory effectively evicted the French from North America
(with the notable exception of Quebec). At that point, a
drained British treasury caused Parliament to seek ways
World History
156
to better control and more heavily tax the American colonies.
As you probably know, the immediate cause of rebellion in
the Atlantic colonies was onerous taxation.
Taking a global view of the period, think of it as an era of
revolutions. The French Revolution of 1789 followed the
American Revolution and was related to it in terms of
ideologies and citizen aspirations. The same could be said,
paradoxically enough, of the successful revolution of black
Haitians against the French in 17911802. In Latin America,
Creole consciousness was giving rise to rebellions throughout
the 1770s and 1780s. Spanish reforms suggested through
the superb scientific work of Alessandro Malaspina were in
the end ignored and repudiated. Instead, in response to the
French Revolution, the Spanish monarchy did exactly what
was needed to assure the successful rise of South American
liberators like Simon de Bolivar in Grand Colombia.
Study Map 21.6 on page 559 of your textbook to see the wide
areas of the Americas that declared and gained independence
from European powers as of 1828. Conclude your study of
this assignment with a careful inspection and review of the
Chronology sidebar on page 560.
ASSIGNMENT 22: THE EXCHANGE
OF ENLIGHTENMENTS:
EIGHTEENTH-CENTURY THOUGHT
Read this assignment. Then read Chapter 22, pages 562585,
in your textbook.
The Character of the Enlightenment
In a sense, the character of the Enlightenment was a reaction
to the sterile clockwork universe implied by Isaac Newtons
genius. As people confronted nature up close and personal or
beheld the stars and planets in their courses, they couldnt
but sense that not everything of importance could be weighed
and measured. In effect, the Enlightenment was about fitting
human experience into the mysteries of existence.
Lesson 4
157
The Enlightenment in Global Context
This section focuses on how Westerners viewed and were
influenced by other global civilizations.
China
In the mid-eighteenth century, China wasnt only the fastest-
growing empire in the world, it was the most modern and the
best-educated society on the planet. The influential French
thinker and sharp-witted social critic Voltaire expressed
glowing admiration for China. Following a pattern that
would be taken up elsewhere, he was engrossed by the
wisdom of Confucius. Of course, that was partly because
Voltaire (16941778) despised Western organized religion.
Echoing Voltaires fascination with the Orient, Western elites,
literati, and prominent artists were entranced by Chinese
culture. It was all the rage to design interiors in Chinese
style as it became possible to import Chinese wallpaper and
furniture.
By contrast, the Baron de Montesquieu (16891755) viewed
China as a cruel autocracy that ruled by fear and offended
the inner spirit of human nature. Montesquieu authored The
Spirit of Laws, a work that would have an immense impact in
the framers of the United States Constitution. He maintained
that government should serve to ensure the maximum free-
dom of the governed.
Japan
Japan was mainly a mystery to the West at this point. The
Jesuits had nothing good to say about Japan, having been
evicted from Japan along with Christianity. The Dutch trade
relationship with Japan served as the Wests only window on
Japanese society. In that context, an employee of the Dutch
East India Company characterized Japanese society as
orderly, peaceful, and prosperous even as its people were
bound in submission and slavery.
World History
158
India
Voltaire saw India as a progenitor of Western culture, the
land of the first philosophers, theologians and lawgivers. In
1786, in England Sir William Jones recognized that Sanskrit
was related to both Greek and Latin. Joness work led to the
discovery that most European languages were branches of a
large family of Indo-European languages.
The Islamic World
The influence of the Islamic world on Western art and ideas
was mixed. Earlier views of Turkey and the Ottomans reflected
admiration. Playwrights adopted Turkish settings for comedic
or satirical dramas. Persian sages engaged the Western
imagination. Informed travelers expressed positive views of
Ottoman religious tolerance. Later, Western pundits voiced
critical views of Ottoman autocracy, administrative corrup-
tion, and slavish public submission to the whims of sultans.
The Enlightenments Effects in Asia
The Enlightenment and China
The Chinese continued to see China as the center of the
world. China had long outpaced the West in social and tech-
nological development. However, especially through the
influence of visiting Jesuits, Chinas emperors were ready to
absorb Western advances in mathematics, geometry, and
astronomy. Above all, they were keen on adopting Western
military technology. Jesuit-designed artillery was used in
China until the mid-nineteenth century.
Western Science in Japan
Japan was a bit more open to Western ideas than was China.
Translations of Western books were allowed to circulate
through the empire. Western advances in studies of human
anatomy were recognized and applied. However, the most
important sources of new developments in Japan were internal
reactions to Confucianism. In particular, we find a case of
Lesson 4
159
parallel invention. Independently, Japanese thinkers began
to reach the same conclusions as Western natural philoso-
phers as to the role of reason and empirical observation in
making sense of the world. Somewhat similar reactions to
Confucianism occurred in Korea and Vietnam.
The Ottomans
The Islamic world was slow to adopt Western developments
in science and philosophy. India and the regions around
the Ottoman domain waxed enthusiastic about Western ideas
only in the nineteenth century, mainly as a reaction to the
obvious superiorities of Western military technology.
The Making Connections feature on page 571 of your text-
book summarizes Enlightenment influences in Asia.
The Enlightenment in Europe
In France, Denis Diderot (17131784) masterminded the
creation of the 17 volumes of text and 11 volumes of illustration
entitled Encyclopedia: A Reasoned Dictionary of the Sciences,
Arts, and Trades. By 1779, 25,000 copies of this hyper-tome had
circulated throughout Europe. Its impact, including its
critical views of monarchies and aristocracies, was enormous.
Under this influence, the writings of the Englishman John Locke
(16321704) got a lot of attention. Locke would give the West
seminal ideas about inalienable natural rights, human
equality, and constitutional guarantees of life, liberty, and the
pursuit of property. (The American founding fathers, following
Thomas Jeffersons lead, substituted happiness for property.)
In this section, youll be introduced to four fundamental
features of Enlightenment thought: belief in progress; new
economic thought; ideas of social equality, particularly of
women; and anticlericalism. These topics are discussed
in-depth on pages 572575 of your textbook.
Enlightenment thinkers embraced a belief in progress. The
long contest between good and evil in human affairs could be
viewed as a very long road from tyranny and ignorance to a
new social order designed to undo the damage summed up in
World History
160
a (paraphrased) quote from the French philosopher and
social critic Jean Jacques Rousseau (17121778): Man is
born free yet everywhere we find him in chains.
New economic thought paralleled economic realities emerging
to create the foundations of the rise of capitalism and indus-
trialism. French philosophers coined the doctrine of laissez
faireleave the market alone. The British economist David
Ricardo proposed that wages naturally rise or fall as a func-
tion of supply and demand and should never be regulated by
government legislation. However, the blockbuster work in
political economy arrived from a Scot moral philosopher by
the name of Adam Smith. Smith published the Wealth of
Nations in 1776. Indeed, his notions about self-regulating
markets and the natural human proclivity to trade, truck,
and barter would inform the content of the Declaration of
Independence.
Social equality was a theme in Lockes writings. He pro-
claimed an idea that would appear in the Declaration of
Independence: We hold these truths to be self-evident . . .
that all men are created equal . . . . Given that more than
half of men are in fact female, feminist writings were a
predictable outcome of the social-equality concept.
Anticlericalism reflected both a political and a philosophical
rebellion. The corruption and power of the church invited
political revulsion. The rise of science and faith in reason
challenged the dogmatic perspectives of organized religion
Protestant or Catholic.
The Crisis of the Enlightenment
The idea that nothing of transcendent importance could be
detected or understood beyond the boundaries of hard
science is called scientism. Science as a way of understanding
nature becomes an all-encompassing worldview. What cant
be weighed and measured, which includes the ultimate
nature of consciousness itself, is beyond what we can know.
Lesson 4
161
Humans can employ thinking and reason to know things.
But in actual experience, feelings and intuitions (hunches)
also matter. People want to feel that the universe is friendly.
They also feel the need for a moral compass, some way to tell
the difference between good and evil. Religion either can or
should serve these purposes. By contrast, unadulterated
scientism has little to say about morality and nothing at all
to say about spirituality. Thinking about these ideas will help
you understand the crisis of the Enlightenment.
Religious revivals, in both Catholic and Protestant domains,
jumped over the heads of the intellectual elites to appeal to
the masses of less educated people. In this context, religious
appeals tended to be anti-intellectual, sometimes starkly
irrational as in the case of millennial movements.
The cult of nature and romanticism served as an escape from
cold scientism and produced alternative perspectives on both
human nature and spirituality. As artists and poets turned to
nature to seek meaningful experience, nature itself became a
sort of open-air spiritual sanctuary. In the case of the greatest
scientist of the age, Alexander von Humboldt (17691859),
explorations of the Americas and of ocean currents were
informed by a romantic view of nature.
Rousseau extolled the superiority of natural passions over
cultivated refinement. Rousseaus notion of general will pro-
posed that society is a sort of organism. Individual identities
should be subsumed within this collective consciousness.
Therefore, maximum freedom within a state can be attained
only to the extent that all conform to the general willthe
will of the majority. The German philosopher Immanuel Kant
(17241804) responded to this self-contradictory logic with a
lengthy treatise, arguing that universal principles of morality
can be established through reason.
Especially as a result of the exploratory voyages of Captain
James Cook all over the Pacific, Europeans were exposed to
isolated and alien cultures. That people like the Maori of New
Zealand or the Hawaiians could have admirable traits in the
absence of Christianity or Western notions of civilization
was an eye-opener. The notion of the noble savage gained
currency.
World History
162
The Huron tribe in Native America was romanticized beyond
measure. The Huron were held up as examples of the noble
savage, excelling in hunting, love, and war. However, among
the Hurons chief entertainments was torturing captives from
neighboring tribes. Understandably, the Huron were utterly
despised by their Iroquois neighbors. As it happened, the
Huron were wiped out by disease before objective reporting
on these people could tarnish their romantic image.
The French Revolution and Napoleon
History is full of paradox and irony. The American Revolution
couldnt have succeeded without the help of the French and
even the Dutch. George Washington couldnt have defeated
Cornwallis at Yorktown in 1783 had French naval forces not
blockaded the British garrison and a fair number of French
ground forces filled out the ranks of Washingtons Continental
Army.
Ben Franklins canny diplomacy managed to grab ever
greater handfuls of treasure from French coffers, such that
King Louis XVI (r. 17741792) was forced to impose onerous
taxes on the French people. The French people rebelled.
Frances Independence Day is July 14. It commemorates the
storming of the Bastille prison in Paris in 1789. The activism
that led to open revolution actually got underway in June of
1789. For the first time since 1614, King Louis felt compelled
to convene the medieval Estates General. The delegates of
that body were, at that point, mostly members of the French
intelligentsia who embraced Enlightenment ideas.
What began with idealistic fervor, turned into a bloody
mess. The monarchy was overthrown in 1792. In March of
that year, France was invaded by Austria and Prussia. In the
first global example of total war, the French people rallied to
defeat the assault. The royal family was executed in 1793.
The worst of what would be called the Reign of Terror took
place in June and July of 1794. In Paris, 1,584 heads rolled
from the bloody guillotine into baskets. Thousands of peas-
ants, clerics, aristocrats, landlords, clerks, and whoever else
offended the self-appointed citizen tribunals were murdered
in other cities and in the provinces. Chaos reigned.
Lesson 4
163
In the midst of this chaos, in 1799 a young French officer,
a Corsican colonel of artillery by the name of Napoleon
Bonaparte, appeared on the scene. He led a military coup to
overthrow the French Directorate, declaring himself First
Consul of the Republic. His agenda was simple: conquer and
unite Europe.
A summary of key dates of the French Revolution is provided
in the box on page 583 of your textbook.
Napoleon (17691821) was a military genius. Before and after
he crowned himself Emperor of the French in 1804, he
scored victory after victory against the Prussians, Austrians,
and Italians. He gained control of Spain. He might have com-
pleted his agenda of European unification had he not made
the same mistake made by Adolph Hitler in the twentieth
century. In 1812 he attacked Russia. His Grand Army was
defeated by Russias sheer size and the Russian winter. He
would meet his final defeat at the hands of the British in
1815.
Map 22.2 on page 582 of your textbook provides an overview
of the impact of the Age of Napoleon. The Chronology sidebar
that ends this chapter is long and complex. Use it to review the
information in this chapter.
Now, review the material youve learned in this study guide
as well as the assigned pages in your textbook for
Assignments 1622. Once you feel you understand the
material, complete Self-Check 8. Then check your answers
with those provided at the end of this study guide. If youve
missed any answers, or you feel unsure of the material,
review the assigned pages in your textbook and this study
guide. When youre sure that you completely understand the
information presented in Assignments 1622, complete your
examination for Lesson 4.
World History
164
Self-Check 8
Indicate whether each of the following statements is True or False.
______ 1. The Wahhabi reform movement called for a return to traditional and purist interpreta-
tions of the Quran.
______ 2. Napoleon imposed a uniform legal code on the lands he conquered, and the Code
Napoleon remains the basis of law in much of Europe, Africa, and Latin America.
______ 3. The decline of the population in the Ottoman Empire relative to that of Europe con-
tributed to a decline in Ottoman power.
______ 4. Like the British, the Dutch colonialists successfully expanded mainland interests in the
Dutch East Indies.
______ 5. In Japan, Enlightenment science had less impact than did internal struggles against
Christianity.
______ 6. The development of Western science both preceded and caused the Industrial
Revolution.
______ 7. China was the worlds fastest-growing empire of the eighteenth century.
______ 8. Under plantation life in the Americas, black slaves lost all control over domestic
practices and any other efforts to craft their own social worlds.
Fill in the blank with the correct term.
9. Immanuel Kant questioned Rousseaus concept of the general _______, recognizing that it
invited the tyranny of majorities.
10. New Zealand was an outstanding example of what Alfred Crosby called a new _________
because its environment was adaptable to a Western style of living.
11. Inspired by the Arabian Wahhabist movement and claiming to be the forerunner of the Muslim
Mahdi, Usuman da Fodio founded the _______ Empire with its capital at Sokoto in the West
African Sahel.
(Continued)
Lesson 4
165
Self-Check 8
12. The British economist David _______ recognized that labor contributed value to products.
13. Scientific measurements gathered by the French scientist Pierre Louis Moreau de Maupertuis
confirmed that the earth bulges at the _______.
14. Based on massive hunting of buffalo, trade, and conquest, the _______ of North America
established a short-lived land empire.
15. Galileo Galilei was to the ________ as Anton van Leeuwenhoek was to the microscope.
16. As prelude to the French Revolution of 1789, the medieval Estates General called to assembly
by Louis XVI, declared itself a/an _________ Assembly empowered to interpret the general
will of the public.
Answer each question in not more than four complete sentences.
17. What part did the Kew Gardens in London and similar gardens in Spain and elsewhere play
in the expansion of resources?
__________________________________________________________
__________________________________________________________
__________________________________________________________
__________________________________________________________
18. What were the four main themes of the Enlightenment in Europe?
__________________________________________________________
__________________________________________________________
__________________________________________________________
__________________________________________________________
Check your answers with those on page 221.
World History
166
NOTES
Frustrations of Progress;
Chaos and Complexity
FRUSTRATIONS OF PROGRESS
The first part of Lesson 5 covers Chapter 23, Replacing Muscle:
The Energy Revolutions; Chapter 24, The Social Mold: Work
and Society in the Nineteenth Century; Chapter 25, Western
Dominance in the Nineteenth Century: The Westward Shift of
Power and the Rise of Global Empires; and Chapter 26, The
Changing State: Political Developments in the Nineteenth
Century.
ASSIGNMENT 23: REPLACING
MUSCLE: THE ENERGY
REVOLUTIONS
Read this assignment. Then read the Part 9 opening spread and
Chapter 23, pages 586613, in your textbook.
Global Demographics: The Worlds
Population Rises
Despite the challenges wrought be disease and ecological
pressures, the global population increased enormously
during the nineteenth century.
Map 23.1 on page 592593 of your textbook is an extension
of a similar map you studied earlier. Notice that along with
continued increases in population growth in Europe and Asia,
a spike in population growth occurs in the Americas and
Australia by 1900.
167
L
e
s
s
o
n

5
L
e
s
s
o
n

5
World History
168
Food: Transition to Abundance
Improvements in food production outstripped population
growth, and increased use of fertilizers helped raise harvest
yields. Particularly in Europe and North America, farming
became more business oriented. As a result, larger farms
that could benefit from economy of scale began to replace
small farms. As that happened, more money was invested in
scientific agronomy.
Industrialization revolutionized food processing, food preser-
vation (through canning technologies), and the ability to
transfer foods to markets over great distances. The advent of
refrigeration was a major factor in extending the range of
food transport. As manufacturers took to the tasks of food
processing, the scale and geographic range of food marketing
increased greatly.
Due to industrialization, food production increased with rela-
tively little addition of human labor. The result was a large,
well-fed population to engage in trade and other industries.
In that context, new cities appeared and urban centers saw
gains in population.
Map 23.2 on page 595 of your textbook gives a geographic
overview of patterns of industrialization, technology, and food
production in the nineteenth century.
Energy for Power: Militarization
and Industrialization
A nineteenth-century trend was the creation of ever-larger
standing armies. The trend began in Europe in the 1790s as
a response to the French Revolution. The main idea wasnt
the militarization of society; it was simply enlarging the con-
cept of citizen armies that harkened back to Roman times. In
any case, with the institution of drafts nations could mobilize
truly enormous armies. In this context, military theorist Karl
von Clausewitz (17801831) conceived the concept of total
war. What that meant, in effect, was that wars would now be
fought between massive armies representing entire national
populations, who, in turn, were expected to support national
war efforts.
Lesson 5
169
The interaction of militarization with industrialization was
inevitable, because the new armies demanded immensely
complex logistics. Massive industrial mobilization was
required to produce weapons and munitions as well as
vehicles.
From the beginning, fossil fuels drove industrialization. At
the outset, the vital fossil fuel was coal. Coal and steam
characterized industrial growth during the nineteenth cen-
tury. Even less industrially developed regions like India,
China, and Japan were familiar with railways, steamships,
and the electric telegraph. Indeed, railways and steamship
routes provided the framework of a new global system.
Study the sidebar table Population, Food, and Energy on
page 600 of your textbook. Then consider the Making
Connections feature on page 601 and consider the military
adaptations of industrial developments and their consequences
with respect to food technology, fossil fuels, new transportation
technologies, and electricity.
Industrializing Europe
Industrialization gained ground fastest in Europe. All the
European nations were crisscrossed by rail lines. Also,
between 1815 and 1914, major cities gained population even
faster than the new armies. By 1900, nine European cities
had populations exceeding one million. However, industrial-
ization tended to be concentrated in particular regions, such
as Belgium and the Ruhr region of Germany. However, the
Netherlands remained almost entirely agricultural. This pat-
tern of interlaced agricultural and industrial regions created
an efficient economic ecology. Regions specializing in food
production fed industrial regions, which, in turn, marketed
manufactured goods to regions specialized in agriculture.
This pattern spread around the world. Some nations special-
ized in producing raw materials and food for industrialized
nations while industrial regions marketed manufactured
goods to the less industrialized nations.
Map 23.3 on page 602 of your textbook provides an overview
of the development of industrialization in Europe by 1914.
World History
170
Industry in the Americas
In the Americas, significant industrial development occurred
almost entirely within the United States. And, within the
United States, most development was in the Northeast and
Midwest. By the 1890s, U.S. manufacturing levels were twice
those of Great Britain and half as much as the whole of
Europe and Britain combined. Reasons for this spectacular
industrial growth included a wealth of resources, adequate
labor forces, and an extensive fleet of merchant marine ves-
sels plying the worlds sea lanes. Meanwhile, the economy in
the U.S. Southeast, with a few exceptions, remained bound to
plantations, agricultural exports, and the exploitation of
black labor. Even though slavery became illegal after 1865,
former slaves who hadnt migrated to the North or West had
few options beyond sharecropping and plantation labor.
Throughout Latin America a dismal pattern dimmed prospects
of industrialization. Latin nations might have formed resource-
rich trading blocs that would have provided significant internal
markets, but they didnt. Rancorous political and boundary
disputes defeated nationalist agendas. Meanwhile, foreign
investment in Latin America gained control of resources,
exploited local labor, and exported profits to the industrial-
ized nations. That pattern would continue throughout the
twentieth centurynot just in Latin America, but around
the world.
Japan Industrializes
This section begins with a description of U.S. imperialism in
the mid-nineteenth century. It briefly describes the arrival of
Commodore Perry in Tokyo Bay with a flotilla of battleships.
Perrys gunboat diplomacy was aimed at awakening the
Japanese backwater to Western ways, thereby forcing
Japan to open its ports to foreign trade. That was achieved.
In fact, it could be achieved because Japan had been carefully
and selectively acquiring and adapting Western industrial
technologies even under the Tokugawa shoguns.
Lesson 5
171
At the outset, following typical Western practices of business
imperialism, Japan was forced into unfair deals that tipped
the balance of trade in favor of foreign interests. However, in
a remarkably brief period, between about 1884 and 1889,
Japan managed to reverse the terms of its trade treaties in
favor of Japans interests. In part, this happened because
Japanese industrialists partnered with the state to effectively
expand Japanese industrialism in the interests of Japan. It
also happened because Japan rapidly became a military
power that would defeat China in 1895 and force foreigners
to view Japan in a new light.
China and Industrialization
Nineteenth-century China felt the impact of Western industri-
alism. Its trade advantages dwindled as the military balance
of power shifted to the West. However, internal problems and
foreign incursions made Chinese efforts at industrialization
slow and painfully awkward. Peasant revolts drained Chinese
resources. An Anglo-French military expedition occupied
Beijing. Chinas militarization efforts were shown to be lack-
ing when Japan defeated Chinese naval forces in a struggle
over Korean domination during the Sino-Japanese War of
18941895.
Beginning in 1861, the Chinese response to its troubles
was the development of self-strengthening policies. Railroads
were built. Telegraph lines were strung. Capital investment
attempted a focus away from munitions and iron ships to
domestic manufacturing of trade goods such as textiles
(especially silk). However, by 1880 China was a preindustrial
exporter of food and fiber goods to the industrial core
nations of the Northern Hemisphere.
India and Egypt
During the nineteenth century, India deindustrialized. Its for-
mer centers of manufacturing collapsed along with the faded
power of Mughal rule. Indians were mainly reduced to peas-
ant farmers who paid taxes to British overloads. British
capitalists took control of Indias manufacturing capacity,
World History
172
and India became a source of raw materials for the British
Empire. For example, the British imported raw cotton from
India and North America. Britains textile mills processed the
cotton into finished fabrics for export to the world, and the
Bank of England bulged with the profits.
Reflect on Map 23.4 on page 609 of your textbook and con-
sider the politics of cotton.
In most of the world, nonindustrialized nations clamored for
Western arms and munitions. In addition, they often invited
foreign capital for the construction of railways and dams and
so forth. Domestic interest in creating industrial economies
was all but absent.
Egypt was an exception. Napoleons Egyptian expedition in
1799 had an impact on Egyptian rulers and administrators.
At that point, recall that Egypt was nominally part of the
Ottoman Empire. In fact, it was a politically independent
state ruled by Mehmet Ali (17691849). Indeed, Mehmet Ali
founded the dynasty that would rule Egypt until 1953.
Egypt exerted vigorous efforts to industrialize Egypt after
1805. However, because the Suez Canal, which was con-
structed by Europeans, was in Egyptian territory, the West,
and Britain in particular, was less than enthusiastic about
Egypt becoming a European-like state. After a military
intervention in 1881, Egypt became a British protectorate
and Egypts monarchy became a puppet government. Egypts
industrializing agenda was thwarted by European interests.
Conclude your study of this chapter with a careful study of the
Chronology sidebar on page 611 of your textbook.
Lesson 5
173
ASSIGNMENT 24: THE SOCIAL
MOLD: WORK AND SOCIETY IN
THE NINETEENTH CENTURY
Read this assignment. Then read Chapter 24, pages 614637,
in your textbook.
The Industrialized Environment
Palaces of Work: The Rise of Factories
Imagine a village somewhere in Europe in the fifteenth cen-
tury. During an ordinary day of an ordinary life, you might
chat with the baker, greet the local landlord, or visit the
village priest. People of different social stations werent
unaware of differences in wealth or learning, but social
identities were strongly associated with parentage, occupa-
tions and trades, religious life, and the turns of the seasons.
A walk beyond the village would take you into open meadows
or forests. Children gathered to frolic in a local pond. Nature
was a neighbor.
Now imagine rows of shoddy dwellings in a crowded factory
town in nineteenth-century England or Belgium. Stench and
gloom pouring from factory smokestacks hangs in the air.
From dawn to dusk, your companions and peers are fellow
workers. You seldom even glimpse the people who own the
factory or, in all likelihood, your modest home. The rich
people live in other places. You, and even your work super-
visors, own little other than the value of your labor. However,
you have little or no control over work conditions, the hours
youre expected to work, or how much youre paid. And,
especially if you happened to be a miner, nature was more
like your daily tomb than a neighbor.
Critics of Industrialization
The rise of industrialism radically altered patterns of social
organization. Social identities became almost entirely defined
by relative wealth. The varieties of social stations gave way to
World History
174
the concept of social class. Social class can be defined as
ones relationship to the means of production. The working
class owned nothing but their labor. The capitalist class
owned the means of production and, in effect, the lives of
workers. The middle classes served the purposes and inter-
ests of the capitalist class.
Adam Smith, the author of Wealth of Nations, saw both sides
of the rise of capitalism. On the one hand, he observed
correctlythat industrialism in the hands of captains of
industry generated unprecedented levels of wealth. On the
other hand, he recognized the wretched conditions imposed on
workers and their families. In that light, he felt that the new
economic system should require a level playing field for aspir-
ing entrepreneurs, equal access to information about markets,
and adequate government regulation to prevent abuses of labor
and concentrations of wealth based on monopolies. Smith was,
after all, a professor of moral philosophy.
Karl Marx (18181883) was the sharpest and by far the most
influential critic of the capitalist system. As a tireless scholar
and radical journalist in his German homeland, Marxs
evaluations of the nature and contradictions of capitalism were
essentially correct. For example, his observation that workers
felt alienated by the harsh conditions of factory life and having
no meaningful sense of accomplishment from performing
tedious factory work was correct. However, his theory of
history as a record of class struggle was at best incomplete
and at worst, wrong. Marx proposed that the opposed interests
of the proletariat (workers) and the bourgeois class (capitalists)
would lead inevitably to revolution. The proletariat would
overthrow the capitalists to establish a dictatorship of the
proletariat, which, in turn, would give birth to a communist
world order based on equality and social justice.
As you may know, the Communist experiment in Russia turned
out to be little more than a new approach to totalitarianism.
However, the influence of worker rebellions, partly informed
by the international labor movement, did lead to improvements
in workers rights and working conditions in the United
States and elsewhere.
Lesson 5
175
Urbanization
Urbanizationthe growth of citiestook place in both indus-
trialized and nonindustrialized nations. As trade stimulated
economic activities around the globe, cities tended to grow up
around ports or, inland, around centers of production and
trade. As all this happened, the grim, crime-infested conditions
of eighteenth-century cities gave way for much improved
city planning, including more rational street layouts and
improved sewage systems. At the same time, public invest-
ment in schools improved literacy rates and helped train
workers for industrial jobs.
Check out the Most Populous Cities in 1900 table on
page 620 of your textbook. Take a look Map 24.1 on the same
page to consider the growth of Manchester, England. The
Making Connections feature on page 621 summarizes
industrial transformations in the nineteenth century.
Beyond Industry: Agriculture
and Mining
Industrialization changed the global social and economic
picture even though extensive regions of the world remained
rural. In particular, patterns of relationships between land
and people were changing. For example, in Australia, Argentina,
and the American Midwest, mechanized farming increased
production of grains and meat with less labor. As a result,
people left farms to take up other kinds of occupations,
often in urban areas.
The Ottoman Empire became an enormous net exporter of
produce like grapes and opium, as what had been peasant
lands increasingly fell under the private ownership of local
chiefs, merchants, moneylenders, and public officials. Often
the new private landowners were outsiders from places like
Greece or Armenia.
In India under the British, the traditional caste system
became a justification for favoring the upper castes, such
as the Brahmans, as landholders. As a result, many lower-
caste peasant-farmers were thrown into poverty.
World History
176
In nonindustrialized regions of Africa, peasant and village life
was heavily disrupted by European ventures. In areas of
Portuguese control, like Angola, illegal slaving continued
into the twentieth century. In South Africa, the discovery of
diamonds led to the virtual enslavement of native Africans
recruited for mining.
Study the Making Connections feature on page 621 of your
textbook for a summary of industrial transformations in the
nineteenth century.
Changing Labor Regimes
Your textbook reminds you that slavery was both common
and socially acceptable in most areas of the pre-industrial
world. Slavery was an economic foundation of the Greco-
Roman world. Serfdom persisted in Russia and Eastern
Europe into the early nineteenth century. However, with the
rise of industrialism, both slavery and serfdom vanished
from North America and Europe. Evangelical Christianity,
Enlightenment concepts of egalitarianism, and neo-liberal
beliefs that people were more productive if they worked for
wages were major factors in this trend.
The inset table on page 625 of your textbook, Slavery in the
Nineteenth Century, provides a summary of benchmarks in
the history of the slave trade.
Yet, as slavery and serfdom were abolished, the need for
cheap labor accelerated. Traditional forms of forced labor
faded away to be replaced by various forms of labor exploita-
tion. Massive migration out of China provided so-called
coolie labor that would help build American railroads.
Women and even children filled elites demands for cheap
labor in Ottoman textile factories and British tea plantations.
Australias version of cheap labor was supplied by convicts
from other parts of the British Empire. Massive immigration
from Europe was a vital source of cheap labor for the United
States and Canada.
Map 24.2 on page 627 of your textbook offers a geographic
overview of the movement of indentured labor in the nineteenth
century.
Lesson 5
177
Hunters and Pastoralists
People were still pursuing ancient, traditional life-ways
during and after the nineteenth century. However, across
the planet, the lot of indigenous tribal peoples ranged from
tragic or terminal to transformed. In the United States and
Canada, Native Americans were mainly pushed onto reserva-
tions. The free-ranging lifestyles of the Plains Indians were
mainly a memory by the 1870s. Basically, the only places where
hunters and pastoralists survived were in areas that were
too remote or isolated to get much attention from Europeans.
The Arctic Inuit peoples, the Australian Aboriginals of the
outback, and isolated tribal peoples of the Amazon are exam-
ples. Particularly beyond the Old World, disease and outright
violent eradication greatly reduced indigenous populations.
In the Old World, the more typical pattern was inducing
nomadic pastoralists to adopt modern ways of life.
Elites Transformed
If it can be said that the common people of the world were
often brutalized or traumatized by social and economic
change during the nineteenth century, it was also the case
that traditional aristocracies took a beating. Disparities in
wealth characterized the emergent social class system of the
industrialized world. In Europe, for example, the traditional
basis of aristocracy had been landed estates. With the rise of
capitalism and industrialism, wealth, power, and influence
fell increasingly into the hands of successful entrepreneurs
whose ancestors were more likely to be merchants or trades-
men than blue-bloods with long pedigrees. In North America,
where traditional aristocracy was largely nonexistent, wealthy
captains of industry became the founders of socially promi-
nent families with names like Roosevelt, Vanderbilt, Carnegie,
and Rockefeller.
A major key to this chapter is Map 24.3 on pages 632633 of
your textbook . Notice the multiple streams of immigration flow
into the United States from Europe and Asia. Also, an inset
map on page 632 offers a fascinating map of Manhattan ethnic
neighborhoods in 1920.
World History
178
Industrialization imposed a considerable degree of uniformity
on Western society. However, an immense gap between devel-
oped and developed countries emerged, largely characterized
by a commensurate wealth gap. These conditions remain,
characterizing many of the problems of the twenty-first
century.
The Chronology sidebar that concludes this chapter is long
and complex. As you use it for review, make sure you take
the time you need to ponder key events and their dates.
ASSIGNMENT 25: WESTERN
DOMINANCE IN THE
NINETEENTH CENTURY: THE
WESTWARD SHIFT OF POWER
AND THE RISE OF GLOBAL
EMPIRES
Read this assignment. Then read Chapter 25, pages 638663,
in your textbook.
The Opium Wars
In 1839, the emperor of China appointed Commissioner Lin
to oversee new, harsh penalties for dealing in or using opium.
In that context, Lin gathered up all the opium he could find
in China and dumped it into the sea. The British werent
pleased, because they dominated and profited from the
opium trade. In 1840, declaring a principle of international
free trade, British naval forces blockaded and sacked Chinese
ports, forcing a reopening of trade. Under the Treaty of
Nanjing, China ceded Hong Kong to Britain and opened other
ports to British trade. Soon thereafter, the United States,
France, and Sweden forced similar trade deals from the
Chinese. In 1860, with China all but paralyzed by the Taiping
Rebellion, French and British forces marched to the Chinese
capital at Beijing, burned the imperial summer palace, and
forced new trade concessions from China.
Lesson 5
179
The Opium Wars were all about arriving at a balance of trade
with China that favored the British. Thats the immediate
significance of the Opium Wars. The broader significance of
this tale of Chinese woe is what it tells us about the global
balance of power. Mainly due to superior military technology,
the nineteenth century witnessed a westward shift in power.
China was no longer the central global superpower it had
been for many centuries.
The White Empires: Rise and
Resistance
The end of China as a dominant power coincided with the
rise of Western imperialism. Thats what youre exploring in
this section. Frankly, the key to this section is reading the
textbook while studying the map. Take note of two significant
factors in the ascendance of Western power. One was popula-
tion. The population in Africa was shrinking dramatically.
The population of Western states had expanded considerably
relative to the dense populations of Asia. Second, what
Western powers lacked in manpower they made up for with
industrial capacity and superior technology.
Map 25.1 on page 642 of your textbook can help you grasp the
extent and geographic focuses of foreign imperialism in East
and Southeast Asia between 1840 and 1910. Study it in con-
junction with Figure 25.1 on page 643 to contrast areas and
populations controlled by European empires by 1939. Note
Great Britains considerable predominance. Map 25.2 on
pages 644645 is a global map of the imperial world as of
1900. Refer back to it as you work with the textbook
discussions.
The Making Connections feature on page 648 challenges you
to think about relationship between technological development
and imperialism. The discovery and use of quinine to combat
malaria and the development of steam power are examples.
World History
180
Methods of Imperial Rule
The focus of this section is on how the massive populations
of the European empires were controlled by relatively small
numbers of colonial administrators. The key word here is
collaboration. For example, British India was administered by
collaboration between local British and native Indian gover-
nors and administrators. In that context, substantial regions
of India were ruled by Indian princes. British armed forces of
the Raj (British India) depended heavily on Indian troops,
such as the famed Bengal Lancers. In Belgian Rwanda, Tutsi
tribesman were schooled and trained to serve as bureaucrats
and administrators. French protectorates in Morocco and
Tunisia were ruled by puppet Arab monarchs while native
sultans ruled autonomously in parts of the Dutch East
Indies.
Business Imperialism
Nineteenth-century imperialism was aimed at combining
maritime and land dominance. In economic terms, that
meant controlling both trade and production. However, the
profit surpluses of Western business corporations created the
possibility of straight-out business imperialism. The formula
is simple: Deals are made with local governments allowing
foreign capital investments in land and resources. The deals
are sweetened by giving local elites a cut. As foreign investors
capture some or most of the local economy, native labor is
exploited and local government policies are shaped through a
combination of bribes and intimidation. Meanwhile, corporate
profits fly back to the foreign investors and foreign bank
accounts. The concept of a banana republic characterizes
the final stage of business imperialism, although in cases like
Chile you would substitute copper for banana.
Following on the Monroe Doctrine of 1823, the United States
declared exclusive trade (and intervention) rights in the
Western Hemisphere. European and other powers were to
keep out. Thus, the primary arena of U.S. business imperial-
ism was resource-rich Latin America.
Lesson 5
181
The Chronology sidebar on page 653 of your textbook will
help you associate key events in the context of new European
imperialism in Africa and Asia. Map 25.3 on page 654 pro-
vides a geographic overview of the manner in which Africa
was divided up into European colonial possessions. Make a
point of locating the sole exception to this processEthiopia.
Ask yourself how this nations history may have accounted for
its successful resistance to European territorial designs.
Imperialism in the New Europes
Europeans sought colonies with climates and ecologies
where European-style societies could thrive. Most of the
New Europes, such as Australia, New Zealand, and Canada,
were part of the British Empire. South Africa had been
conceived as a European-style nation while the Dutch (Boer)
dominated it. When the British took control, after 1902, Boer
populations were accommodated and the character of South
Africa remained much as it had been. The French, meanwhile,
attempted to recast Algeria as an extension of France.
A significant aspect of nearly all of the New Europes, including
Argentina and Uruguay, was the relocation or eradication of
indigenous peoples. South Africa was an exception in that
native Africans were allowed to live, if only to provide their
labor for white elites.
Empires Elsewhere: Japan, Russia,
and the United States
The British were top dog among global imperialists. However,
Japan, Russia, and the United States were also engaged in
imperial ventures.
In East Asia, Japans imperial sentiments were spurred by
anxious awareness of Western and, in particular, Russian
imperial agendas. Between 1870 and 1910, Japan consoli-
dated its control of the Japanese islands and gained control
of Korea and Taiwan.
World History
182
Russian expansion in the east included extending its Siberian
boundaries and occupying Manchuria between 1900 and
1905. Expansion in Central Asia embraced mainly Muslim
states downward to the borders of Mongolia, Afghanistan,
and Persia (Iran). Westward expansion swallowed Finland,
Ukraine, and areas south of the Caucasus.
U.S. imperialism got underway with the acquisition of
Thomas Jeffersons Louisiana Purchase (from Napoleon)
around 1801. But the big push took place during Jacksons
presidency in the 1830s as Native Americans were pushed
toward Oklahoma. The relocation was brutal. The Cherokee
referred to it as the Trail of Tears. The Mexican War of 1846
overturned Mexicos claims in the Southwest. U.S. forces
pushed the Mexican border southward to the Rio Grande.
Alaska was purchased from Russia in 1867. American traders
overthrew the Hawaiian monarchy in 1893. Finally, as a
result of the Spanish American War of 1898, the United
States took possession of Puerto Rico, American Samoa,
and the Philippines, while establishing a de facto protectorate
in Cuba. In 1904, after enabling Panama to secede from
Colombia, the United States took control of the Panama
Canal Zone.
Map 25.4 on page 657 offers a geographic overview of Russian
and Japanese imperial expansion between 1868 and 1918.
Rationales of Empire
People like to justify their aggressive actions. In that respect,
empires are like overgrown people. Therefore, it was inevitable
that Western imperial agendas would proclaim narratives
that justified conquest. Looking at the big picture, there were
two fundamental rationalizations for Western imperialism.
The most important was spun from scientific studies in biol-
ogy. A close second was the zeal of Christian missionaries.
Lesson 5
183
Doctrines of Superiority
The publication of Charles Darwins Origin of Species in 1859
ignited a firestorm of controversy on the one hand and a bible
for social theorists on the other. Darwin drew conclusions that
remain cornerstones of modern biology: (1) New and divergent
species arise from common ancestors. (2) This process has
been going on for a very long time, as less complex organisms
have evolved into more complex organisms.
(3) The mechanism by which species adapt to their environ-
ments is natural selection. In effect, animals or plants that
are better adapted to their environments are more likely to
reproduce and pass on these nature-preferred adaptive traits
to their offspring. This is what Darwin meant by survival of
the fittest.
Darwin cant be blamed for all the bent and twisted ways
political theorists and quasi-scientists would interpret his
idea about more evolved species or survival of the fittest.
He wasnt entirely responsible for notions that white people
were superior (more highly evolved) than black or brown
people. And most certainly be cant be blamed for the ideas
of social theorists like Herbert Spencer. Spencer purloined
Darwins concept of survival of the fittest to justify the
radical social disparities of capitalism and the agendas of
imperialism. He gave birth to what we now call social
Darwinism.
The second motivator and justifier of imperialism was the
ever-so-well-intentioned desire to introduce heathen souls
to Christianity. As noted earlier in this course, wherever
Europeans claimed new lands in the name of (name your
favorite monarchy), priests or pastors were there to bless the
occasion. Wherever there were imperialist conquests, there
were missionaries. And, lest you forget, keep in mind that
missionaries equated Christianity with civilization.
Once again your assignment concludes with a Chronology
sidebar on page 662 of your textbook.
World History
184
ASSIGNMENT 26: THE
CHANGING STATE: POLITICAL
DEVELOPMENTS IN THE
NINETEENTH CENTURY
Read this assignment. Then read Chapter 26, pages 664689,
in your textbook.
Nationalism
What we call modernization is actually Westernization. Thats
the case because the features of states around the world,
even including the adoption of Western styles of dress by
power elites, were developed among the great Western pow-
ers. Powerful forces that shaped Westernized states include
nationalism, constitutionalism, and centralization accompa-
nied by militarism.
In a sense, nationalism can thought of as a romantic fabrica-
tion. Nationalism is based on the assumption that people who
speak the same language, share similar customs, and live
within distinct regions share a common nationality. This
assumption is flawed. The French Revolution and the
Napoleonic Wars, for example, spread the idea that all the
peoples of France were French. In fact, not unlike the United
States, French borders embrace several nationalities,
including people of southern France who speak Provenal.
The cantons of Switzerland include French and German
ethnic populations.
Study Map 26.1 on page 667 of your textbook. In particular,
study the patchwork of nationalities contained solely with the
boundaries of the Hapsburg Empire. Simply imagining possible
nationalist stresses within the Hapsburg domain will give you
a clue as to why this region has long been a hotbed of political
firestorms.
Europeans and politicians embraced the concept of nationalism,
especially in the context of efforts to unify countries that
were patchworks of distinct regions, such as Italy or what
would become Germany under the influence of the statesman
Otto von Bismarck in the late nineteenth century.
Lesson 5
185
The Case of the Jews
As nationalism became a patriotic buzzword and a key
concept of citizen identity, the Jews scattered across Europe
were caught in a dilemma. Since medieval times, the Jews
had had no distinct nationality. There were Italian Jews,
German Jews, Polish Jews, and so on. As the age-old
European disease of anti-Semitism reared its head, only two
options seemed viable: establishing a Jewish homeland or
assimilation. The movement that would come to be called
Zionism called for establishing a Jewish homeland, perhaps
in Africa or Palestine. However, most Jews took the second
option and attempted to assimilate into whatever nation they
were living in. Indeed, before World War II, German Jews
were, for the most part, completely assimilated into German
culture, making significant contributions to the German state
as scientists, artists, and even as politicians.
Nationalism beyond Europe was almost entirely a product
of reaction and resistance to Western imperialism. Your text-
book offers numerous examples of such instances in the
nineteenth century. But in fact, nationalist sentiments con-
tinue to inform rebellion and resistance to Westernization in
current times.
Map 26.2 on page 669 of your textbook will help you locate
centers of global resistance to European and United States
imperialism between 1880 and 1920.
Constitutionalism
Constitutionalism embraces the idea that the rulers and
citizens of a state should be mutually bound by laws and
rules acceptable to the governed. Think of the opening line
of the United States Constitution to get a sense of that idea:
We the people of the United States of American, in order to
form a more perfect union . . . .
In fact, responses to sentiments endorsing constitutionalism
in Europe had a minimal impact on the realities of governance.
The nineteenth century was an age of monarchy. However,
gradually and painfully, monarchs began to accept limited
republican ideas, either by accepting constitutions for their
realm or granting decrees or charters of citizen rights.
World History
186
In that context, the British Parliament was gradually trans-
formed from an aristocratic club to a legislative body with
middle-class and even working-class members.
Beyond Western Europe, Russia and the Ottoman Empire
went to great lengths to stave off demands for constitutional-
ism. However, constitutionalism did spread to Japan. That
nations 1882 constitution offered some concessions to the
middle class while maintaining that Japanese sovereignty lay
solely in the hands of the emperor. Latin American states
also adopted constitutions. However, because de facto rule
was in the hands of military strongmen, Latin American con-
stitutions were mainly window dressing. They imposed no
real constraints on what amounted to autocratic, dictatorial
governance.
Centralization, Militarization, and
Bureaucratization
Bureaucracy is a basic feature of Western and westernized
states. A bureaucracy is a hierarchical organization that
specifies the roles of its personnel through formal, written
guidelines and rigid lines of authority. The centralization of
power in modern states is dependent on bureaucracies
designed to manage such things as census information,
taxation, and the administration of public facilities, trade,
education, and so on. Militarization, a phenomenon youve
already considered, is organized in the manner of bureau-
cracies designed to manage logistics (supply and transport),
military operations, intelligence gathering, and personnel
through strict chains of command. You can get a sense of the
rationality and organization of bureaucratic organizations
civil or militaryby thinking of them as social expressions of
mechanization. A state bureaucracy or an army is a sort of
human machine. The military version of these machines is
designed to destroy other such machines and their capacity
to function. A basic principle of modern militarism is absolute
destruction of an enemys capacity to make war. The aim is
total victory and unconditional surrender.
Beyond the industrializing world, one might think that these
Western ideas would be considered alien. In fact, however,
exposure to Western ways of governance and waging war was
Lesson 5
187
an inevitable effect of colonization and imperialism. One of
the first things nonindustrial nations tended to emulate in
their efforts to resist Western dominance was Western ways
of war. Interesting examples of resistance to Western power
in the Islamic world are offered in your textbook. Also, the
case of Thailand is fascinating because that nation adopted
progressive, liberal reforms precisely by being able to evade
conquest by European powers.
The Making Connections feature on page 677of your textbook
summarizes key factors in state modernization over the nine-
teenth century: nationalism, constitutionalism, centralization,
militarization, and bureaucratization.
Religion and Politics
The nineteenth century provided stages for all kinds of dramas
involving religion and the state. In Japan, the emperor styled
himself the chief priest of a reorganized approach to Shinto, an
ancient and traditional Japanese religion that was a mix of
nature and ancestor worship. His objective was to emulate
Western success in exercising state control over religion.
The situation in the West wasnt as simple.
In Europe and the Americas, church-state competition was a
theme of the age. In Catholic domains, with the exception of
Pope Leo XIII (r. 18781903), who was a progressive reformer,
the Catholic Church stood fast against change of any kind. In
nearly all of Latin America, Catholicism was the state reli-
gionperiod. Mexico would eventually overthrow the power of
the Catholic Church, but that was an exception to the rule.
Elsewhere in the West, contests between church and state took
a variety of forms. In the United States, the U.S. Constitution
espoused religious toleration and the separation of church and
state. But that didnt do much to stem the zeal and appeal of
all kinds of Christian evangelical movements. In both Europe
and America, organized religion fell under shadows of doubt.
Mainly in the Northeast United States, educated Americans
were influenced by the Transcendentalist movement espoused
by thinkers like Ralph Waldo Emerson and Henry David
Thoreau and poets like Walt Whitman. Transcendentalist
World History
188
thought absorbed aspects of Eastern mysticism. By contrast,
educated people influenced by Darwin and science in general
were often inclined to embrace atheism.
The Muslim world featured a number of Islamic reform move-
ments, especially in Africa. Among the most remarkable of
these was the rise to the Mahdist State of Sudan proclaimed
by Muhammad Ahmad in 1881. Its called Mahdist because
Ahmad saw himself as a Mahdi, an inheritor of the mission
of the Prophet Muhammad. The Egyptian and British govern-
ments were unable to contain Ahmads jihad until 1898 at
the Battle of Omdurman.
Map 26.4 on page 679 of your textbook locates Muslim reform
movements in Africa and Arabia over the nineteenth century.
New Forms of Political Radicalism
The United States was an unprecedented (radical) experiment in
democracy, and the idea of government of, by, and for the peo-
ple was an American invention. In spite of unresolved issues
like slavery and states rights, the U.S. Constitution and the
Declaration of Independence provided templates for democracy
all over the world. That doesnt mean that American-style
democracy swept the world during the nineteenth century.
It didnt. But ideals of democracy became seeds that would
continue to sprout in the twentieth and early twenty-first
centuries.
Even in places where democratic reform wasnt on the
horizon, there was another radical development during the
nineteenth century. Namely, in pubs, parks, barber shops,
union halls, and parlors people gathered to ponder and
debate public concerns and issues. Historians refer to this
phenomenon as the broadening of the public sphere. Where
that sphere was best developed, mainly in North America
and parts of Europe, democratic reform was most likely to
followsooner or later.
Lesson 5
189
Western Social Thought
The opposition of haves and have-nots has always been a
dynamic factor in the rise and fall of states and empires.
However, in the West during the nineteenth century ideals of
democratic reform and the broadening of the public sphere
gave rise to a spectrum of radical political ideologies.
An ideology is a more or less consistent system of ideas about
how the world ought to be. Important political ideologies of the
nineteenth century included socialism, anarchism, capitalism
(including social Darwinism), utilitarianism, and libertarianism.
Socialism in all its varieties represents the interests of work-
ers and ordinary citizens. Socialist agendas focus on social
equality and economic equity. Anarchists want governments
abolished; they set fires and throw bombs. Capitalism is an
ideology of economic elites. In the United States, capitalism
and nationalism are joined at the hip. Capitalists pretty much
dominate the shaping of public opinion and government policies.
Utilitarianism, which is associated with the British reformer
Jeremy Bentham (17481832), called for reforms that would
provide the greatest happiness to the greatest number.
Libertarianism is associated with Benthams protg, John
Stuart Mill (18061873). Mill called for institutional reforms
that would give individuals the greatest degree of autonomy
and liberty. In that context, he embraced feminist causes.
The faces of social and political ideologies include religion,
utopianism, democracy and political radicalism. Key people
and events associated with these ideologies are provided in
the inset box page 686 of your textbook. Also, consult the
Chronology sidebar on page 687.
Please complete Self-Check 9 now.
World History
190
Self-Check 9
Indicate whether each of the following statements is True or False.
______ 1. With the collapse of its traditional industries, India underwent a process of innovative
industrialization.
______ 2. The Prussian military theorist Carl von Clausewitz maintained that militarization made
wars worse and, therefore, they should be avoided in favor of diplomacy, save in cases
when a state was directly attacked.
______ 3. The first step in the transformation of the North American prairies toward greatly
expanded food production was the increase of lands devoted to grazing.
______ 4. Under British rule, Indias caste system was largely eliminated.
______ 5. As industrialization produced generations of the new rich in Britain, expensive new
public schools arose. They were called public, even though they were private, to
infer that the new universities had public importance.
______ 6. During the era of European industrialization, Belgium and Holland developed
complementary agricultural-industrial economies.
______ 7. Outside of Europe, nationalism was directly related to resistance to European
imperialism.
______ 8. Mainly through rebellion, African slaves in the Americas played a major role in their
gradual liberation.
______ 9. Rising anti-Semitism occurred in Europe at the same time that European Jews were
seeking to assimilate into Western society.
(Continued)
Lesson 5
191
Self-Check 9
Fill in the blank with the correct term.
10. In the view of Karl Marx, the rise of industrialization and capitalism created inevitable
antagonism between the bourgeois class that owned the factories and the worker class he
called the _______.
11. In the context of advances in food preservation, bottling and canning were based on
simultaneous sealing and ________ processes.
12. Laborers called _______ were imported into the Americas from China and India to serve as
poorly paid workers in mines, plantations, and wherever railways were under construction.
13. A nineteenth-century development in the West was a broadening of the _______ sphere as
people more often gathered to exchange ideas and opinions in clubs, pubs, civic associations,
and places of worship.
14. As a general principle, the pace of commerce is a direct function of the size of the _______.
15. Social _______ was an ideological misinterpretation of the concept of the survival of the
fittest.
16. Chinas response to Western industrialization and the erosion of Chinas dominant role in the
global economy, was a program of reforms they called _______-_______.
Answer each question in not more than four complete sentences.
17. What were some basic factors that help explain why people left farms to work in factories?
__________________________________________________________
__________________________________________________________
__________________________________________________________
__________________________________________________________
(Continued)
World History
192
CHAOS AND COMPLEXITY
The second part of Lesson 5 covers Chapter 27, The
Twentieth-Century Mind: Western Science and the World;
Chapter 28, World Order and Disorder: Global Politics in the
Twentieth Century; Chapter 29, The Pursuit of Utopia: Civil
Society in the Twentieth Century; and Chapter 30, The
Embattled Biosphere: The Twentieth-Century Environment.
At this point, weve crossed over threshold into the world all
of us sharefor better or for worse. Names, if not dates, will
be more familiar to you. World events, like the World Wars
and the Cold War, will be easier to connect to the world we
know today. In that light, your study tips for these final
chapters will focus less on details and more on generalized
overviews and commentaries.
Self-Check 9
18. During the era of white imperialist expansion, what made Ethiopia different?
__________________________________________________________
__________________________________________________________
__________________________________________________________
__________________________________________________________
19. Briefly explain the main idea embraced in Jeremy Benthams utilitarianism and where it stood
in the spectrum of nineteenth-century political ideologies.
__________________________________________________________
__________________________________________________________
__________________________________________________________
__________________________________________________________
Check your answers with those on page 222.
Lesson 5
193
ASSIGNMENT 27: THE
TWENTIETH CENTURY MIND:
WESTERN SCIENCE AND THE
WORLD
Read this assignment. Then read the Part 10 opening spread
and Chapter 27, pages 690717, in your textbook.
As Western science became an overwhelming, transforming
force around the world, it shattered worldviews and created
deep uncertainties. It seemed that every scientific advance
had side effects. Every advance in military technology made
war more horrifying. Every advance in medicine was accom-
panied by side effects that raised ever greater uncertainty
about the causes and treatment of disease even as public
demand for medical miracles was feverish. Every advance in
manufacturing and mass production seemed to steal the
dignity of craft and trivialize individual lives.
Western Science Ascendant
In China, the revolution of 1911 ended the Qing Empire and
the birth of a Chinese Republic. Western ideologies informed
the revolutionaries, and after the formation of the Science
Society of China in 1914 Western science became increas-
ingly influential.
In 1899, British Viceroy Lord Curzon proclaimed a British
mission to bring the gifts of Western law, literature, religion,
and science to India. Thereafter, with British support and
funding, Indians were educated and trained in various areas
of medicine and science. Similar, if less ambitious, agendas,
especially with respect to medical training, unfolded in the
Dutch East Indies.
Adoption of Western science was weakest in the Islamic world,
but there were notable exceptions. Scientific interpretations of
the Quran were popular in some regions. In Turkey, Kemal
Atatrk (18811938) transformed that nation into a secular,
European-like state. A similar development occurred in Iran.
In both cases science gained ground.
World History
194
Scientific encroachments into Africa were weak or limited.
The Swiss theologian and physician Albert Schweitzer
(18751965) devoted his life to bringing Western medicine
to West Africa.
Map 27.1 on pages 696 and 697 summarizes the spread of
Western science and learning by noting some key events in
different places across the globe.
The Transformation of
Western Science
The advancement of science presents a paradox. Even as
science has made astonishing advances, ever greater areas of
uncertainty have raised doubts and questions. Imagine an
expanding circle. Within the circle resides all science has
confirmed through observations and experiments. Yet, as the
circle expands, the circumference of the circle also expands,
and the boundary between the known and the unknown or
unknowable increases proportionally.
Physics, accompanied by astronomy, has led the way in
unveiling uncertainties and baffling observations that defy
the imagination. Under Einsteins theory of relativity, time
becomes variable as a function of velocity, and matter is but
a form of energy, or vice versa. Under his General Theory,
gravity warps space, not to mention time.
In the field of quantum mechanics, Heisenbergs uncertainty
principle suggests that whats observed and what an observer
chooses to observe are entangled. Mind and matter are corre-
lated, or perhaps consciousness precedes manifestation and
Plato was right after all. What we call atomic particles (like
neutrons and protons) turn out to be composed of not-things
called fermions, which include electrons and quarks. These
not-things vibrate as circular standing waves that are both
local and everywhere at the same time. The implication is
that everything is connected to everything else under a
phenomenon called nonlocal connection. As one physicist put
it, Not only is the universe stranger than we imagine, its
stranger than we can imagine. Or, as another great physicist
suggested in the 1930s, the universe appears to be more like
a great thought than a great machine.
Lesson 5
195
The life sciences, particularly molecular biology and genetics,
have offered the world remarkable advances in medicine and
food production. The problematic side effects there have to do
with unequal access to hi-tech medicine and as yet unknown
ecological side effects of genetically modified seeds and
plants. Also, moral issues related to cloning loom on our
horizon.
The cybernetic sciences, along with related advances in
electronics, have transformed modes of communication. The
great moral and philosophical issue related to computers is
the search for artificial intelligence (AI). Can a machine
think?
The human sciencesanthropology, sociology, and psychology
have opened eyes and raised troubling issues. For example,
the concept of cultural relativism posed by anthropologists
maintains that cultural worlds must be evaluated in terms of
their unique norms, values, and worldviews. That sort of idea
is a slap in the face to people struggling to protect and pro-
mote human rights around the world. How can cultural
relativism and concepts of universal human rights be recon-
ciled? Meanwhile, advances in psychology that began with
Sigmund Freud raised a problem. If, as seems apparent from
clinical research, many of our motivations are subconscious,
how can we apply laws based on the assumption that people
are rational actors who are automatically able to differentiate
right from wrong if they are judged sane?
Philosophical responses to Western sciencesome of them
not all that interested in understanding actual sciencehave
reflected the gloom that has attended the bloodiest century
known to human history. European philosophers declared
that we live in an age of alienation, others that we live in age
of anxiety. Existentialists like French philosopher Jean Paul
Sartre (19051980) maintained that theres no reality but the
one we create for ourselves.
Study the sidebar box on page 709 of your textbook to check out
key dates in the diffusion and transformation of Western science.
Refer to the Making Connections feature on pages 710711. It
lists key scientific disciplines and summarizes new theories and
their effects on society.
World History
196
The Mirror of Science: Art
The main theme here is the idea that art changes in step
with advances in the sciences. And most would agree that
seems to be the case. The French impressionists viewed a
world dissolved into sensory impressions. Abstract painters
like Kandinsky and Picasso created images that dissolved
the objective world into alien geometries. Surrealists like
Salvador Dali created dreamlike landscapes where watches
melted. Marc Chagall gave us an image of lovers seeming to
float in a bizarre interior. One of the most interesting obser-
vations here is the rise to prominence of fantasy artart that
escapes the hard-edged materialist modern world.
The Turn of the World
During the second half of the twentieth century, apparently in
reaction to perceived flaws in Western thought and Western
institutions, there arose something now referred to as the
counterculture, the age of hippies and flower power.
However, even as that era of the fabled sixties began to
fade, we may recognize that what began in the era of
psychedelics and antiwar protestors was the vanguard
of a wider phenomenon.
Today, there continues to be a very wide and diverse interest
in Eastern wisdom, alternative methods of healing, and alter-
native paths to spirituality. A lot of this has to do with a
parallel concern about ecological and social damage that
began to arise in public awareness around the time of the
counterculture.
Review the Chronology sidebar on page 716 of your textbook.
Lesson 5
197
ASSIGNMENT 28: WORLD
ORDER AND DISORDER:
GLOBAL POLITICS IN THE
TWENTIETH CENTURY
Read this assignment. Then read Chapter 28, pages 718747,
in your textbook.
The World War Era, 19141945
The First World War began in August of 1914 and ended
in November of 1918. The initial German invasion through
Belgium quickly ground to a halt, and the fields of combat
on the Western Front, mostly in France and Belgium, were
relatively stagnant. Movement by either side was likely to
gain only limited ground, which might be taken back again
in the next round of carnage. This situation led to the horror
and misery of trench warfare as the static line widened to run
from the border of Switzerland to Calais. The indescribable car-
nage of the war was due to insane tactics that sent massed
infantry against artillery fire, barbed wire, and machine guns.
The first stage of an assault was typically marked by massive
artillery barrages that could continue for days. Thousands of
men not shredded by shrapnel drowned in mud. Efforts to
break out of the stalemate led to the use of poison gas and
the development of the first tanks. By 1918, ten million men
had died in combat. Millions more died from disease or the
misfortune of being a civilian in the wrong place at the wrong
time. The best and brightest of an entire generation went to
sudden graves.
Take a look at the box at the bottom of page 721 of your
textbook to compare figures for dead and wounded among
the various states involved in the First World War. The war
involved states with roughly similar ideologies.
The Western Allies included Britain (including the
Commonwealth states of, Australia, Canada, and New
Zealand), France, the United State, and Russia. The Central
Powers included the Austro-Hungarian Empire, Germany,
World History
198
the Ottoman Empire, and the Kingdom of Bulgaria. Italy
dropped out of its earlier alliance with the Central Powers
and ended up fighting against the Austro-Hungarian Empire
and Germany.
The War ended the Austro-Hungarian and Ottoman Empires
and, overall, the political map of much of the world was
redrawn. Colonies switched hands in Africa. Most of Ireland
gained independence from the British Empire.
Map 28.1 on page 722 of your textbook shows Europe, the
Middle East, and North Africa in 1914 and then again in 1923.
Notice that the changes are mainly characterized by the
emergence of new nation states.
Under President Woodrow Wilson (18561924), the United
States initially adopted an isolationist stance. It took German
submarine attacks on neutral shipping, including American
shipping, to change public sentiment. In 1917, two crucial
events altered the course of the war. Russia withdrew from
the war as the Bolshevik Revolution overthrew the Russian
Czar. In the same year, the United States entered the war on
the side of the Allies. With Russia out of the picture, Germany
turned its main efforts to the Western Front. But it was too
little too late. The arrival of the Yanks decisively shifted
the balance of power in favor of the Allies.
At least in America, the Great War, as it was called, was
heralded as the war to end wars. But postwar disillusionment
set in quickly. A major aspect of that disillusionment was
the failure of Wilson to establish a League of Nations. The
initiative was doomed when the U.S. Senate refused to ratify
the treaty. After that, the League was basically a party where
nobody showed up.
Another important side effect of the war took place in India. As
the British continued to impose wartime measures on India in
1919, riots broke out and the British suspended civil liberties.
Mohandas Gandhi (18691948) organized resistance under his
philosophy of satyagraha. His leadership of nonviolent resist-
ance eventually led to Indian independence in 1947.
The foundations for further conflict in Europe began as
the Treaty of Versailles (1919) divided up pieces and parts
of Europe while imposing harsh demands for reparations
on Germany. Then came the worldwide Great Depression.
Lesson 5
199
Hard times rolled over America and Europe. Labor unrest
grew everywhere and, in Europe, desperation helped lay the
groundwork for the rise of totalitarian ideologies. Out of
Russia, Communism was one of them. In Italy, Benito
Mussolini (18831945) rose to power in Italy in 1922. He
declared the birth of fascism, a movement partly based on
gaining the support labor on the one hand and powerful cor-
poratists in the other. To the north, a charismatic firebrand,
Adolph Hitler gazed at Mussolinis fascist movement with
admiration. He adopted his version of fascism in the same
manner as Mussolinihoodwinking labor and negotiating
with powerful industrialists. Hitlers agenda called for declar-
ing eternal enmity against Communists and the Jews, who
he blamed for the humiliation of Germany after the First
World War. In his view, his National Socialist German
Workers PartyNazi Party for shortwould lead Germany
into a new world order and establish a thousand-year Third
Reich.
Meanwhile, in Japan, as Japanese armies were busy trying to
conquer China, the cult of Shinto was co-opted to give rise to
a resurrected samurai mythology. In that context, Japanese
militarists insisted that Japan must gain control of needed
natural resources, such as petroleum and rubber, by taking
control of British and Dutch colonies in Asia. Of course the
fact that the United States had imposed an embargo on
exporting vital resources to Japan didnt help matters much.
After he became German Chancellor in 1933, the preface to
the rise of Hitlers Reich amounted to grabbing pieces of
Europe that he claimed as ethnically German lands, such as
Bohemia in Czechoslovakia, the Sudetenland, and Austria.
This was the era of appeasement as European leaders tried
placate Hitler by conceding to his demands. The onset of war
in Europe began in 1939 as the German Wehrmacht rolled
into Poland. Britain had a mutual defense treaty with Poland
and the new Prime Minister, Churchill, demanded that
Parliament declare war on Germany. The onset of war in the
Pacific began on December 7, 1941, with a devastating
Japanese aerial attack on the U.S. naval fleet anchored in
Pearl Harbor at Oahu, Hawaii.
World History
200
The Second World War was a terribly complex and brutal
global drama that ended in the deaths of around 60 million
people. The Allied victory in Germany was, in large measure,
a result of incredibly bloody contests between the German
Wehrmacht and the Red Army under Josef Stalin. Its not at
all clear that the Allied invasion of Europe in 1944 would
have succeeded had Germany not attacked Russia. The
Germans met devastating defeats in the snowy ruins of
Stalingrad and, even more decisively, at Kursk, in the largest
tank battle in history. The scale of the contest in Russia can
be deduced from a single number: Russian losses exceeded
20 million.
The victory over Japan, which came several months after the
victory in Europe, was probably inevitable. The Japanese
high command never had many illusions about that. But
after the atomic bomb attacks on Hiroshima and Nagasaki in
August 1945, the Japanese had finally had enough of war.
The sidebar on page 728 of your textbook summarizes key
dates and events of the World War era. The Making Connections
feature on page 729 can be helpful to you in sorting out the
actors, conflicts, causes, and the outcomes of the World War
era. Map 28.2 on page 730 provides an overview of World
War II conflict boundaries across the globe.
The Cold War Era, 19451991
At the end of the World War II, Stalin was anxious to secure
the Soviet Unions western boundaries. To that end, the
USSR established control of Poland, Czechoslovakia, Hungary,
Romania, Bulgaria, and the eastern portion of Germany
under the Warsaw Pact. Berlin was partitioned among the
Allies. The fact that East Germany surrounded Berlin became
an immediate and ongoing source of tension between the two
new world superpowersthe United States and the Soviet
Union. In 1947, in a famous speech delivered in Missouri,
Winston Churchill declared that an Iron Curtain had fallen
across Europe. The Cold War was underway.
Lesson 5
201
Under the guidance of General Douglas MacArthur, Japan
was transformed from a defeated militarist state into a pro-
gressive demilitarized constitutional democracy and a firm
ally of the United States.
In 1947, the United States initiated the Marshall Plan,
named after then Secretary of State George Catlett Marshall
(18801959). The strategic aim of the plan was to rebuild the
European economy in order to forestall the advance of
Communist interests in Europe. It was, overall, hugely
successful.
By 1947, Mao Zedong had established the Communist Peoples
Republic of China. Neither the United States nor the Soviets
greeted the development with enthusiasm. Stalin and Mao
didnt see eye-to-eye.
In 1949, the USSR exploded an atomic bomb. The Cold War
arms race began in earnest.
In 1950, forces from Communist North Korea crossed the
38th parallel on the Korean peninsula into semi-Westernized
South Korea with the objective of uniting all of Korea under
Communist rule. The U.N.-sponsored armed resistance was
led by the United States. After communist China came to the
aid of North Korea, the war ended in a stalemate along the
tense boundary marked by the 38th parallel.
In China, the abject failures of Maos economic reforms and his
brutal Cultural Revolution markedly weakened Chinas economy
and Chinese society. In this context, between 1969 and 1972,
U.S. President Richard Nixon (19131994) managed to persuade
Chinese leaders to endorse trade agreements linking the Chinese
economy to trade with the West. The upshot of that would be
the advent of Chinese-style capitalism.
In the wake of French expulsion from Indochina in the 1950s,
Vietnam was partitioned between a Communist North under
Ho Chi Min and a non-Communist South Vietnam under the
corrupt Diem brothers. Civil war broke out between north and
south. The Kennedy administration attempted limited interven-
tion on behalf of the south. However, after both Diems and
President Kennedy were assassinated (in 1962 and 1963,
respectively), the new U.S. president, Lyndon Johnson,
elected to expand the war. Richard Nixon continued that
agenda to the extent of illegally carpet-bombing Cambodia.
World History
202
The overall result was a 10,000-day war that sparked anti-
war protests across the land and ended in abject U.S. defeat
in 1975. Somewhere in excess of three million Vietnamese,
Cambodians, and Laotians diedabout two million of them
civilians. Some 58,000 Americans were killed.
The end of the Cold War was associated with two world lead-
ers. In the Soviet Union, Mikhail Gorbachev (b. 1931) saw
the writing on the wall. He initiated policies to dismantle the
inefficient command economy and eventually agreed to demo-
cratic reforms demanded by the USSRs ethnic minorities.
In the United States, Ronald Reagan (19112004) had been
busy accelerating the arms race, trying to bankrupt the Soviet
Union in the process. After previously declaring the Soviet Union
the evil empire, he met face-to-face with Gorbachev to rethink
U.S. interests. Accords were reached. What followed next was
both inevitable and shockingly unexpected. In 1989, the Berlin
Wall collapsed and the Warsaw Pact states dismantled their alle-
giances to the Soviet Union. By 1991, the Soviet Union was
history and the Russian Republic was born.
Map 28.3 on page 733 of your textbook offers a global
overview of the alliances of the Cold War.
Decolonization
Two major themes played out around the globe in the wake
of World War II. Both of them would complicate the grand
chessboard objectives of the Soviet Union and the United
States. First and foremost was wave after wave of decoloniza-
tion. Second was a trend toward democratic reform.
Despite the benefits of decolonization, the resulting
problems have been many and continuous. The discussion
on pages 738741 details many of the problems of decoloniza-
tion in different parts of the world.
In addition, the world experienced two so-called revolutions:
the green revolution and the information revolution, which
both had huge global impacts. As a result of the green revo-
lution, grain production swamped demand and grain prices
plummeted. Agrarian third world states bore the brunt of
falling commodity prices. Beginning in the 1970s, an elec-
tronic information revolution introduced Western states into
Lesson 5
203
a postindustrial age. In this context, manufacturing and
other traditional industries were displaced to undeveloped or
developing states as Western multinational corporations
sought cheap labor. As a result, both within and outside of
the former imperial states, the gap between haves and have-
nots has widened tremendously.
Map 28.4 on page 740 of your textbook offers a global look
at decolonized regions in Africa and across South and
Southeast Asia. The map is color-coded to give you an overall
timeline from before 1950 to post-1956.
The New World Order
Were now exploring an era of history that will no doubt bend
and sway with revisionist perspectives. The new world order
emergent since the end of the Cold War offers good news and
bad news. The good news includes global activism aimed at
protecting human rights. That trend, in turn, has accompa-
nied the termination of autocratic regimes in favor of new
democracies around the world. Also in the good-news
column, with some reservations, has been the rise of the
European Union (EU). The initial economic boom brought
about by loosening of trade and travel barriers among
Western Europes stronger economies was good news.
However, problems have arisen as less economically vibrant
states, such as Italy, Greece, Spain, and Portugal, have
joined the EU.
The bad news has come from ethnic tensions in the devel-
oped nations and the rise of antidemocratic forces in parts of
the Muslim world and Latin America. Unfortunately, often in
the interests of business imperialism, U.S. foreign policy has
supported a number of antidemocratic regimes in the Middle
East and Latin America. Another item in the bad-news col-
umn derives from migrations from poor to rich countries.
In Europe, Muslim immigration has created ethnic tensions.
In the United States, Latin American immigration, especially
World History
204
from Mexico, has created ethnic stereotyping and resent-
ments on the part of whites. In both cases, the issues have
been complicated by nationals willing to exploit immigrant
labor.
Conclude your study of this chapter by examining the
Chronology sidebar on page 745 of your textbook.
ASSIGNMENT 29: THE PURSUIT
OF UTOPIA: CIVIL SOCIETY IN
THE TWENTIETH CENTURY
Read this assignment. Then read Chapter 29, pages 748773,
in your textbook.
The Context of Atrocities
The twentieth century was a century of war. Periods of war
are, and always have been, punctuated or accentuated by
atrocities. However, as technological and industrial progress
glittered and a sunny utopian world seemed attainable,
people killed each other at unprecedented rates. The chapter
opens with an account of the Rape of Nanjing by Japanese
soldiers in 1937.
The most systematic and ruthlessly efficient genocide of the
twentieth century was the Nazi Holocaust attendant to World
War II. Around 12 million people were slaughtered, including
homosexuals, the mentally disabled, political dissenters,
Gypsies, and some 6 million or more European Jews. The
Armenian genocide by Turks in 19151917 resulted in the
murder of 1.5 million men, women, and children and has
never been officially repudiated by the Turkish government.
In the aftermath of the Vietnam War, the Khmer Rouge
regime in Cambodia slaughtered 1.7 million men, women,
and children between 1975 and 1979. The Khmer Rouge
continues to have a political presence in Cambodia. Late
twentieth-century genocides, styled as ethnic cleansings,
included the Rwandan genocide of 1994 (800,000 dead)
and ethnic cleansing in Bosnia-Herzegovina (200,000 killed
between 1992 and 1995).
Lesson 5
205
Map 29.1 on pages 752753 of your textbook offers a global
overview of genocides and atrocities between 1900 and the
present. The topic is troubling at best, terrifying at worst. But if
civilized societies are to survive, the causes and nature of these
hellish events must be better understood and effectively pre-
vented. An inset map shows the locations of Nazi concentration
camps and the relative percentages of Jewish populations
exterminated in different European countries.
The Encroaching State
Efforts to understand the horrific social pathologies of the
twentieth and the early twenty-first centuries have, at best,
been incomplete or inconclusive. However, some general
observations appear to be relevant.
The power of the state has increased steadily over the twentieth
century. In that context, state control over economic policies,
education, medicine, law, and public information have
inclined people to accept, or at least tolerate, collectivization
and regimentation. This sort of phenomenon was clearly
evident in Hitlers Germany, the Soviet Union, and Maos
China. In the United States, the power of the state was
rallied to cope with the Great Depression (19291933).
Under President Franklin Roosevelt, a New Deal was aimed
at suppressing the predations of laissez-faire capitalism
through programs like Social Security and public works
programs designed to create jobs. Through the 1950s and
1960s, the U.S. federal government was also active in
addressing the era of social reform that began with the
civil rights movement. Integration got federal support under
Eisenhower. A bit later, as hippies brandished flower power,
the Johnson administration oversaw the passing of civil
rights legislation and a federally funded War on Poverty
Unplanning Utopia: The Turn
toward Individualism
In the last quarter of the twentieth century there was a
marked rise in individualism. Contributing to this shift were
the rise of postindustrial economies and the information
World History
206
economy, which resulted in the creation of a knowledge
class. An additional factor was the global turn toward
conservatism. The impact of the rise of individualism was
dramatic, particularly with regards to the generation gap
that emerged.
Counter-colonization and
Social Change
The era of decolonization that followed World War II was
accompanied by an immense wealth gap between undevel-
oped and developing states. At the same time, as population
levels soared in the nondeveloped and marginal states,
populations in Western and Westernized states was either
stabilizing or declining. In this context, migration from the
so-called Third World accelerated. In the United States,
migrants poured in from India, the Philippines, and Southeast
Asia. The heaviest drift toward multiculturalism in the United
States resulted from Latin American immigrants, mainly
from Mexico.
Map 29.2 on page 758 of your textbook gives you a global
overview of streams of migration and the relative impact of
migrant populations in different regions ca. 2005.
Globalization and the World Economy
Just as global populations were being intermingled, so were
national economies. Today, a product marked Made in
Japan may actually include parts and pieces manufactured
in India, Indonesia, and Taiwan. Major Japanese automakers
have factories in Kentucky and Alabama. An enormous per-
centage of American consumer goods are now manufactured
in China. Giant corporations like Coca-Cola (headquartered
in Atlanta) operate through subsidiaries all over the world.
McDonalds restaurants are popular in Paris and Moscow. In
the new global economy, concentrated wealth doesnt respect
international boundaries. The largest concentration of billion-
aires is in Moscow. In this context, access to global markets
can spawn overnight corporate success in places far from
New York or Tokyo. Nokia mobile phones out of Finland are
used around the globe.
Lesson 5
207
Map 29.3 on page 764 of your textbook show you international
trade flows as of 2004.
Culture and Globalization
Particularly as an effect of the World Wide Web and the Internet,
anybody, anywhere can be found in a computer chat room with
anyone, anywhere. Otherwise, the global influence of American
media is global. Hollywood films are popular from Japan to
Slovakia to . . . you name it. In this light, perhaps the most
problematic impact of media-driven cultural diffusion is the
spread of consumerism. Your textbook definition of consumerism
is on the money: Consumerism [is] . . . a system of values
that puts consumption and the ownership of consumer goods
at or near the top of social values [like] . . . social obligations,
spiritual fulfillment, or moral qualities.
Study Figure 29.1 on page 766 of your textbook to consider the
accelerated upward curve of worldwide telephone lines and
Internet users. Spend some time with the Globalization box
on page 767 to check out technological and economic-political
events between the 1950s and 2001.
Secularism and Religious Revival
Secularism can be thought of as any kind of worldview that
puts reason above feelings and solutions above sentiments.
Paradoxically, secularism is largely based on interpretations
of science that support materialism. Materialism maintains
that reality is simply matter and energy governed by random
chance. Nothing is real that cant be weighed and measured.
Thus, consciousness itself is reduced to molecules-in-motion.
The paradox here is revealed at the leading edges of actual
science. Maybe youll recall the brief discussion of the nature
of modern physics and the discovery of such things as non-
local connection. Heres another example of science echoing
ancient ideas about humankinds apparent spiritual dimen-
sion. It turns out that the universal constants that make life
human life possible are uncannily and precisely balanced
throughout the universe. In that light, a hypothesis called
World History
208
the anthropic principle asserts that the universe is arranged
to allow the emergence of human existence. In the words of
one astrophysicist, The universe saw us coming!
Of course, it may be quite a long time before science and reli-
gion share coffee and gossip. In the meantime, the historical
actuality of reactions to secularism has taken the form of
religious revivals. Some of these revivals have found ways to
reconcile science and religion. But, overall, those have been
marginal developments. More typical religious revivals have
turned back to traditional values of organized religions
informed by political agendas. In the Muslim world, religious
revivals in states like Iran are largely focused on rejecting
Western hegemony and Western materialism.
In the United States, many of the loudest voices of religious
revival have been marked by anti-intellectualism and a radi-
cal rejection of science. So, while the reaction to secularism
and materialism may echo the stirring of deep human long-
ings for a meaningful universe, the wholesale rejection of
science by some fundamentalists is problematic. For one
thing, scientific understanding is required for understanding
and coping with our current era of global climate change.
For another, science devoted to the common good can be a
faithful servant as opposed to a cold and heartless master.
Carefully study the Chronology sidebar on page 771 of your
textbook.
ASSIGNMENT 30: THE
EMBATTLED BIOSPHERE: THE
TWENTIETH-CENTURY
ENVIRONMENT
Read this assignment. Then read Chapter 30, pages 774796,
in your textbook.
This chapter will introduce you to a host of environmental
issues that are now shaping and will continue to shape
historical conditions for the foreseeable future.
Lesson 5
209
The lead-in to the chapter discusses the problems of the
African state of Burkina Faso. Located on the margins of the
Sahara, it faces environmental problems that have been part
of this region for thousands of years. But most of the local
problems have to do with human actions resulting from too
many people trying to live off land being depleted by over-
farming and overgrazing. Further, though, the problems of
such impoverished places around the planetas youll see
can only be understood in terms of globalization and its
worldwide effects.
Map 30.1 on page 777 of your textbook shows a stylized world
map wherein the area of nations or regions is proportional to
the size of their population. Note that most of the worlds
people line in China, India, and immediately adjacent nations,
like Pakistan and Bangladesh. Map 30.2 on page 778 features
a color-coded world map. The color codes show you the com-
parative wealth of global regions. Note that the wealthiest
nations are in North America, Western Europe, and Australia.
Fuel Resources
From the outset of the Industrial Revolution, ever-increasing
energy demands have depended on fossil fuels. Coal was the
basic energy source during the age of steam. Petroleum
became a critical energy source with the arrival of the inter-
nal combustion engine. Coal is still a major energy source,
especially in developing nations like China.
The burning of fossil fuels releases carbon dioxide into the
atmosphere. While carbon dioxide is a normal component of
air, its typical level at the end of the Ice Age was around
200 parts per million (ppm). By the 1880s, the level was
280 ppm. Presently, the level is around 380 ppm and rising
steadily. The level of carbon dioxide in the atmosphere is
directly proportional to the capacity of the atmosphere to
retain heat from solar radiationa process called the
greenhouse effect. As you may or may not know, the scientific
community worldwide has, for some time now, made it clear
to all with ears to hear that the primary source of increased
carbon dioxide in the atmosphere is human activity. Youll
want to keep this in mind as you pursue your textbooks
World History
210
discussion of the global environmental situation and the
uncertainties we all face. For example, can the development
of renewable and sustainable sources of energy offset the ris-
ing energy needs of more than six billion people on a planet
with finite resources?
Food Output
The best laid plans of mice and men often go askew, noted
the poet Robert Burns. Modern efforts to increase food pro-
duction appear to feature a catalogue of best laid plans.
Dams, such as the Hoover Dam in the United States and the
Aswan Dam in Egypt, may be engineering marvels, but the
end result of dam building has been the disruption of agri-
cultural production and an increase in desertification.
Fertilizers became inexpensive and plentiful after the German
chemist Fritz Haber invented a process for extracting nitro-
gen from the air in 1909. Nitrogen is the foundation of
agricultural fertilizers. The problem is that nitrogen from
synthetic fertilizers is expelled in runoff from farms, polluting
groundwater reservoirs (called aquifers). At present about
two-thirds of U.S. aquifers are contaminated from fertilizer
runoff.
The green revolution was a convergence of innovations in the
uses of hybrid seeds, fertilizers, and pesticides. The increase
in food production saved millions. However, surpluses of
wheat and rice swamped global markets. As grain prices went
down, farmers took a beating. Following on the success of the
green revolution, some scientists have trumpeted the promise
of genetically modified (GM) foods. However, GM foods must,
by their nature, alter their environment. As this happens,
unforeseen side effects are inevitable. Fearing threats to
ecological sustainability, European nations have resisted the
introduction of GM seeds and the importation of GM foods.
Urbanization
Our era has seen the rise of what might be called mega-
cities. The population of both Mexico City and Sao Paulo
are pushing toward 20 million. The phenomenon is global.
Lesson 5
211
Your textbooks analysis suggests that unsustainable urban
growth may begin to recede through the opening of new
markets for exotic foods. Perhaps this will be the case. But
for the present, looked at in the broadest sense, Earth has
become what demographer James Davis calls a planet of
slums.
Map 30.3 on page 785 of your textbook provides an overview
of relative levels of urbanization in different regions.
As for the Rest . . .
Historical analysis tends to get sharper when aided by hind-
sight. In the final discussions offered in your textbook, the
author cant have that benefit. As pundits have remarked, its
difficult to make predictions, especially about the future.
Your textbook discussion will introduce you to apparent
relationships between burgeoning human populations and
ecological destruction. In a nutshell, human activity is
destroying other species at an alarming rate. Some climate
scientists and ecologists have begun referring to the current
level of species die-off as the anthropocentric, or human-
centered, extinction eventthe end of what geologists call the
Holocene Era. The last such event, some 65 million years
ago, appears to have resulted from Earths encounter with an
asteroid in the vicinity of the Yucatan Peninsula.
The author of your textbook offers a basic point for you to
ponder. Namely, regardless of what humans do or fail to do,
there are environmental and ecological variables over which
human have no control. Beyond metaphysical speculations to
the contrary, humans didnt make the rules of ecology, genet-
ics, or physics. Those rules are built into what you might call
Earths programming. Again and again, Earth has adapted
to radical environmental changes over the roughly four billion
years of its existence. It will continue to do so. If a significant
proportion of Earths human population succumbs to some
new plague-like pathogen, Earth will adapt. If current
declines in the oceanic food web become lethal for many of
the planets species, Earth will adapt.
World History
212
Perhaps, as the futurist Arthur C. Clarke suggested, our
impulsive, predatory, juvenile species will find new paths to
sustainability and cooperation. Well greet and pass the
threshold of our childhoods end. But if we dont, and
humankind passes from the scene, Earth will remain and
Earth will adapt.
The Chronology sidebar on page 793 of your textbook will
help you review key events and dates related to the current
environmental crisis.
Now, review the material youve learned in this study
guide as well as the assigned pages in your textbook for
Assignments 2330. Once you feel you understand the
material, complete Self-Check 10. Then check your answers
with those provided at the end of this study guide. If youve
missed any answers, or you feel unsure of the material,
review the assigned pages in your textbook and this study
guide. When youre sure that you completely understand the
information presented in Assignments 2330, complete your
examination for Lesson 5.
Self-Check 10
Indicate whether each of the following statements is True or False.
______ 1. The secularism imposed by the Shah of Iran during the 1960s led to an Iranian
revolution that brought the Ayatollah Khomeini to power in 1979.
______ 2. In India, the expansion of scientific education and research was encouraged and even
funded by the British government.
______ 3. During the period of counter-colonization after World War II, population growth in the
former colonial regions declined while populations in develop countries increased.
______ 4. By the year 2000, the most commercially popular art focused on fantasies adaptable to
computer-generated imagery.
(Continued)
Lesson 5
213
Self-Check 10
______ 5. The Second World War encouraged regimentation and collectivism.
______ 6. During the era of globalization, the United States was able to declare free trade where
it was in its interest to do so and to impose tariffs where it wasnt.
______ 7. During the Cold War, China allied with Russia as an enemy of the West.
______ 8. The League of Nations charter gained only limited influence after World War I, when it
was finally ratified by the U.S. Senate in 1920.
______ 9. Having abandoned militarism, Japan ended up with the highest per capita income in
the world.
Fill in the blank with the correct term.
10. The school of philosophy called _______ was invented by William James, an American
apologist for Christianity, who maintained that a concept could be considered true if it could
be used to produce socially useful results.
11. In World War I, the most powerful states of the central powers were Germany and the
Austro-Hungarian Empire. The three most important Allies were the United States, Britain, and
_______.
12. The idea that Biblical truths cant be questioned by critical inquiry or scientific evidence is
called _______.
13. In the years leading to World War II, Japan had become devoted to militarism in the context
of its war with _______.
14. In 1929, the use of a powerful telescope by astronomer Edwin Hubble proved that the visible
universe was _______, an observation that would lead eventually to the Big Bang theory.
15. _______ can be defined as a system of values where people put having and buying goods
above social obligations, moral values, or spiritual fulfillment.
16. After World War II, a generous foreign aid program called the _______ Plan was launched to
rebuild Western Europe.
17. The doctrine that cultures must be evaluated by their own standards and not those of outside
observers is called cultural _______.
(Continued)
World History
214
NOTES
Self-Check 10
Answer each question in not more than four complete sentences.
18. What are some basic reasons why Western science was ascendant even as much of the world
seethed with resentment at Western hegemony?
__________________________________________________________
__________________________________________________________
__________________________________________________________
__________________________________________________________
19. As briefly as you can, explain how socialism can be associated with the rise of fascism in
Italy and Germany.
__________________________________________________________
__________________________________________________________
__________________________________________________________
__________________________________________________________
20. What is the greenhouse effect, and why does it pose challenges to human life?
__________________________________________________________
__________________________________________________________
__________________________________________________________
__________________________________________________________
Check your answers with those on page 224.
Self-Check 1
1. False
2. True
3. True
4. True
5. False
6. False
7. True
8. alluvial
9. horse
10. Africa
11. beans
12. shaman
13. In places like the Middle East, changes in climate favored
the growth of grasses that bore grains. Gatherers who
paid attention to plants could have experimented to see
if they could plant and harvest certain type of grasses,
like wheat, to increase their food supply.
14. As the ice sheets retreated, early hunters in the tundra
were able to hunt and feed on herds of animals like
caribou, horses, and Arctic hares. Animals of the tundra
stored ample, highly nutritious body fat. Therefore, Ice-
Age hunters were better fed than most people would be
until modern times.
Self-Check 2
1. True
2. True
3. False
4. False
5. False
6. True
215
A
n
s
w
e
r
s
A
n
s
w
e
r
s
7. True
8. True
9. Phoenician
10. Mesopotamian
11. Brahman
12. Greek
13. Egypt
14. The Hittites organized farmers and herders into a unified
economic system. Livestock provided fertilizer for farmers,
and farmers werent enslaved, as was the case in other
places. Independent peasants raised healthy families
that provided manpower for Hittite armies.
15. Logograms were used in Chinese and Egyptian writing.
Logogram figures are like little pictures or icons that help
readers and writers represent an object or a concept.
16. In Shang China, the king was, above all, a mediator with
the gods. To manage this duty he depended on oracles
and diviners. Animal bones were heated until they
cracked. The diviners then interpreted the cracks,
inscribing notes on the bone. In this way, the king was
presumed to be able to foretell the future and make
decisions accordingly.
Self-Check 3
1. False
2. False
3. False
4. True
5. True
6. False
7. False
8. True
9. rationalists
Self-Check Answers
216
10. Persian
11. Jainism
12. Plato
13. steppe
14. Legalists
15. concrete
16. The Greek philosopher Aristotle maintained that valid
arguments could be made by proposing two premises
that, if self-evidently true, could establish a valid conclu-
sion. In a famous syllogism, the first premise is, All men
are mortal. The second premise is, Socrates was a
man, and the concluding statement is, Socrates was
mortal.
17. Seafaring in the Indian Ocean was encouraged by its
seasonally variable monsoonal wind system. In the
winter, the wind currents are consistently northeasterly.
During the summer, the wind currents shift to blow
consistently to the south and west. This fact allowed sea
voyagers to get to where they wanted to go, from Africa
to Southeast Asia, and still get home again.
Self-Check 4
1. True
2. False
3. True
4. False
5. True
6. False
7. True
8. True
9. Sufis
10. Justinian
11. Constantine
Self-Check Answers
217
12. Polynesian
13. Hinduism
14. Quran
15. Byzantine
16. Theravada Buddhism maintained that the perfection of
the soul could be achieved in tiny steps over many life-
times. Its dominant in India. Mahayana Buddhism
teaches that perfection can be reached in one lifetime.
Its dominant in China.
17. In both Rome and China, constant plots, intrigues, and
violence surrounded the selection of imperial heirs.
Eunuchs couldnt sire families. Therefore, in both Rome
and China they were seen as perfect servants because
they couldnt have children who would become pre-
tenders to the throne. However, in the end eunuchs
created factions and did sometimes rise to power.
18. The broad expanse of the Sahara Desert isolates sub-
Saharan Africa from the Mediterranean and from ready
access even to the Middle East. Further, sub-Saharan
Africa has very few navigable rivers, and many inland
rivers never reach the sea at all. Also, dense, malarial
tropical forests obstruct inland communication.
Self-Check 5
1. True
2. True
3. False
4. False
5. True
6. True
7. True
8. Angkor
9. India
10. shoguns
Self-Check Answers
218
11. commune
12. trade (commerce)
13. Original European technological advances of this period
included windmills, clocks, and ground lenses. Imported
innovations included paper, the compass, firearms, and,
later in this period, blast furnaces. The latter permitted
the adoption of heavy iron plows that made it easier to
farm marginal lands.
14. Latin was the language of the West; Greek was the
language of the East. For that reason alone, Christian
creeds, dogmas, and practices diverged. Meanwhile, as
the Pope at Rome claimed sovereignty over European
states, the Pope wasnt recognized as the supreme head
of the Church. In Byzantium, Russia, and elsewhere in
the east, church leaders acceded to the power of the
state.
Self-Check 6
1. True
2. True
3. False
4. True
5. True
6. False
7. False
8. False
9. Mali
10. climate
11. millenarian
12. hurricane
13. Francis
14. chivalry
15. Russian
Self-Check Answers
219
16. Delhi
17. Bar Sauma was a Nestorian Christian and Chinese subject.
At a time when many from the West were taking long
treks across Mongol-policed Silk Roads to the East, Bar
Sauma was an exception. He undertook a documented
exploratory journey to the West. After getting as far as
Muslim Azerbaijan, he was appointed as the Mongol
ambassador to the Christian kingdoms of the West,
traveling as far as Rome and Paris.
18. Zimbabwes were fortified, stone-built administrative centers
in East Africa. They served the purposes of African societies
looking to benefit from Indian Ocean trade. The most
famous of the zimbabwes was the Great Zimbabwe
located not far from the Limpopo River in Mozambique.
The golden age of Zimbabwe extended from the late
fourteenth into the fifteenth century.
19. Zen was a branch or school of Buddhism that focused on
burning away the ego through direct mystical experi-
ence. It gained influence in Japan as Chinese Buddhist
monks fled Mongol violence on the mainland. Just as
important, especially under Shogun rule, Zen appealed
to the ancient code of the warrior class, the samurai.
Self-Check 7
1. False
2. True
3. False
4. True
5. True
6. True
7. False
8. True
9. tobacco
10. maroons
11. Amsterdam
Self-Check Answers
220
12. sugar
13. German
14. realpolitik
15. country
16. Hindu
17. In some cultures, theres an inclination to cooperate with
and defer to outsiders. This was the case in Mexico after
the arrival of the Spaniards. Native peoples readily
turned to Spanish friars as arbiters of disputes, healers,
and holy men. This stranger effect helped the Spanish
rally support from tributary tribes under Aztec oppres-
sion and successfully bring down their empire.
18. In the Americas, the introduction of herd animals such
as sheep, cattle, and horses led to colonial economies
based on herding. In Brazil, for example, land for grazing
encroached on lands occupied by sedentary native
peoples. In sub-Saharan Africa, in regions of the Sahel
especially, pastoral economies were dominant in states
like Songhay. In this context, the Moroccan conquest of
Songhay in the late sixteenth century displaced pastoralist
imperialism for only a few decades.
19. In Spanish colonies, the first generation of Europeans
born in the New World tended to identify with the New
World, not Spain or Europe. Intermarriage was accepted,
and the Creoles took pride in having Native ancestry,
particularly if they could claim descent from indigenous
royalty. A theme in Creole consciousness was the superi-
ority of New World colonial cultures over their European
counterparts.
Self-Check 8
1. True
2. True
3. True
4. False
5. False
Self-Check Answers
221
6. False
7. True
8. False
9. will
10. Europe
11. Fulani
12. Ricardo
13. equator
14. Sioux
15. telescope
16. National
17. The Royal Kew Gardens, along with similar gardens
in Paris, Madrid, and the University of Leiden in the
Netherlands, served as botanical laboratories for
studying and breeding plants from around the world.
Europe had an advantaged position in adapting plants to
different environments because their colonial territories
encompassed a wide variety of climate conditions.
18. There was a widespread belief in progress; human nature
assured that ever more technological, scientific, and
social progress was inevitable. A doctrine of laissez-faire
maintained that free trade across borders with minimal
restrictions imposed by governments was better than the
older system based on states hoarding wealth. Ideals like
All men are created equal led to a surge of feminist
ideals based on the assumption that all men and women
are created equal. Finally, the power of the church and
of religion was questioned and repudiated.
Self-Check 9
1. False
2. False
3. True
4. False
Self-Check Answers
222
5. True
6. True
7. True
8. False
9. True
10. proletariat
11. heating
12. coolies
13. public
14. market
15. Darwinism
16. self-strengthening
17. For the most part, people had no choice. Mechanization
of agriculture meant that fewer people were needed
to produce food. Global specialization shifted the
production of food and raw materials away from the
industrializing countries. Meanwhile, factory owners in
industrial countries were competing with each other to
attract factory workers.
18. Although the Maori of New Zealand managed to fight off
British rule nearly thirty years, the case of Ethiopia was
unique. Beginning in the late nineteenth century, the
Ethiopian Emperor, Menelik III was busy modernizing
and strengthening the black Empire. The empire not only
expanded, its forces managed to utterly defeat an Italian
invasion. Ethiopia was the only African state to stand up
to and evade the European scramble for Africa.
19. Jeremy Bentham was opposed to socialism. He also
rejected authoritarian law-and-order ideologies based on
a negative view of human nature. Bentham felt that the
social good could be defined as a surplus of happiness
over unhappiness. In that light, he proposed institutional
reforms that would produce the greatest happiness for
the greatest number of people.
Self-Check Answers
223
Self-Check 10
1. True
2. True
3. False
4. True
5. True
6. True
7. False
8. False
9. True
10. pragmatism
11. France
12. fundamentalism
13. China
14. expanding
15. Consumerism
16. Marshall
17. relativism
18. Western science was adopted because it worked. It
transformed communications, transportation, business,
education, and even how people spent their leisure time.
Also, Western science offered knowledge that accounted
for observation, provided an experimental method for
testing suppositions, and permitted accurate prediction
within definable limits.
19. Socialism has had both democratic centrist and militant
schools. With the rise of the Soviet Union after 1917, mili-
tant socialism and brutal authoritarianism characterized
the regime of Josef Stalin. In Italy, Mussolini invented
fascism after having been a militant socialist. In Germany,
lip service was given to socialism, but in both Italy and
Germany fascism was an alliance of the state with powerful
corporate interests, and communists were seen as archene-
mies in both fascist states.
Self-Check Answers
224
20. When solar radiation strikes the earth, a portion of it is
bounced back toward outer space. As the carbon dioxide
level in the atmosphere increases, less radiation is
bounced back toward space, and the lower atmosphere
absorbs more heat in whats called a greenhouse effect.
As carbon dioxide levels in the atmosphere have increased
rapidly since 1750, global warming and climate change
have accelerated, posing unpredictable threats to human
civilizations.
Self-Check Answers
225

You might also like